MCAT Missed Practice Questions

Réussis tes devoirs et examens dès maintenant avec Quizwiz!

One type of metabolic feedback loop that influences weight control involves the regulation of glucose levels in the blood. Which organ in the digestive system participates in this regulation by breaking down glycogen?

In order to regulate blood glucose levels, glycogen is made, stored, and broken down in the liver

The Na2CO3 used in Step 5 of Figure 3 is necessary to:

Na2CO3 acts as a Brønsted base. Until a proton is removed from the alkylammonium salt in Compound 4, it does not have the lone pair needed to be a nucleophile. As a nucleophile, Compound 4 performs the SN2 reaction that furnishes Compound 5.

What mass of NaCl (MM = 58.5 g/mol) was present in 100 µL of the PBS solution?

The answer to this question is A which can be obtained by multiplying the solution concentration, volume, and molar mass using the correct units to cancel: 58.5 g/mol × 0.1 × 10-3 mol/L × 100 × 10-6 L = 5.85 × 10-7 g = 585 × 10-9 g = 585 ng.

ribosome subunits in prokaryotes

30S and 50S

ribosome subunits in eukaryotes

40S and 60S

Which of the following phase changes is used to determine the heat of fusion for a pure substance?

Fusion means melting. Thus, a chemist can melt a solid or freeze a liquid to determine the heat of fusion of a pure substance.

Glu

Glutamate (E) Acidic -> - charge at pH 7 R group PI = 4.1

His

Histidine (H) Basic -> + charge at pH 7 Aromatic R group PI = 6

In the Bohr model of the atom, radiation is emitted whenever electrons:

In the Bohr model of an atom, energy is emitted only when an electron falls from a higher to a lower energy level. Thus, answer choice C is the best answer.

Pro

Proline (P) hydrophobic, nonpolar R group

In which of the following steps is the carbamate amide group cleaved?

The carbamate R2NCO2C2H5 in Compound 3 is cleaved by hydroxide to a secondary amine.

A drug that increases the risk of a tubal pregnancy is most likely to inhibit which one of the following actions?

The main reason for the occurrence of ectopic pregnancies is the failure of the fertilized egg to be transported from the oviducts to the uterus.

acetylation

addition of acetyl group

Social stigma is most commonly associated with:

an attribute that is devalued

Which event is LEAST likely to occur during apoptosis?

damage to nearby cells caused by lysis is a characteristic of necrosis, not apoptosis.

cardiac output

heart rate x stroke volume

The side chain of which amino acid can form a bond that is similar to a peptide bond?

lysine side chain can form isopeptide bonds by reacting with a carboxylic acid group, which is the same way that peptide bonds are formed.

beta decay

radioactive decay in which an electron is emitted. net effect = replace neutron with a proton

What quantity of NAG3 was required to reach the equivalence point in the titration?

the graphs in Figure 1 both imply a 1:1 mole ratio of NAG3 was added at the equivalence point and the solution contained 1.0 mL of 0.10 mM = (1 × 10-3 L)(0.10 × 10-3 mol/L) = 1.0 × 10-7 mol = 100 nmol.

Trp

tryptophan (W) hydrophobic, nonpolar R group Aromatic

Which type of enzyme removes the chemical groups that are added to proteins by kinases?

while kinase catalyzes the addition of a phosphate group to a substrate, the removal of the phosphate groups are catalyzed by a phosphatase

Which statement best illustrates the concept of anomie?

Anomie refers to a lack of social norms, which leads to a breakdown in the connection between an individual and their community. Thus, option B is correct. The other options address group decision making or indicate social control.

If the genetic and autoimmune theories of inflammatory bowel disease are true, then the gastrointestinal antigen being targeted by the immune system is probably on:

Assuming the genetic and autoimmune theories of inflammatory bowel disease are true, then the gastrointestinal antigen being targeted must be located on the surface of proteins encoded by the genes for the disease. Antigens are carried on the surface of cells, not on the chromosomes, DNA segments, or RNA.

Suppose that, under the experimental conditions employed, Compound 1 is "saturated" with cations. What graph depicts the expected change in rate of K+ transport from the "IN" to the "OUT" phases at pH 2 (Trial 1 conditions) as a function of increasing K+ ion concentration in the "IN" phase?

Compound 1 acts like a catalyst for cation transport across the organic phase. If Compound 1 is saturated with substrate, increasing the substrate concentration will not increase the rate. In a plot of velocity V versus [S] (classic Michaelis-Menten kinetic treatment) the rate of cation transport is at the limiting value, called Vmax.

Erythromycin interferes with protein synthesis by binding to which ribosomal subunit?

Based on the passage, erythromycin inhibits bacterial protein synthesis by binding to the 23S rRNA component of the large subunit of the bacterial ribosome. The large subunit of the bacterial ribosome has a sedimentation coefficient of 50 Svedberg (S)

periodic table trends

Moving L to R: atomic radius decreases, ionization energy increases, electronegativity increases. Moving TOP to BOTTOM: atomic radius increases, ionization energy decreases, electronegativity increases

Considering the structure of STN, what is the most likely mechanism for its entry into the cell?

The answer is C because the structure of STN shows sufficient planarity and hydrophobicity to pass through the membrane by simple diffusion

If a solution containing the compounds shown in Figure 4, is injected into a gas-liquid chromatograph, the first peak observed in the gc trace is attributable to which compound

The answer to this question is B because 2-methyl-2-butene will exhibit the lowest molecular weight and also the weakest intermolecular forces of attraction. This substance will therefore migrate the fastest and be the first peak in the gas chromatograph (gc) trace.

Which event is NOT a likely outcome of glucagon binding to its receptor? Increase in:

glucagon binding to its receptor and activation of its coupled G protein, activities of the adenylate cyclase and the protein kinase A, and level of cAMP are all increased. In contrast activation of the G protein promotes the dissociation of bound GDP and its exchange for GTP on the α subunit.

According to Erikson's theory of psychosocial development, which group of participants are most likely dealing with the challenges of generativity versus stagnation? Adolescents Young adults Adults

he answer to this question is B because, in Erikson's theory, the generativity versus self-absorption crisis occurs in mid-adulthood. Thus, the adults compose the only age group that is likely to experience this stage.

Compared to untreated WT mice, antibiotic treatment of WT mice is likely to result in:

if SCFA producing microbes were removed by antibiotic treatment, then GPCR43 would not be activated to reduce glucose uptake into adipocytes. increased volume in adipocytes

Which experimental approach would be LEAST effective to determine the localization of pRB within a cell?

tagging the rb transcript would be the least helpful since this is expected to be found in the cytoplasm where the ribosomes are located.

In humans, the characteristic tissue of which of the following organs is NOT derived from mesoderm?

the brain is part of the central nervous system, which is derived from ectoderm. Heart, kidney, and skeletal muscle are derived from mesoderm.

pentose phosphate pathway results in the generation of NADPH, which is utilized in reductive reactions.

because as observed in 1/V0 versus 1/[S] plot (Lineweaver-Burk plot), competitive inhibitors are the only inhibitors that do not alter the intercept on the y-axis of the plot and therefore the Vmax of the enzymatic reaction.

How would participants with Alzheimer's disease (AD) perform on the verbal fluency and negative priming tasks? AD will:

both schizophrenia and Alzheimer's disease are characterized by cognitive dysfunction particularly in tasks of verbal fluency and negative priming.

A protein with which properties will most likely have the largest negative net charge at pH 7?

a protein with a low pI would be negatively charged at pH 7. This protein, being anionic, would bind to an anion exchange column (eliminates choices C and D). Largest negative net charge implies the presence of a large quantity of negatively charged amino acids, allowing the protein to bind tightly to the column. A high concentration of NaCl would be required for elution (eliminates choice B).

self-serving bias

a readiness to perceive oneself favorably --> may attribute being tardy as due to a situational variable

Two gel electrophoresis analyses are performed on a sample of purified protein with unknown structure: SDS-PAGE (1 band appears) and SDS-PAGE under reducing conditions (2 bands appear). Which prediction about the protein is directly supported by these results? The protein:

a reducing agent is used during SDS-PAGE to cleave disulfide bonds. The appearance of 1 band on SDS-PAGE without the reducing agent and 2 bands on SDS-PAGE with a reducing agent, suggests that at least one disulfide bond is present in the protein and that the disulfide bond(s) hold(s) two separate subunits of different masses together. T

A rat receives food for pressing a lever on a variable ratio schedule. Which pattern of responding is most likely to be observed?

a variable ratio schedule leads to high response rates with no predictable pauses.

Which experimental approach(es) can be used to analyze the effect of ROS on the lifespan of yeast? Comparing the lifespans of: wild-type yeast versus yeast lacking antioxidant enzymes wild-type yeast versus yeast overexpressing antioxidant enzymes yeasts growing in the presence or absence of hydrogen peroxide

all listed options influence ROS levels in yeast and can be used to analyze the role of ROS in regulating the lifespan of yeast.

Which protein has the highest electrophoretic mobility in SDS-PAGE under non-reducing conditions? (Note: There are no disulfide interactions unless stated in the table.)

an SDS-PAGE gel that is run under non-reducing conditions, proteins 1, 3, and 4 will run as monomers. Protein 2 will run as a dimer because the disulfide bonds between Cys residues are not reduced. The running masses will then be: A = 32, B = 38, C = 25, D = 38. As the smallest one, Protein 3 will have the greatest electrophoretic mobility.

What event will most likely occur if Protein X is inserted into the inner membrane of mitochondria?

based on information from the passage, Protein X forms membrane-spanning channels that alter the permeability of the inner membrane, thereby dissipating the proton gradient across the inner membrane. C. The proton gradient across the inner membrane will dissipate.

A researcher compares the brain sizes of two groups of rats. Group 1 has been raised in impoverished cages with minimal access to socialization and exercise. Group 2 has been raised in enriched cages with regular access to socialization and exercise. This is best described as a study of

changes in brain size as a function of environmental influences is an example of neural plasticity.

In a follow-up experiment, researchers treated cells with 50 µM of C75 in the presence or absence of the free radical scavenger N-acetyl cysteine (NAC) and then measured cell viability. The most likely rationale for this experiment was to test whether:

free radical scavengers such as NAC mitigate the effects of reactive oxygen species (ROS), thereby indicating if ROS were involved in mediating the effects of C75.

The brain imaging technique used in Study 2:

functional magnetic resonance imaging (fMRI), which uses magnetic fields and radio waves to look at brain function and measures blood flow and not glucose.

Which transition produces a photon with the longest wavelength?

The separation between the energy level equals the energy of the photon. The energy of the photon is inversely proportional to the wavelength. The longest wavelength corresponds to the smallest photon energy, which corresponds to the transition between the closest energy levels.

How many molecules of NADH are produced from six molecules of glucose that undergo glycolysis?

12 each cycle of glycolysis produces two NADH molecules and requires an input of one glucose molecule.

alpha decay

A nuclear reaction in which an atom emits an alpha particle consisting of two protons and two neutrons. This increases the atomic number by 2 and the mass number by 4.

Which statement provides the LEAST likely explanation for why the researchers dropped the highly anxious participants from their sample?

A. Ethical committees do not allow research with participants who may have psychological disorders.

According to the information in the passage, the gas that evolves is:

According to the passage, the gas is given off when HCl is added to one portion of the slurry. HCl reacts with CO32- to form CO2(g).

What is the geometry of the hexafluoroaluminate ion (AlF63-)?

An ion consisting of a central metal ion and six ligands is expected to have an octahedral geometry.

Cys

Cysteine (C) Polar with neutral R group but uneven charge disulfide bonds no H bonding R group PI = 8.4

HIV (human immunodeficiency virus) is a retrovirus, an RNA virus that can insert itself into the human genome. This virus can reproduce in host cells because it contains:

For the virus to reproduce itself in host cells, its RNA must be converted to DNA, which requires the enzyme reverse transcriptase.

Which of the following expressions gives the amount of energy lost to friction by the toboggan and rider between points A and B?

Frictional forces dissipate an amount of energy (in joules) given by the product of the force (in newtons) times the distance (in meters) over which the force acts. The distance between A and B is given as l in the passage. For the toboggan, the product is (60 N) × (l). Choice B is the correct answer.

A resident of a famine area who appears undernourished and extremely emaciated has eaten only starches for the past 3 months. A urine analysis shows that a large amount of nitrogen is being excreted. This is most likely evidence of:

In starvation, the body uses up its stores of carbohydrate and lipids, and then begins to break down body proteins for metabolic energy. A byproduct of the metabolism of the amino acids from protein is nitrogen.

Inflation of the lungs in mammals is accomplished by:

Inflation of lungs in mammals is accomplished by negative pressure pumping action.

A glass fiber carries a light digital signal long distances with a minimum loss of amplitude. What optical property of glass allows this phenomenon?

Light can be carried along a distance within a transparent material by means of total internal reflection. Thus, C is the best answer.

Met

Methionine (M) hydrophobic, nonpolar R group

A circuit similar to that in Figure 2 is set up in which X = Pb, Xn+ = Pb2+, Y = Cu, and Ym+ = Cu2+. Will an oxidation-reduction reaction occur spontaneously?

Pb2+(aq) + 2e- → Pb(s) E°red = -0.127 V Reaction 1 Cu2+(aq) + 2e- → Cu(s) E°red = +0.339 V Reaction 2 A spontaneous reaction occurs when E° is greater than zero, and this occurs if the oxidation of Pb(s) is combined with the reduction of Cu2+(aq), resulting in a net E° of +0.466 V.

Which variant of DNA polymerase will most likely retain catalytic activity?

Since the side chain carboxylate groups bind the metal ions, the only variant that would retain this function is D429E.

How many ppm is 1%?

The abbreviation ppm stands for parts per million (parts). Since 1% is one part in 100 parts, the question becomes 1 is to 100 as × is to 1,000,000, where × is the unknown. For the equality 1/100 = ×/106, × = 104 or 10,000.

What is the electron configuration of the metal ion produced when Ca reacted with water in Experiment 1?

When Ca is added to water, the following reaction takes place: Ca(s) + 2H2O(l) → Ca2+(aq) + 2OH-(aq) + H2(g), producing a basic solution. The metal ion produced is Ca2+. Ca has the atomic number 20 and therefore has 20 electrons. Two electrons are removed to form Ca2+, so Ca2+ has 18 electrons. The electron configuration for Ca2+ is 1s22s22p63s23p6

A dopamine antagonist will most likely lead to:

decreased mobility and increased frequency of tremors in patients with Parkinson's disease.

How should the ideal gas equation be rearranged to most easily use the values from the experiment in the passage to calculate the gas constant?

memorize equations R = PV/nT

Melanosomes most likely move along microtubules that originate in and radiate from the:

microtubules are cellular structures that originate from centrosomes.

In humans, cholesterol is a precursor to:

testosterone

Which peptide sequence is most likely found in a transmembrane helix of a protein?

transmembrane helices are made up of mostly unbroken stretches of hydrophobic amino acids.

What is the approximate value of ΔS° for binding of NAG3 to HEW at 27°C?

ΔG = ΔH - TΔS. Solving for ΔS gives -(ΔG - ΔH)/T = -(-30,000 J + 50,000 J)/300 K = -67 J/K.

Which aspect of equality is considered a requirement in a meritocracy?

A meritocracy is when societal rewards, status, and positions are awarded to individuals based on their own ability and work (that is, merit). In order for a meritocracy to operate, everyone within the society would need the same opportunity to succeed, so that rewards are actually based (primarily) on merit.

In a nearsighted individual, the image of a distant object is focused:

A nearsighted (myopic) person can focus on nearby objects but cannot clearly see objects far away. The nearsighted eye has a focal length that is shorter than it should be, so the rays from a distant object form a sharp image in front of the retina. This condition is corrected by glasses with a diverging lens.

Coulomb

Amps/s

Gln

Glutamine (Q) Polar with neutral R group but uneven charge

Which of the following shows the electron configuration of chlorine in NaCl?

In NaCl, chlorine exists as the chloride ion. A chloride ion has 18 electrons with the electron configuration 1s22s22p63s23p6. Thus, C is the best answer.

In the schematic diagram shown in Figure 1, the Cd electrode is

In an electrochemical cell, oxidation always occurs at the anode and reduction at the cathode. Oxidation is a process in which electrons are lost. The passage states that reduction occurs at the silver electrode, which means the silver electrode is the cathode. Since reduction occurs at the silver electrode, the Cd electrode is the anode. That Cd is the anode is verified by Equation 3, which shows that oxidation occurs there (i.e., Cd is oxidized).

If red litmus paper is dipped into the Na2CO3 solution, it will:

In water, carbonate will undergo the following reaction: CO32-(aq) + H2O(l) → HCO3-(aq) + OH-(aq). Red litmus paper will turn blue in a base.

Consider the reaction rate described by the following equation. Rate = k[A]m[B]n The overall reaction is said to be second order when:

K is constant, m=1 and n= 1

Substituting residues in a peptide with which amino acid will most likely result in a peptide with an increased pI?

Lys is a basic residue, meaning it would contribute to a high pI. Glu is an acidic residue, meaning it would contribute to a low pI. Gln and Val are both neutral. They would not contribute to the pI.

The fundamental, resonant wavelength of a pipe open at both ends that is 1 m long and 0.1 m in diameter is:

Pipes and tubes have their resonant wavelengths when standing waves develop. An open pipe has its fundamental, resonant wavelength at twice the length of the pipe. Both ends have displacement antinodes (maximum amplitudes) with a node in the middle of the pipe. Thus, the pipe is half a wavelength long. The resonant wavelength is independent of the diameter of the pipe. 2

According to the passage, platelets are LEAST likely to contain

Platelets are cell fragments without nuclei and therefore would not be expected to contain a protein like Ki67 that is detected exclusively in the nuclei of proliferating whole cells. Because this option presents a situation that is unlikely to be true, it is the correct answer to the question.

The two age groups that participated in Study 1 are most likely to perform differently in which task developed by Piaget?

The 5-to-6-year-olds are in the preoperational period and the 8-to-9-year-olds are in the concrete operational period. In the preoperational period, children have not typically mastered concepts of conservation, but these concepts are mastered through concrete operational period. So, the two groups are most likely to differ on a conservation task. According to Piaget, both groups will have mastered A (object permanence) and B, and therefore do not differ on these tasks. D requires hypothetical reasoning, which neither group has mastered yet. C. Deciding whether a given quantity of a liquid changes if it is poured from a narrow container to a wide container

Which structure represents a phosphorylated subunit of glycogen as described in the passage?

The answer is D because the phosphate residues are added to the C2 and C3 positions of the sugar.

Based on the information in the passage, PDK1 catalyzes the addition of phosphate to what functional group?

The answer to this question is A (hydroxyl) because reactions involving either Ser or Thr would involve the hydroxyl group in the side chain of these amino acids.

In oxidative phosphorylation, cytochrome c acts as:

The answer to this question is A because cytochrome c is a heme protein that only cycles between a ferrous and ferric state during oxidative phosphorylation. Therefore, only single electron transfers are possible. It is a Knowledge of Scientific Concepts and Principles question because you must recall the role of cytochrome c in oxidative phosphorylation. A. a 1-electron carrier.

Which measurement unit CANNOT be used to express power?

The answer to this question is A because the measurement unit of power is watt, defined as J/s = ft•lb/s = kg•m2/s3.

What percentage of standard atmospheric pressure is the pulse pressure of a healthy adult?

The answer to this question is B because the pulse pressure in a healthy adult is (120 − 75) mmHg = 45 mmHg, and so the percentage is 45 mmHg/760 mmHg = 6%

What causes duplex DNA with a certain (A + T):(G + C) ratio to melt at a higher temperature than comparable length duplex DNA with a greater (A + T):(G + C) ratio?

The answer to this question is C. G-C base pairs form stronger π-stacking interactions than A-T base pairs, thereby creating the most thermal stability. This disparity has often been used to explain the increased melting temperature of DNA rich in GC content. C. Increased π- stacking strength

A patient with Korsakoff syndrome is very guarded toward the therapist and is reluctant to answer the therapist's questions. At subsequent appointments, the patient indicates not recognizing the therapist but is less guarded and more willing to answer questions. This suggests that the patient:

patient does not remember the therapist explicitly, but the change in his behavior indicates that he has an implicit memory of therapist, which likely results in him becoming less guarded.

What is the best experimental method to analyze the effect of tdh2 gene deletion on the rate of histone acetylation? Comparing histone acetylation in wild-type and Δtdh2 cells by:

posttranslational modification of proteins such as histone acetylation is analyzed by Western blotting.

What is the resistivity of the best-performing PANI described in the passage?

resistivity is the inverse of the conductivity, which is 1/5.0 × 10-3 (Ω∙cm)-1 = 200 Ω∙cm

Protein secondary structure is characterized by the pattern of hydrogen bonds between:

secondary structure is represented by repeated patterns of hydrogen bonds between the backbone amide protons and carbonyl oxygen atoms

Which major developmental milestone best explains the difference between the results seen for the 6- versus 9-month olds in Study 1?

stranger anxiety is the fear of strangers (or other-race faces in the case of this passage) that infants commonly begin to express around 8 months of age.

Which of the following best describes the bonds between Cu2+ and the nitrogen atoms of the ammonia molecules in [Cu(NH3)4]2+?

the Lewis acid-base interaction between a metal cation and an electron pair donor is known as a coordinate covalent bond.

The complement to the WT sequence does NOT contain which nucleoside?

the WT sequence does not contain deoxyguanosine. The complementary sequence will therefore not include deoxycytidine

Which reaction leads to the formation of DHB whose structure is shown in the passage?

the carboxylation of hydroquinone leads to the formation of 2,5-dihydroxybenzoic acid according to the reaction C6H4(OH)2 + CO2 → C6H3(CO2H)(OH)2.

Which of the following properties of a 2.3 MHz ultrasound wave remains unchanged as it passes into human tissues?

the frequency of a wave is not affected by the medium through which it propagates.

What is the magnitude of the electric field used in the MALDI-MS imaging device described in the passage?

the intensity of a uniform electric field E is related to the voltage V and distance d over which it is applied as E = V/d = 4.5 kV/(0.5 m) = 9 kV/m.

Based on the passage, CatB or CatL or both would be expected to have which of the following effects, if any, on EGP?

the passage states that CatB and CatL are proteases. Proteases function to digest proteins into smaller fragments.

Researchers collected the following data for an enzyme-catalyzed reaction. What is the approximate numerical value of the slope of the Lineweaver-Burk plot of these kinetic data?

the slope of a Lineweaver-Burk plot is equivalent to KM/Vmax. From the data it is apparent that the Vmax is 200 μM/min, and the KM is therefore the substrate concentration at a V0 of 100 μM/min, or 10 μM. The numerical value of the slope is then 10/200 or 0.05.

Henry's Law

the solubility of a gas in a liquid is directly proportional to the partial pressure of that gas on the surface of the liquid

The coefficient of viscosity of blood is directly proportional to the frequency of interactions of erythrocytes with the vessel walls. As blood enters small vessels, many erythrocytes tend to flow in the center of the vessel. Given this information, which graphic best describes the effect of vessel diameter on the coefficient of viscosity?

this graphic shows a reduced viscosity force in smaller vessel diameters which increases in larger diameter vessels.

base rate fallacy

using prototypical or stereotypical factors while ignoring actual numerical information

Allosteric inhibition of an enzyme involves which of the following events?

allosteric inhibition involves binding of an inhibitor to a site other than the substrate binding site.

The data in the passage suggest that the substrate binding domain in PfSET10 is:

According to the data in Figure 2, a PfSET10 fragment that contains the PHD domain and the SET domain binds unmodified histone H3. A fragment lacking the PHD domain but containing the SET domain does not bind unmodified histone H3. Taken together, these data best support the conclusion that the PHD domain is the substrate binding domain.

Asn

Asparagine (N) Polar with neutral R group but uneven charge

Neighborhood-level socioeconomic inequalities are most likely to affect physician-patient interactions through which phenomenon?

Neighborhoods that are segregated by socioeconomic status create increased social distance to the extent that people from different backgrounds have trouble relating to each other's experiences. The correct answer succinctly explains this by referring to closed networks and social boundaries that exist across neighborhoods b. Physical boundaries create social boundaries and closed networks which develop their own cultures.

In Phase 1, the researchers instructed participants to perform which task?

The answer to this question is A because repeating the digits presented to a specific ear is an example of shadowing.

The most likely result of various alanine to proline mutations in subunit A is

The answer to this question is C because proline disrupts α helices, and subunit A is primarily α-helical, which is a form of secondary structure. It is a Scientific Reasoning and Problem Solving question because it involves applying a scientific model of helix formation to make a prediction about the stability of secondary structure in response to amino acid changes. C. loss of secondary structure.

The concentration of enzyme for each experiment was 5.0 μM. What is kcat for the reaction at pH 4.5 with NO chloride added when Compound 3 is the substrate?

The fact that the rate of product formation did not vary over time for the first 5 minutes implies that the enzyme was saturated with substrate. Under these conditions, kcat = Vmax/[E] = (125 nM/s)/5.0 μM = 2.5 × 10-2 s-1.

What is the maximum energy of the UV photons generated by this plasma pencil? (Note: Speed of light is c = 3.0 × 108 m/s; Planck's constant is h = 6.63 × 10-34 J•s.)

energy of a photon is given by E = hf. The passage gives the wavelength of the UV radiation. Inserting this into the formula for photon energy yields E = hc/λ = 6.6 × 10-34 J × (3 × 108 m/s)/(200 × 10-9 m) ≅ 1 × 10-18 J

The researchers determined that two participants in the S group were taking medication that may have exacerbated their negative symptoms. Which classification of medication were these participants most likely taking?

neuroleptics are the first antipsychotic drugs used to treat schizophrenia and though they are effective in treating positive symptoms, their side effects include cognitive dulling, which can exacerbate negative symptoms.

Uracil is produced from cytosine as a result of a conversion of:

the deamination of cytosine, the amine group of cytosine is replaced with a carbonyl group, resulting in the structure of uracil.

When the stopcock is opened, how far up the tube will the water rise?

the water will rise to a height such that the weight (mass multiplied by gravitational acceleration) of the water column equals the atmospheric pressure multiplied by the tube cross-sectional area A. Because mass is density times volume, it follows that 103 kg/m3 × h × A × 10 m/s2 = 105 N/m2 × A, where h is the height sought. Solving for h yields h = 105 N/m2/(104 N/m3) = 10 m.

What is the pH of a .001 M NaOH solution?

A .001 M NaOH solution has a pOH (-log [OH-]) of 3. The pH of the solution is therefore 14 - 3 = 11. Thus, D is the best answer.

Suppose a certain far-sighted person can see objects clearly no closer than 300 cm away. What is the minimum distance from a plane mirror such a person must be to see his reflection clearly?

A plane mirror produces an image behind its plane at a distance equal to the object distance in front of the plane. If the sum between the object and the image distances must be at least 300 cm, then the mirror must be at half this distance, meaning 150 cm.

Are the assumptions of SCT regarding the role of participants' beliefs (paragraph 3) consistent with a traditional behaviorist approach?

A rigid behaviorist approach rejects SCT's assumption that cognitions can act as motivators for behavior. Instead, it holds that only actual outcomes of a behavior determine whether that behavior will be repeated.

cross-sectional study

A study in which a representative cross section of the population is tested or surveyed at one specific time.

Epimers

A subtype of diastereomers that differ in absolute configuration at exactly one chiral carbon

Normally, a hypothalamic factor stimulates the release of adrenocorticotropic hormone (ACTH) from the pituitary gland. In a patient with Addison's disease, the secretion of the hypothalamic factor will:

ACTH stimulates the adrenal cortex to secrete glucocorticoids. According to the passage, Addison's disease occurs when cells of the adrenal cortex are destroyed, rendering them unable to secrete glucocorticoids. As part of a hormone cascade pathway, ACTH secretion is expected to be higher than normal in a patient with Addison's disease to attempt to stimulate the adrenal cortex.

Suppose that an extract from a muscle cell contains only the following: all the enzymes of the glycolytic pathway, including the enzyme that converts pyruvate to lactate; phosphate and other salts; NAD+ and ADP. When the extract is incubated anaerobically and glucose is introduced, neither pyruvate nor lactate is produced. What must be added in order for pyruvate to be made?

ATP must be added. The overall glycolytic pathway yields two ATP molecules, but in order to gain these two ATP molecules by the end of the process, some ATP must be consumed at the beginning. The first reaction, the phosphorylation of glucose, for instance, requires ATP. If this is lacking in the reaction mixture, glycolysis will not be initiated. Once initiated with a small amount of ATP, however, more ATP is produced and the reaction will become self-sufficient. Therefore, the correct answer is option B. Oxygen and acetyl-coenzyme A are needed only for oxidative metabolism, so options A and D are incorrect. NAD is supplied and the process will produce NADH, but NADH is not needed for the reaction sequence to proceed, so option C is incorrect.

Which of the following graphs best illustrates how the force between fragments from the fission of a uranium nucleus varies as the fragments move away from each other?

According to Coulomb's law, the repulsive force versus fragment separation (r) has a 1/r2 dependence. Graph D shows this behavior.

The finches observed by Darwin on the Galapagos Islands are an example of adaptive radiation. In order to set up conditions that would produce adaptive radiation, it would be necessary to place members of:

Adaptive radiation involves the divergence of one species into multiple species over time, which can occur when subgroups of the original species are separated or isolated in different environments so that these subgroups evolve independently of one another. C. once species in several different environments

Ala

Alanine (A) hydrophobic, nonpolar R group

Assuming Hypothesis B to be correct, which of the following endocrine disorders would cause hypertension that could NOT be rectified by physiologically normal kidneys?

Aldosterone is a hormone released by the adrenal glands. Physiologically normal kidneys respond to aldosterone by increasing the reabsorption of both sodium and water. This leads to an increase in blood volume and therefore blood pressure.

Would an increase in the level of plasma aldosterone be expected to follow ingestion of excessive quantities of NaCl?

Aldosterone, which is produced by the adrenal cortex, causes Na+ reabsorption by kidney tubules. Such a mechanism decreases Na+ levels in the urine.Because ingestion of excessive NaCl would trigger Na+ secretion into the urine, plasma-aldosterone levels would not increase. Rather, the body would rely on those homeostatic mechanisms that excreted the excess Na+. Thus an increase in plasma aldosterone would not be expected to follow ingestion of large quantities of NaCl.

A receiver is in a jet flying alongside another jet that is emitting 2.0 x 106 Hz radio waves. If the jets fly at 268 m/s, what is the change in frequency detected at the receiver?

Because there is no relative motion between the jets, there is no frequency shift. 0

Which of the following functional groups is found in benzoin, C6H5CH(OH)C(O)C6H5?

Benzoin contains a ketone carbonyl group and an alcohol hydroxyl group. It has no carboxylic acid, ether, or aldehyde. Thus, answer choice D is the best answer.

Glucose is labeled with 14C and followed as it is broken down to produce CO2, H2O, and ATP in a mammalian liver cell. In theory, during this process the label will be detectable:

Breakdown of glucose proceeds first by glycolysis, then by oxidation in the citric acid (Krebs or tricarboxylic acid) cycle. The enzymes for the former process are located in the cytoplasm and those for the latter are in the matrix of mitochondria. The 14C label, therefore, would first appear in the cytoplasm, then in the mitochondria

Based on the findings in Study 2, which strategy for decision-making groups would be most likely to prevent groupthink?

C. Encourage a group norm of critical evaluation and dissent in decision-making. Those in the social identity condition exhibited more groupthink symptoms than those in the interpersonal condition. They make more rationalizations, introduce fewer facts, and discuss risks less frequently. Pressures to reach consensus and stifle dissent are key to groupthink, and thus a group norm that encouraged dissent and critical evaluation would address the central problem of groupthink

Which of the following statements best applies to the inactive X chromosome in mammalian females?

C. It is one of the last chromosomes to replicate.

Which cation is most likely to be found in place of Fe(II) in the square planar binding domain of hemoglobin?

Co2+ is closely related to Fe2+ as a transition metal and can support a square planar coordination environment.

Retinol uptake was measured in untransfected and STRA6-transfected cells in the presence of retinol bound to either RBP or bovine serum albumin (BSA). Uptake of retinol bound to RBP was also measured in cells that were cotransfected with a small inhibitor RNA (siRNA) targeting STRA6. Which graphic shows the expected result of this experiment?

3H-retinol uptake should be highest in STRA6-transfected cells when retinol is bound to RBP. The passage states that STRA6-mediated retinol uptake is specific to RBP, so retinol uptake when retinol is bound to BSA should be at control levels. siRNA targeted against STRA6 will reduce STRA6 levels so uptake of retinol bound to RBP should be lower than in the absence of the siRNA.

Based on Reaction A, if all the energy produced from glycolysis were used to remove Na+ from a cell, how many molecules of Na+ would be removed per molecule of glucose? 3 Na+(inside) + 2 K+(outside) + ATP4− + H2O → 3 Na+(outside) + 2 K+(inside) + ADP3− + Pi 2− + H+

6 Since glycolysis produces two ATP molecules per glucose molecule, and 3 Na+ molecules are extruded for each ATP molecule utilized by the sodium pump, a total of 6 molecules of Na+ are extruded per molecule of glucose.

The negative control used in Experiment 1 was most likely:

A negative control is the one that is performed in the conditions that are not expected to give positive results. Thus, unconditioned media will need to be used. As antibodies are highly specific, if 53BP1 and γH2AX are detected in the negative control, this means that AEC are present.

In helping to explain the results of the study, which other concept would be most similar to a homophilous social network?

A reference group is defined as "any group that individuals use as a standard for evaluating themselves and their own behavior," and is similar to the mechanism behind the effect of the "homophilous" group in the study.

Compared to control mice, the genetically modified mice were more likely to exhibit which expression levels of Aβ and NFT?

A. Higher expression levels of both Aβ and NFT in the brain regions examined in the study AD is associated with a build-up of Aβ and NFT levels in certain brain regions. Therefore, the researchers would most likely overexpress both Aβ and NFT in the genetically modified mice to create a good mouse model for AD.

Based on the passage, ACC2 is most likely compartmentalized to:

ACC2 is involved in fatty acid oxidation, which takes place in the mitochondria.

Which one of the following cations or anions would be useful in differentiating between a solution containing FeCl3 and one containing FeF3?

According to the data in Table 1, CaF2 will form a white precipitate, but CaCl2 is water-soluble. Using Ca2+, one could differentiate between a solution of FeCl3 and FeF3.

The chemical valinomycin inserts into membranes and causes the movement of K+ into the mitochondria. Based on Figure 1, if mitochondria are treated with valinomycin, the rate of ATP synthesis in the mitochondria will most likely:

An influx of another positively charged ion into the compartment would disrupt the electrochemical gradient responsible for the necessary flow of protons. Answer B, therefore, seems more plausible than any other choice. decrease, because movement of K+ into mitochondrial compartments will disrupt proton movement into the intermembrane space

What is the structure of the product formed when Compound 3 is the substrate of laccase-catalyzed reaction?

Based on Reaction 1 and the fact that the product of oxidation absorbs strongly in the visible region, the most likely structure of the product is the highly conjugated quinone that is depicted in structure D

At STP, the volume of N2(g) produced by the complete decomposition of 1 mole of nitroglycerin would be closest to which of the following?

Based on the balanced equation provided, 4 moles of nitroglycerin produce 6 moles of N2(g). Therefore, 1 mole of nitroglycerin will produce 1.5 moles of N2(g). At STP 1.5 moles of N2(g) will occupy 33.6 L since the molar volume of an ideal gas at STP is 22.4 L/mol.

What is the approximate concentration of reaction product in a solution that has an absorbance of 0.7 at pH 6.0?

Based on the equation A = εbc given in the passage, the concentration c is the absorbance A divided by the absorptivity ε in a 1 cm path length cell. 0.7 divided by approximately 1400 gives 500 μM.

The left cerebral hemisphere in humans is most often linked with which cognitive function?

C. Vocabulary skills Vocabulary skills tend to be lateralized to the left hemisphere, whereas visuospatial skills, music perception, and emotion processing tend to be lateralized to the right hemisphere

The electric field inside each of the conductors that forms the capacitor in the defibrillator is zero. Which of the following reasons best explains why this is true?

Conductors contain both atom-bound electrons and free electrons. Free electrons arrange themselves on the surface of conductors, and their collective electric field produced inside the conductor cancels any external electric field. The resulting electric field inside the conductor is zero.

In the circuit shown above, the current in the 2-ohm resistance is 2 A. What is the current in the 3-ohm resistance?

Currents in parallel resistors are inversely proportional to their individual resistances because they have the same voltage drop across them (Ohm's law). Since the 2 ohm resistor in the diagram has 2 A, the parallel 4 ohm resistor has 1 A through it. These currents add to 3 A as they combine to pass through the 3 ohm resistor (Kirchhoff's junction rule).

From the data in Figure 1, one can conclude that the sensitivity of aortic smooth muscle to acetylcholine is:

Figure 1 allows the conclusion that the sensitivity of aortic smooth muscle to ACH is increased at least 10 times in the presence of endothelium. Relaxation occurs in the ring with endothelium at 10-7 M ACH but in the other ring does not occur even with 10-6 M ACH; these concentrations differ by a factor of 10.

Based on the reaction scheme in Figure 1, what is the mechanism of substrate binding to RT?

Figure 1 shows that the TP substrate binds first without any catalysis occurring and then the dNTP substrate binds. This is an ordered mechanism.

Which of the following chemical species is NOT isoelectronic with a neon atom?

If two atoms are isoelectronic, they have the same number of electrons. Neon has ten electrons. F- has 9 + 1 = 10 electrons and so is isoelectronic with Ne. Mg2+ and Na+ have 12 - 2 = 10 and 11 - 1 = 10 electrons respectively. Only He, with two electrons, is not isoelectronic with neon.

Researchers conducted a replication of the pilot training study with bus drivers. How accurately will the measure of a person's ability to redirect attention predict accident rates among drivers? The error rates in:

In fact, it was the primary finding of a follow-up study that used bus drivers as participants. Thus, the selective listening task provides a robust measure of a person's susceptibility to having a change in the orientation of attention cause a transient disruption of attention on any task that demands the ability to redirect attention.

What control experiment was necessary to ensure that the apparent subcellular locations of the cat6xbs and lacY6xbs transcripts were NOT skewed by the location preference of the bound MS2-GFP?

Lack of the 6xbs sequence would leave the MS2-GFP protein without a binding partner. It would then be located in either the cytoplasm or the membrane (in this case, the cytoplasm) based on the properties of the MS2-GFP protein itself rather than being restricted to the location of the cat6xbs and lacY6xbs transcripts

Why did the liposomes fluoresce during size-exclusion chromatography?

Liposomes can be difficult to detect since they do not absorb visible light and many molecules absorb UV light. The experimental design allowed fluorescent dye to be trapped inside during liposome formation, which allowed their detection by fluorescence spectroscopy.

Lys

Lysine (K) Basic -> + charge at pH 7 R group PI = 10.5

If a cell's membrane potential changes from -60mV to -70mV after treatment with an adrenergic drug, the NE receptor is most likely linked to:

Of the options listed, D, a K+ channel, is most likely to cause such a change in membrane potential. A change in membrane potential from -60mV to -70mV indicates that the cell membrane is most likely repolarizing to its resting potential of -70mV following an action potential. In a neuron, this is accomplished through the opening of K+ channels that release K+ from the cell to counterbalance the influx of Na+ that occurred during depolarization.

When a downward force is applied at a point 0.60 m to the left of a fulcrum, equilibrium is obtained by placing a mass of 10-7 kg at a point 0.40 m to the right of the fulcrum. What is the magnitude of the downward force?

One force's torque will attempt to twist the "see-saw" supported by the fulcrum clockwise, while the other force provides a counterclockwise torque. The torque of a force F applied at a distance L from the fulcrum is given by FLsinθ, with θ is the angle between the beam and the force. At equilibrium the left and right forces point vertically downward with the beam horizontal, hence θ = 90° and sin90° = 1. At equilibrium the torque magnitudes are equal so: Fleft Lleft = Fright Lright. The force at Lright = 0.40 m is the weight of the given mass, mg = (10-7 kg)(9.8 m/s2) = 9.8 × 10-7 N. The unknown force is at Lleft = 0.60 m, so Fleft (0.60 m) = (9.8 × 10-7 N)(0.40 m) which may be solved to yield Fleft = 6.5 × 10-7 m. Thus B is the best answer.

Which of the following correctly pairs a cellular process with the location in which that process occurs in a prokaryotic cell?

Prokaryotic cells do not contain nuclei, membrane-bound organelles in which nuclear genes are transcribed in eukaryotic cells. Consequently, transcription occurs in the cytoplasm of bacterial cells.

Which statement is NOT compatible with the hypothesis that the self-serving bias can account for participants' explanations of their body weights?

Self-serving bias suggests that when explaining their own behavior, individuals attribute positive behaviors to internal, stable sources, but attribute negative behaviors to external sources. A non-obese individual would attribute his or her healthy weight to an internal, stable source, such as strong willpower. However, a non-obese individual would not attribute his or her healthy weight to an external source, such as not having any fast food restaurants near home. Therefore, D is incompatible with the self-serving bias.

Which of the following atoms has the largest atomic radius?

Sodium, aluminum, sulfur, and chlorine are all in the third row of the periodic table. Atomic radius tends to decrease from left to right across a given row of the periodic table.

Which technique CANNOT be used to analyze gene expression?

Southern blotting is a technique used to detect a particular sequence in a sample of DNA

The radioactive decay described in the passage results in the formation of which two atoms?

The answer to this question is A because the type of radioactive decay is b-1 so a neutron is converted into a proton, which results in carbon becoming nitrogen and phosphorus becoming sulfur

Which feature of the kinetics of L-alanine transport would provide evidence that DOM is an important source of nutrients under low-food conditions?

The answer to this question is A because under low-food conditions, the capacity of transport is irrelevant as no transporters are saturated with L-alanine. However, the transporter's affinity for L-alanine would be important as only high affinity transporters would be able to facilitate L-alanine uptake at low concentrations. It is a Scientific Reasoning and Problem Solving question because you are asked to apply knowledge of transport kinetics to make a prediction about what features of transport would be important under low L-alanine conditions. A: High-affinity transport (Kt) of L-alanine

Gamma decay occurs when a nucleus emits:

The answer to this question is A because, by definition, gamma decay means the emission of photons by the nucleus

Based on the relative energy of the absorbed electromagnetic radiation, which absorber, a peptide bond or an aromatic side chain, exhibits an electronic excited state that is closer in energy to the ground state?

The answer to this question is B because aromatic side chains absorb in the near UV region of the electromagnetic spectrum, which has longer wavelengths, and hence lower energy, than peptide bonds. Because the energy of the photon matches the energy gap between the ground and the excited state, this implies that the aromatic side chain has more closely spaced energy levels.

A fatty acid is comprised of a long hydrocarbon tail and a head consisting of:

The answer to this question is B because fatty acids contain a carboxylic acid head group and a hydrocarbon tail. It is a Knowledge of Scientific Concepts and Principles question because it requires you to recognize the components of a fatty acid.

A separate group of scientists repeated the amino acid transport experiments described in the passage. All protocols were performed as outlined in the initial study with one major exception: D-alanine was used as the primary amino acid substrate. What effect will this change have on the synthesis of new proteins in the animals studied? Protein synthesis will:

The answer to this question is D because only L isomers are used to form proteins during ribosomal protein synthesis. It is a Reasoning about the Design and Execution of Research question because it requires you to reason about the features of a research study and determine the implications of changing a variable in the study.

What is the energy of the photons emitted by the LED at a frequency of 610 THz? (Note: h = 6.6 × 10-34 J·s)

The answer to this question is D because the energy of a photon of frequency 610 THz is equal to 6.6 x 10-34 J•s x 610 x 1012 Hz = 4 x 10-19 J

During glycolysis, pyruvate CH3C(=O)CO2- is reduced to lactate CH3CH(OH)CO2- by nicotinamide adenine dinucleotide (NADH). What is the balanced reaction for this conversion?

The answer to this question is D, which can only be arrived at by using half-reactions. The balanced half-reaction for the reduction of pyruvate is CH3C(=O)CO2- + 2H+ + 2e- → CH3CH(OH)CO2-. Balance was attained by first balancing hydrogen with H+ (adding two H+ to the reactant side) and then the charge with e- (adding two to the reactant side, also). The oxidation half-reaction is: NADH → NAD+ + H+ + 2e-, which is balanced in a similar manner. At this point the half-reactions are added in such a way that the e- involved will cancel. Since two e- are involved in both the oxidation and the reduction, the two half-reactions can be added directly to provide the balanced reaction. This question requires Scientific Reasoning and Problem Solving to arrive at the answer.

Considering the stage that the participants are in according to Kohlberg's theory of moral development, what changes to the study design are most likely to result in decreased aggression?

The correct answer is B. The participants are in Kohlberg's preconventional stage of moral development (since they are pre-adolescents). In this stage, morality is determined by what is punished or rewarded. A and D are not related to Kohlberg's theory. C describes a situation that is more likely to be relevant in later stages of moral development.

Based on the information in the passage, which description of an enzyme-substrate covalent intermediate is most likely correct? The substrate is covalently attached to:

The covalent intermediate will occur through the nucleophilic substitution by the side chain carboxyl of Asp14 at the electrophilic phosphorus atom in the substrate, displacing a leaving group.

Which of the following graphs best shows the number of moles of S4O62-(aq) in Tube 6 as time passes?

The data in Table 1 show that the reaction is complete in 19 sec. The amount of S4O62- would increase for 19 sec and then level off

Ba2+(aq) is an ion that is very toxic to mammals when taken internally. Which of the following compounds, mixed in water, would be the safest if accidentally swallowed?

The lower the value of Ksp is, the lower the concentrations of the cation and anion in an aqueous solution and the lower the solubility of the compound in water. If mixed with water and accidentally swallowed, the Ba salt with the lowest value of Ksp would be the safest.

Which of the following properties is most useful in explaining the trend in the reactivities in Experiment 1?

The metals that reacted in Experiment 1 underwent oxidation. The energy required to remove an electron from an atom is the ionization potential. The reactivity of a metal depends on its ionization potential. Among the metals listed in Table 1, potassium has the smallest ionization potential and magnesium has the largest ionization potential. Therefore, potassium reacted the most vigorously and there was no obvious reaction with magnesium.

Increasing the frequency of each photon that is directed at the cathode will:

The only effect the photon frequency has on the ejected electron is on its kinetic energy. Photon energy equals cathode work function plus electron kinetic energy. The number of electrons ejected (the current) depends on the number of incident photons.

The theory of force generation proposed in the passage is best supported by which of the following observations about Amoeba locomotion?

The passage proposes that force is generated as a microfilament elongates and pushes against a structure such as the plasma membrane. This is representative of how an amoeba moves. Cytochalasins are drugs that inhibit the growth of microfilaments. Therefore, if amoeboid movement stops upon exposure to cytochalasins, microfilaments and their ability to elongate are both implicated as being necessary to generate the force for movement in an amoeba

How many dietary calories does a 1-g sample of Olestra contribute to a human consumer?

The passage states that Olestra is not metabolized because digestive enzymes are blocked from cleavage sites. If a molecule is not metabolized, it can provide no energy and thus no calories. Choice A, 0 Cal, is the correct answer.

A patient who is experiencing severe marital problems reports having no memory of any life events surrounding the marriage and the spouse. The patient's memory for other life events is intact. This patient is most likely to be diagnosed with:

The patient is selectively forgetting distracting elements of his/her life, which indicates a dissociative disorder. D. a dissociative disorder.

When viewing an X ray of the bones of a leg, a doctor can tell if the patient is a growing child, because the X ray shows:

The question asks the examinee to identify the characteristic that differentiates growing, developing long bones from adult bones. Long bones grow via endochondral ossification, which requires cartilaginous growth plates at the ends of long bones, that thicken as cartilage and later become ossified

An object is placed upright on the axis of a thin convex lens at a distance of four focal lengths (4f) from the center of the lens. An inverted image appears at a distance of 4/3f on the other side of the lens. What is the ratio of the height of the image to the height of the object?

The ratio of object to image distance equals the ratio of object to image height. The ratio of image to object height is found by rearranging the ratios to give (4f/3)/4f = 1/3. The image is demagnified by a factor of 3

How many sodium ions are in the initial 50.00-mL solution of Na2CO3?

The solution contains 7.15 g Na2CO3×10H2O. Dividing by the molar mass will give the number of moles: (7.15 g)/(286.14 g/mol) = 0.0250 mol. Because each mole of Na2CO3×10H2O contains 2 mol Na+, there is 0.0500 mol Na+. Using Avogadro's number, the number of sodium ions is (0.0500 mol)(6.02 × 1023 ions/mol) = 3.01 × 1022 ions. Thus, B is the best answer.

Based on the passage, the magnitude of ΔH° (in kJ) for the decomposition of 2 moles of nitroglycerin at 25°C is closest to which of the following?

The value of ΔH° can be calculated using the data provided in Table 1 and applying Hess's Law. Two moles of nitroglycerin produce 6 moles of CO2(g) and 5 moles of H2O(g). The value of ΔH° for this amount of nitroglycerin combusted is 2(364.0) - 6(393.5) - 5(241.8) = -2842 kJ/mol. I

The above plot shows how the volume of a 1.0-g sample of Compound 1 in the gaseous state varies with temperature at constant pressure. Where on the graphic would a similar plot of a 1.0-g sample of Compound 2 appear?

The volume of a gas depends only on the number of moles of the gas present and not on the identity of the gas. The ideal gas law in the form V = (nR/P) T shows that, at constant pressure, the volume will increase with temperature. Thus, a plot of V vs. T will always have positive slope. The value of the slope depends on the number of moles of gas. Because Compound 2 has a higher molecular weight than does Compound 1, a one-gram sample will contain fewer moles. Thus, a plot of the volume of Compound 2 as a function of temperature will have a smaller slope than that of Compound 1 and will lie below it on a graph, intersecting only at absolute zero. This situation is described by answer choice A.

The production of a variety of opsins functions to:

The wavelength of light absorbed by a molecule depends on its structure, and so the production of a variety of structurally related opsins functions to enable the detection of different colors

This is a Single Choice Question; skip ahead to question content Incorrect Correct answerYour answer A B C x D Time Spent: 0 min 43 secs Content & Skills: SIRS1 CC1B FC1 BIO Passage 5 (Questions 22-27) Muscle cells are equipped with inherent capacity to perceive and respond to biomechanical stimuli, such as pressure, and convert them into molecular events that regulate multiple anabolic and catabolic processes. MYOD1 is a member of the family of helix-loop-helix myogenic factors that plays a major role in regulating muscle differentiation by binding to the consensus sequences of the regulatory region of muscle-specific genes such as cyclin-dependent kinase inhibitor 1A (Cdkn1a). MYOD1-mediated induction of Cdkn1a results in the irreversible exit of undifferentiated cells from the cell cycle and their commitment to the differentiation program. MYOD1 contains a C‑terminal nuclear localization sequence (NDAFEITKRC) that promotes its translocation to the nucleus. Researchers simulated an in vitro biomechanical stimulus environment by using a cyclic tension strain cell culture chamber and investigated the effect of cyclic tensile strain (CTS) on myogenic differentiation by measuring the levels of MYOD1 in the presence and absence of TNF-α. TNF-α is a proinflammatory cytokine that, upon binding to its receptor TNFR1, induces the expression of nitric oxide synthase (NOS) and NO production. Experiment 1 Cultured myoblast cells were subjected to CTS and the MYOD1 mRNA levels were measured in the presence or absence of recombinant human (rh) TNF-α (5 ng/mL) and NG-nitro-arginine methyl ester (L-NAME), a competitive inhibitor of NOS activity. The results are shown in Figure 1. Figure 1Effects of TNF-α, L-NAME and CTS on MYOD1 mRNA levels The nuclear localization sequence of MYOD1 contains how many potential phosphorylation sites?

There are three amino acids that are phosphorylated in eukaryotes: serine (S), threonine (T), and tyrosine (Y). The nuclear localization sequence of MYOD1 (NDAFEITKRC) contains one threonine (T) that can potentially be phosphorylated.​

It was hypothesized that the decrease in blood flow to the skin resulted from a change in the activity of the sympathetic nerves to the skin. Which of the following observations would support this hypothesis?

This hypothesis would be supported if researchers observed a change in the norepinephrine content of blood draining from the skin.

What is the mechanical work done by the cantilever when the extension increases from 10 nm to 15 nm?

This question requires you to apply the graphical method of calculating the work done by a variable force, namely the elastic force, W = ksx2/2. Mechanical work by an elastic force is W = ksx2/2 = Fx/2. Graphically, it is the area under the force-extension graph between the corresponding data points. From Figure 2, the correct response is {[(100 - 0) × 10-12]N ×[(15 - 10) × 10-9]m}/2 = 2.50 × 10-19 J.

The bglF transcript is known to have a short half-life within the cytosol. What mechanism is most likely responsible for transport of this transcript to the cytoplasmic membrane once it is synthesized?

This would allow for more rapid transport than diffusion. Active transport across the cytosckeleton

The amino acid precursor of serotonin is best described as having which type of R group?

Tryptophan, the amino acid precursor of serotonin, contains an aromatic R group.

Under anaerobic conditions, how many net molecules of ATP are produced by the consumption of 5 moles of glucose?

Under anaerobic conditions, 2 moles of ATP are produced from each mole of glucose. Thus, 10 moles of ATP would be generated from 5 moles of glucose. Since there are 6 × 1023 molecules per mole, 10 moles of ATP is equal to 6 × 1024 molecules.

Val

Valine (V) hydrophobic, nonpolar R group

Which of the following substances is most likely to be more soluble in 1.0 M HCl than in 1.0 M NaOH?

When an ionic substance dissolves, it dissociates into its constituent ions. The solubility of any substance whose anion is basic will be increased in solutions of low pH, as the basic anion reacts with the H+ in solution and is drawn out of the equilibrium. As the basic anion is consumed, the dissolution reaction is driven to the right by Le Chatelier's principle. Of the substances given as choices, Pb(OH)2 has the most basic anion, OH-. Thus it is most likely to show increased solubility in an HCl solution.

According to the psychodynamic theory, conflict between the superego and the id leads to unconscious conflict. The ego attempts to reduce this conflict through the use of defense mechanisms. Based on this description, the unconscious conflict functions as

a drive is an internal state that the individual acts to reduce, which is consistent with the definition of unconscious conflict in the stem.

What was the experimentally determined value of KNAG from the study described in the passage?

according to Equation 1, the slope of the graph in Figure 2 is the value of KNAG. The slope is Δy/Δx = (2 - 1)/(0.020 - 0.00) = 1/0.020 = 50.

A homodimeric protein was found to migrate through SDS polyacrylamide gel electrophoresis (SDS-PAGE) with a mobility that matched that of a 45-kDa standard. What change in the experiment would increase the chances of observing the mobility expected for the 22.5-kDa monomer?

adding a reducing agent would eliminate any disulfide bridges and allow the monomers to run separately-thus leading to a migration expected for the 22.5-kDa protein. B. Adding a reducing agent

Which statement best characterizes how a proponent of the SG hypothesis will describe the direction of the relationship between stress and depression?

an independent stressor is assumed to occur independently of any influence from the person experiencing the stress. This is a unidirectional relationship (stressor leads to depressive symptoms). Dependent stressors, on the other hand, not only influence depression but could themselves be influenced by depression (i.e., a reciprocal relationship).

Which experiment can be used to investigate the transcriptional regulation of the Cdkn1a protein?

as the question is focused on the transcriptional regulation, it is logical to assess the mRNA levels as opposed to protein levels. RT-PCR is a molecular technique that measures mRNA levels of specific protein.

A constructionist understanding of gender asserts that categories of gender are:

constructionist understanding describes gender as dynamic, fluid, and subject to the processes of meaning-making and collective definition building.

The reminiscence bump is best described as an example of:

cultural transmission addresses how culture is learned. Culture is passed along from generation to generation through various childrearing practices, including when parents expose children to music. As illustrated by the reminiscence bump, parents' musical preferences affected their children's memory of music.

Lytic granules are generally released from CTLs when the T-cell receptors on these cells bind specifically to:

cytotoxic T lymphocytes target virus-infected cells by recognizing the viral antigen presented on the cell surface A. viral antigens presented on the surface of virus-infected cells.

A protein contains four disulfide bonds. In order to break these bonds researchers added a minimum of:

each mole of NADH can reduce a mole of disulfide bonds. Since the protein has four disulfide bonds, four moles of NADH are needed.

Which cells harvested from adult mice were most likely used as the highly proliferative benchmark in the experiment that generated the data shown in Figure 3?

epithelial cells that line the gastrointestinal tract are typically highly proliferative C. Gastrointestinal epithelial cells

IUPAC rules,

first identify the longest unbroken chain of carbon atoms. Next, number the carbon atoms in this chain starting from the end that gives C=C the lowest numbers. The double bond is identified by the position of the carbon atom from the lowest numbered end (2), and then the methyl group is assigned at the 3-position. The stereochemical designator for the double bond is Z because the highest priority groups (methyl at C2 and ethyl at C3) occur on the same side of the double bond. The name is therefore Z-3-methylpent-2-ene.

If the researchers were focused on an evolutionary perspective of human motivation, what aspect of the study results would be of most interest to them?

from an evolutionary standpoint, humans have developed a preference for high caloric foods because they are a good source of fuel in the form of fat. Fat can sustain bodily functions through periods of time when food is scarce (a common problem throughout evolution).

A gas that occupies 10 L at 1 atm and 25oC will occupy what volume at 500 atm and 25oC?

he ideal gas law makes the assumption that molecules have no volume. This assumption is adequate when the gas is at 1 atm, but when the pressure is increased to 500 atm the volume of the gas molecules is no longer negligible. somewhat more than .02 L because of the space occupied by the individual gas molecules

As described in the passage, a medical student's sense of self-efficacy and confidence regarding patient interactions is most likely to manifest via which phenomenon?

impression management in the only concept listed that addresses how individuals actively manifest their sense of self in social interactions.

What is the best explanation for the difference in the number of intrusion errors in Phase 1?

in all but a relatively small number of people, the brain areas that process linguistic information (both production and comprehension) are lateralized in the left hemisphere.

The ATP-dependent phosphorylation of a protein target is catalyzed by which class of enzyme?

kinases catalyze the transfer phosphate groups from ATP to target proteins and are classified as transferases.

gamma decay

nuclear decay that involves the release of gamma rays

Working with depressed patients, a therapist wishing to implement a treatment framework based on the SG hypothesis, will likely pay a great deal of attention to:

one feature of the stress generation hypothesis is the idea that recurrent depressive episodes are particularly problematic and are clearly at the core of the stress-depression connection. Also, the hypothesis does not assume people can rid their lives of stressful events (option C). Rather, the idea is to reduce the negative outcomes of those events.

What is the identity of the substance that has undergone net reduction after the Reaction 1 is complete?

oxygen is the final acquirer of electrons in the reaction and is therefore reduced.

Activation of which enzyme would support the metabolism of newborn infants during the first 12 hours?

passage states that hepatic glycogen supports the basic metabolism of newborn during the first 12 hours. Glycogen phosphorylase is the enzyme that catalyzes the rate-limiting step in glycogen breakdown (glycogenolysis).

Regular breathing and regular, slow brain waves are most consistent with which stage of sleep?

regular breathing and heart rate and slow brain waves are most consistent with deep sleep, NREM3

Roger's humanistic theory

says people are inherently good and we are self-motivated to improve so we can reach self-actualization

During the production of insulin, the translated polypeptide is cleaved into the mature form and secreted from the cell. The cleavage most likely takes place in which of the following locations?

secreted proteins such as insulin are cleaved into mature form within endomembrane system

Thomas Theorem

situations that are defined as real are real in their consequences

Based on the data presented in Table 1, which amino acid residue of prorenin most likely interacts with the residue at position 201 of PRR?

substitution of aspartate (D) at position 201 with asparagine (N) results in a lower PRR affinity (higher Kd) towards prorenin. Since both asparagine and aspartate have equivalent carbon lengths, the decrease in affinity is most likely due to elimination of an ion pair. Among the possible options, only arginine could form an ion pair with aspartate.

Which extraction procedure will completely separate an amide from the by-product of the reaction between an amine and excess carboxylic acid anhydride?

the by-product of the reaction will be an acidic carboxylic acid and the excess unreacted starting material will also be acidic. Extraction with aqueous base will hydrolyze and extract both of these into the aqueous layer, leaving the neutral amide in the ether layer.

Based on the passage, the data from the 2012 study was MOST likely:

the data from the self-assessment completed at the end of the communications course would be cross-sectional data.

The Michaelis-Menten kinetic data for an enzyme in the presence and absence of an inhibitor is shown. Which statement about the mechanism for inhibitor binding is supported by the data in the Table? The inhibitor binds:

the data in the table indicate that the inhibitor is a competitive inhibitor (apparent KM is increased while the apparent Vmax is unchanged). Competitive inhibitors bind in the enzyme active site to the exclusion of the substrate.

Which metabolic reaction is most likely affected by treatment of cells with C75? The reaction that converts:

the passage states that mtKAS is involved in the synthesis of lipoic acid and Figure 1 shows that mtKAS is inhibited by C75. Therefore, treatment of cells with C75 results in lower cellular level of lipoic acid, a cofactor for the enzyme pyruvate dehydrogenase which catalyzes the conversion of pyruvate to acetyl-CoA

Overexpression of which enzyme is likely to result in increased levels of HIF?

the passage states that succinate modulates the level of HIF by inhibiting HIF hydroxylase, an enzyme that induces HIF degradation. Thus, overexpression of succinyl-CoA synthetase, which results in the increased production of succinate, will enhance HIF levels

According to the passage and the data in Table I, what feature must distinguish laser type A from laser type C in order to be suitable for the cleavage of the bonds specified? Laser type A must:

the photon energy must be larger than the bond energy in order to break the bond. Because the photon energy is directly proportional to the radiation frequency, laser type A must have a higher frequency than laser type C.

Which amino acids are most likely present at the dimerization interface of STAT3 proteins?

the polar and charged amino acids most likely interact with water molecules in cytosol and would not be involved in protein-protein interactions. In contrast the side chains of hydrophobic amino acids are free and most likely participate in dimerization of STAT3. I

Anxious about a nagging illness, a patient feels ignored by a doctor who is struggling to catch up with patient examinations on a very busy day. The doctor misinterprets the discomfort and agitation of the patient as hostility. Which sociological paradigm can best explain this scenario?

the scenario concerns the inter-subjective negotiation of symbols or meanings, which is indicative of symbolic interactionist theory

To explain the demographic dynamics in the passage, a conflict theorist is most likely to consider the relationship among which factors?

the sociological paradigm of conflict theory broadly calls attention to competition among social groups, including generational conflict. Given the demographic dynamics raised in the passage, it is likely that competition over resources will emerge between the old and the young (for example, public support of health care versus education, each of which tends to benefit one age group more than the other). A conflict theorist would be interested in explaining how political power varies by generational status and thus affects the allocation of social resources.

The relationship between the steroid hormone, estrogen, and the peptide hormone, insulin, is being investigated. In order to quantify levels of each of these hormones, tissue samples were homogenized and then placed in a mixture of 2:1 hexane/water. What is the expected result from this extraction method?

the structural features of peptides and steroids imply that insulin is hydrophilic and estrogen is hydrophobic. The hydrophilic insulin peptide would be expected to be found in the aqueous phase, while the hydrophobic estrogen hormone would segregate into the hexane phase

To determine a protein's thermodynamic stability, chemical denaturation studies can be performed. Assuming that only the native and unfolded states can be observed under experimentally available conditions, what is the most likely shape of the curve for the dependence of the fraction of folded protein upon denaturant concentration?

the unfolding of proteins is a cooperative process. Cooperative processes are marked by sigmoidal curves

How much work did an 83-year-old female do while stretching the rubber band to the limit of her strength?

the work done is W = 0.5 × kx2 where x = 0.20 m. So W = 0.5 × 200 (N/m) × (0.2 m)2 = 4.0 J.

The stereochemical designators α and β distinguish between:

the α versus β designation distinguishes between molecules with multiple chiral centers, but differ only in the configuration of the site known as the anomeric carbon atom. T

oponent process theory

theory of color vision that proposes four primary colors with cones arranged in pairs: red and green, blue and yellow

Which enzymatic kinetic parameter is NOT affected by the presence of an uncompetitive inhibitor?

uncompetitive inhibitors affect both the kcat and KM of an enzymatically catalyzed reaction in a way that does not change the slope of the Lineweaver-Burk plot, which means the KM/Vmax is not changed.

A researcher replicates the experiment with the addition of a physical stressor to the first phase of the experiment. According to Selye's general adaptation syndrome, this change is:

unnecessary, because the human stress response is not specific to the type of stressor. The answer to this question is C because, according to Selye's general adaptation syndrome, people's response to various stressors is similar. Selye's theory does not make claims about avoidance-avoidance or approach-approach conflicts.

Vasopressin regulates the insertion of aquaporins into the apical membranes of the epithelial cells of which renal structure?

vasopressin regulates the fusion of aquaporins with the apical membranes of the collecting duct epithelial cells. collecting duct

How would the James-Lange theory of emotion explain the aggressive emotions experienced by the participants in the experimental condition? Participants experience physiological arousal from watching violent programs and:

James-Lange theory of emotion suggests that physiological arousal precedes the experiencing of emotions. B describes the Canon-Bard theory, which holds that physiological arousal and emotions are experienced simultaneously. C describes the two-factor theory that brings in cognitive appraisal. D is not a particular theory of emotion.

What happens to the pH of a soapy solution as a result of the introduction of hardness ions?

Removal of weak bases such as RCO2- results in a decrease in pH, and a decrease in [OH - ].

Ser

Serine (S) Polar with neutral R group but uneven charge

Which alcohol will most likely undergo substitution by an SN1 mechanism in acidic conditions?

The answer to this question is D because this alcohol produces the most stable carbocation (tertiary) and consequently will most easily lose a water molecule upon protonation of the hydroxyl group in acidic media.

What mass of Compound 1 (MW = 800 g/mol) is contained in the solution used to prepare liposomes that elute at 20 mL by size-exclusion chromatography?

The solution concentration was 0.10 mM for the liposomes that elute at 20 mL. Since the total volume of solution used in their preparation was 1 mL, the mass of lipid can be calculated as: (0.10 × 10-3 mol/L) × (1 × 10-3 L) × (800 g/mol) = 8 × 10-5 g = 80 µg.

What is the likely structure of the amino acid found at position 19 of LC20?

only serine, threonine, and tyrosine can be phosphorylated.

Which type(s) of restriction enzyme(s) can recognize the HIF binding sequence? A restriction enzyme that has: a four-base recognition sequence a six-base recognition sequence an eight-base recognition sequence

only the CCCGGG within the HIF binding sequence is palindromic. Therefore, only a restriction enzyme that recognizes a four-base sequence or a six-base sequence can recognize this sequence within the HIF binding sequence.

Some studies have found that increases in dopamine activity are associated with increased reward-seeking motivation. This finding suggests an association between dopamine levels and which type of learning?

operant conditioning The finding focuses on reward-seeking motivation, which is most closely associated with operant conditioning (change in behavior due to past outcomes).

A prion is best described as an infectious:

protein because a prion is an abnormally folded protein that induces a normally folded version of the protein to also adopt the abnormal structure, which is often deleterious.

Which variant showed the lowest catalytic efficiency?

ratio of kcat to KM is a measure of catalytic efficiency and the I276A variant exhibits the lowest comparative value of kcat/KM after analysis of the data presented in Table 1.

Which development from the passage best illustrates an organizational change in the context of child abuse diagnosis and prevention?

This Sociology question assesses Knowledge of Scientific Concepts and Principles related to organizations, which falls under the content category "Social interactions." A is the correct answer, based on the basic definition of an organization as a group with an identifiable membership that engages in concerted action to achieve a common purpose. The passage identifies the development of the pediatric subspecialty of child abuse pediatrics, which is an example of a change within pediatrics as an organization. The other options address other concepts or themes, including the proliferation of knowledge and social problems (option B), political mobilization and legal change (option C), and cultural continuity in public discourse (option D).

What is the closest distance the electrodes used in an NCV test can be placed on a nerve in order to measure the voltage change as a response to the stimulus?

This is a Physics question that falls under content category "Translational motion, forces, work, energy, and equilibrium in living systems." The answer to this question is B because the shortest time duration that can be detected is not less than the action potential time response of 1 msec, according to Figure 1. The distance travelled by the impulse during this time is 1 msec × 100 m/s = 0.1 m, which is the closest distance the electrodes must be placed on the nerve. This question requires Scientific Reasoning and Problem-Solving, correct determination and use of a scientific formula, in order to arrive at the answer.

Thr

Threonine (T) Polar with neutral R group but uneven charge

To be an effective therapy, an antisense gene that is incorporated into a genome that contains the target gene must be:

To provide effective therapy, this antisense gene would need to be regulated in a manner similar to the manner in which the target gene is regulated so that the antisense RNA is produced at the same time that the sense mRNA is produced. This would ensure that the antisense RNA is available to bind with the sense mRNA, thereby preventing its subsequent translation.

Tyr

Tyrosine (Y) Polar with neutral R group but uneven charge Aromatic R group PI = 10.5

Radioactively labeled uracil is added to a culture of actively dividing mammalian cells. In which of the following cell structures will the uracil be incorporated?

Uracil is a component of RNA. Therefore, one would expect to find the radioactively labeled uracil in cell structures that contain RNA. B is the best answer because ribosomes contain rRNA and proteins

Which statement does NOT describe a monocular depth cue?

Using the distance from the object of focus as a depth cue is associated with retinal disparity, which is a binocular depth cue.

In regard to their relative size, the described objects are:

Virus structure cannot be visualized with a light microscope. The limit of resolution of a light microscope is about 200 nm. The virions described in the passage are below that limit, 100nm. Bacteria and eukaryotic cells such as the human red blood cell are well above that size so they can be seen with a light microscope, so choices B and C are incorrect. Bacteriophages are viruses in bacteria so they must be much smaller than bacteria, so choice D is not correct. The correct answer is A, they are smaller than all known eukaryotic cells.

The adrenal medulla is part of which branch(es) of the peripheral nervous system?

adrenal medulla secretes epinephrine and norepinephrine in response to short-term stress. Reactions to short-term stress are mediated by the sympathetic branch of the autonomic nervous system

What was the most likely purpose of adding bovine serum albumin to the kinetics experiments in the passage? Bovine serum albumin:

albumin is a protein that mobilizes proteins and lipids in serum. In the context of a kinetics experiment it is perfectly logical to assume that albumin is added to maintain homogeneity and prevent the enzyme from adhering to walls and other surfaces which would inhibit its activity.

Based on the data presented in Table 1, which amino acid residues of PRR are involved in binding to prorenin? Residue 109 Residue 140 Residue 201 Residue 269

amino acid substitutions at positions 140, 201, and 269 result in changes to the Kd, while a non-conservative substitution at position 109 yields no change to the Kd.

G542X is another CFTR allele. If a female heterozygous for G542X bears a child fathered by a male heterozygous for the ΔF508 allele, what is the probability that the child would be homozygous for the G542X allele, given that neither parent has CF?

both parents would need to carry the G542X allele in order for a child to be homozygous for the G542X allele. Because only the mother carries the G542X allele, the probability that the child will be homozygous for the G542X allele is 0.

Which observation disconfirms the theory that the hunger drive is based on a person's interpretation of stomach contractions, while satiety is based on stomach distension?

such patients, who are incapable of interpreting any signals from a nonexistent stomach, do experience hunger

In operant conditioning studies, the subject's motivational state is most typically operationally defined by:

the method that is usually used, in operant conditioning studies, to operationally define the subject's motivational state. depriving the subject from a desired stimulus for a certain amount of tim

Disruption of two disulfide bonds within a protein requires:

the reduction of a disulfide bond requires two electrons and each molecule of NADH can provide two electrons when reducing another compound.

If sounds produced by the human vocal cords are approximated as waves on a string fixed at both ends, and the average length of a vocal cord is 15 mm, what is the fundamental frequency of the sound? (Note: Use 3 m/s for the speed of sound through the vocal cord.)

the relationship between the sound speed and frequency for a string fixed at both ends must be used. The fundamental frequency of a vibrating string is given by f = v/(2L) = (3 m/s)/(30 × 10-3 m) = 1/10-2 Hz = 100 Hz. This incorrect answer results from using the sound speed as 30 m/s

overextension

the use of a given word in a broader context than is appropriate

When experiments are performed on enzymes that display traditional Michaelis-Menten kinetics, what shape does the graph of V0 versus substrate concentration [S] have?

traditional Michaelis-Menten kinetics describes a hyperbolic dependence of V0 on substrate concentration

Which amino acid residue in the pilin subunit is most likely modified by PptB?

Based on the passage, PptB is a phosphotransferase. Of the amino acid options, only serine can be modified by a phosphotransferase.

In [Cu(NH3)4]2+, the subscript 4 indicates which of the following?

Because ammonia is neutral, the number 4 reflects only the number of ammonia molecules that bind to the central Cu2+ cation and does not indicate anything about its oxidation number. B. The coordination number of Cu2+ only

If restriction of blood flow to the kidneys (by placing clamps on the renal arteries) resulted in an immediate but small increase in blood pressure, followed by the gradual development of severe hypertension, which hypothesis would these results best support?

The reduced flow of blood through the renal arteries due to the clamps would cause a decrease in glomerular blood pressure. The kidneys respond to this drop in pressure by activating the renin-angiotensin system of hormones. This increases the amount of sodium and water that is reabsorbed by the kidneys, therefore increasing blo

The liver synthesizes factors that act cooperatively with platelets to facilitate which physiological process?

Platelets form a plug at the site where a blood vessel has been damaged. Blood clotting factors that have been synthesized in the liver in an inactive form then participate in a cascade that leads to a blood clot.

Which of the following properties is associated with the existence of glycine as a dipolar ion in aqueous solution?

Polarity in neutral molecules results from an uneven distribution of electron density, which can arise from separation of unlike charges. This occurs in zwitterions and in ylides. In addition, molecules that contain strongly electron-withdrawing or electron-donating substituents are highly polar and possess correspondingly high dipole moments. Thus, answer choice A is the best answer. High dipole moment

Hawthorne effect

A change in a subject's behavior caused simply by the awareness of being studied

Which change to an equilibrium mixture of compounds 1-4 will increase the ratio of Compound 4 to Compound 1?

Application of Le Châtelier's principle can be used to arrive at the key. The ratio of Compound 4 to Compound 1 can be increased by removing some Compound 3 or adding Compound 2.

In the equilibrium constant expression for Equation 2, [H2O] is omitted because the salt is:

Aqueous acid-base equilibria very often feature water acting as either an acid or a base. It is also the solvent for aqueous systems. In the equilibrium constant expression for such reactions, water is usually omitted. The simplest explanation is that, because it is the solvent, the concentration of water stays essentially constant throughout the reaction. In such a case, that constant could be included in the value of Keq, the equilibrium constant for the reaction. The salt in Equation 2 is the fertilizer (NH4)2HPO4, and the reaction shows that the substance is basic.

Arg

Arginine (R) Basic -> + charge at pH 7 R group PI = 12.5

Synthesis of antibody proteins in eukaryotic cells is associated with what organelle?

As secreted proteins, antibodies are translated by ribosomes attached to the rough endoplasmic reticulum.

Asp

Aspartate (D) Acidic-> - charge at pH 7 R group PI = 3.9

Beck's Cognitive Theory

Certain cognitive style is a pre-existing condition that makes people vulnerable to depression

Individuals who have the ability to delay gratification in pursuit of long-term rewards are most likely to be categorized as having which type of intelligence?

D. Emotional Emotional intelligence refers to the ability to perceive, express, understand, and manage one's emotions. Emotionally intelligent people are self-aware and can delay gratification in pursuit of long-term rewards, rather than being overtaken by immediate impulses. None of the other answer choices is related to delaying gratification.

If the speed of the charged particle described in the passage is increased by a factor of 2, the electrical force on the particle will:

Electrical force depends on the particle's charge and the strength of the electric field experienced by the particle, not on the particle's speed.

Gly

Glycine (G) hydrophobic, nonpolar R group

Based on the structural studies, when NAG3 interacts with amino acids in HEW, it can hydrogen bond to the side chains of the amino acids at all of the positions described EXCEPT for the amino acid at which position?

NAG3 interacts with the amino acids at positions 59, 62, 63, 101, 103, and 107. The side chains of the amino acids at all of the positions contain sites for potential hydrogen bonding with the exception of alanine which is at position 107.

Antidiuretic hormone (ADH) acts to decrease urine output by increasing the water permeability of the walls of:

Passage of fluid from the blood through the walls of the glomerulus is a passive process and depends mostly on the hydrostatic pressure of the blood in the glomerular capillaries, so answer A is incorrect. The walls of Bowman's capsule do not take part in urine processing, so answer B is incorrect. ADH does not exert its effects on the loop of Henle so C is incorrect. ADH acts on the distal tubule and collecting duct to increase water permeability by the insertion of aquaporins in the plasma membrane. This increases water reabsorption, which reduces urine output, so D is correct.

The information in the passage supports the prediction that P. falciparum creates unique protein trafficking structures outside the parasite itself for the trafficking of which parasite protein?

PfEMP1 is a parasite protein that is present in the plasma membrane of the RBC that the parasite inhabits. This suggests that there must be a mechanism for transporting PfEMP1 from the parasite to the RBC plasma membrane.

Colchicine most likely relieves gout symptoms through what mechanism?

Phagocytosis requires that the cell change shape dramatically as it surrounds and engulfs large extracellular particles. Microtubules are one of the cytoskeletal elements that help determine cell shape. This function relies on the ability of the microtubules to disassemble and reorganize. The drug colchicine inhibits microtubule reorganization and would therefore inhibit phagocytosis of uric acid crystals by leukocytes.

The transition of which amino acid from the interior of a soluble globular protein to the surface is associated with the largest positive ΔGº?

Phe is a hydrophobic amino acid that makes favorable interactions within a protein interior but does not make favorable interactions with solvent. This means that any transition to solvent would involve an unfavorable, or positive, ΔGº.

What is the approximate number of wavelengths of light that can travel in 1 direction within a retroreflecting bead that has a diameter of 5 × 10-5 m? (Note: The speed of light = 3 × 108 m/s, and its frequency is approximately 1015Hz.)

The item gives the speed of light to be v = 3.0 × 108 m/s and the frequency to be f = 1015 Hz. The wavelength λ of the light may be found from the relationship for wave speed v = λf, thus, λ= v/f = (3.0 × 108 m/s)/( 1015 Hz) = 3.0 × 10-7 m. The number of waves that will fit along a diameter is found by dividing this wavelength into the diameter D = 5.0 × 10-5 m, so the number of waves is D/λ =(5.0 × 10-5 m)/( 3.0 × 10-7 m) = 1.7 × 102.

A 7-N force and an 11-N force act on an object at the same time. Which of the following CANNOT be the magnitude of the sum of these forces?

The largest net force occurs when the forces point in the same direction; then, the magnitudes will add mathematically to a value of 18 N. Similarly, the smallest net force occurs when the forces point in opposite directions and the lesser magnitude is subtracted from the greater; this results in a value of 4 N. Of the answers listed, only A (2 N) does not fall between these values.

What is the [H3O+] in the solution used in the experiment done at the lowest pH?

The lowest pH is 6. Because pH = -log([H3O+]), [H3O+] = 1.0 × 10-6 M or 1.0 μM.

In a mating of two Tetrahymena strains that are homozygous in their macronuclei and heterozygous in their micronuclei for a recessive gene, what percentage of the F1 generation will express the recessive phenotype?

The macronuclei do not participate in mating so only the genotypes of the micronuclei need to be considered. If we call the recessive gene r and its dominant allele R, then a cross between two heterozygote strains Rr will produce the genotypes RR:Rr:rr in a ration of 1:2:1. The recessive gene will be masked so that its expression will not be observable if a dominant gene is present in the genotype. Only in the double recessive (rr) case will the presence of the recessive gene be observed. The answer then is that on the average 25% of the offspring will demonstrate the recessive phenotype (choice B).

The cell type in the male reproductive system that is most analogous to the female ovum is the:

The mature ovum is the female gamete that has completed meiosis and contains the haploid number of maternally derived chromosomes. This makes it most analogous to spermatozoa, the mature male gametes that contain the haploid number of paternally derived chromosomes.

By what factor would a string's tension need to be changed to raise its fundamental frequency by a perfect fifth?

The relation between string frequency and tension is given in the passage as f α T1/2. To raise the string frequency by a perfect fifth (a factor of 3/2), the tension must be increased by a factor of (3/2)2 = 9/4

Based on the passage, which statement describes Wnt proteins?

based on the passage, Wnt proteins are a family of secretory proteins with isoelectric points around 9, implying that they are positively charged at physiological pH.

If a thin thread is placed between a screen and a bright source of light, a pattern of parallel dark and bright fringes appears on the screen. The phenomenon best explaining the formation of this pattern is:

diffraction The thin thread disrupts the propagation of light by impeding light to pass through it. Diffraction causes the initially plane-parallel wave-fronts of light to change direction and partially enter the shadow region behind the thread due to its narrowness. The overlapping of different wave-fronts on the screen causes the pattern of dark and bright fringes on the screen according to the phase difference between the wave-fronts that interfere there.

The finding from Study 3 regarding the differential effect of emotional arousal on memory for central and peripheral details is best explained by which mechanism? Increasing emotional arousal:

emotional arousal seems to focus a person's attention on the central features of an event. The hypothesis (Easterbrook, 1959) that a person will notice information that elicits arousal, but fail to process other information, has been supported by studies in which memory for an (emotional) event's "central" aspects (directly tied to the emotion elicitor) is compared to memory for "peripheral" aspects (removed from the source of the emotional arousal). Thus, memory for the fundamental gist of the emotional event is retained, whereas memory for details (if they are encoded at all) either fades or undergoes changes.

A partial DNA sequence of the coding strand of a gene is shown. 5-GACATGGACTCGCTA-3 Which sequence corresponds to the mRNA for this DNA sequence?

for any given gene, the nucleotide sequence of both the coding strand and the mRNA are complementary to the sequence of the template strand.

Why was the total amount of Rac measured in Experiment 2?

in order to determine how the presence or absence of VopC affects Rac activation, it is important to know if the total amount of Rac is affected by the strain being used. If the Rac expression is upregulated in one strain and not another, this will affect interpretation of the gel results.

Which concept takes into account the negative impact of long-term exposure to stressful events addressed in the first paragraph?

general adaptation syndrome (GAS) is a model of the body's stress response that consists of three stages: alarm, resistance, and exhaustion. An individual enters the stage of exhaustion only after that individual has encountered the stressor for a prolonged period of time.

The amino acids Asp6, Asn9, Thr10, His11, and Arg27 are found near the ADP binding site of PanK3. Which two amino acids contribute to the stabilization of ADP binding described in the passage?

he answer is B because both His and Arg side chains can carry a positive charge. They would have favorable ionic interactions with the negatively charged phosphates of ADP. This is Scientific Reasoning and Problem Solving question because you must use the scientific model of favorable interactions in ligand binding to determine which amino acid side chains can make electrostatic interactions with ADP.

Which of the four DNA bases contains the largest number of hydrogen bond acceptors when involved in a Watson-Crick base pair?

hydrogen bond acceptors are N and O. Adenine contains 1 donor and 1 acceptor, thymine contains 1 donor and 1 acceptor, guanine contains 2 donors and 1 acceptor, and cytosine contains 1 donor and 2 acceptors.

The diagram shows the size and position of the exons (numbered) and introns (lines) of a gene that codes for hypothetical Protein X, which can exist as two isoforms (either 16 or 17 amino acid residues long).

isoforms are produced through alternative splicing of pre-mRNA. Based upon the lengths of the two isoforms, each isoform must contain at least exons 1 and 4. From that, it can be determined that one consists of exons 1, 2, and 4 (17 residues) and the other of exons 1, 3, and 4 (16 residues). Generating cDNA from mRNA will generate a sequence for each isoform, as splicing will have occurred. Since both isoforms must contain exons 1 and 4, primers overlapping exons 1 and 4 will produce cDNA for both of the isoforms. The band pattern of the gel will identify whether both or only one of the isoforms is expressed.

Capillaries in the kidney and elsewhere in the body maintain fluid homeostasis by balancing hydrostatic and osmotic pressures. Which of the following is the initial effect of a blood clot forming on the venous side of a capillary bed?

it is necessary to know that blood flows from arteries to capillaries and then to veins. If flow is blocked at the venous side, blood would accumulate in the capillaries. Thus, hydrostatic pressure would build up in the capillaries, causing a net increase in fluid flow into the interstitial spaces.

Which statement is NOT consistent with a conflict theoretical analysis of the findings in the passage?

it represents a functionalist understanding of religion, with the reference to social solidarity. The other options each articulate how a conflict theorist, focused on social control and social inequality, would view the role of religion in society

Finding the number of stereoisomers in a compound

2^n where n is the number of chiral centers

Which participant in the electron transport chain has the greatest attraction for electrons?

A and B are incorrect because FAD and NAD+ do not directly donate electrons to the electron transport chain—it is their reduced forms that do. Electrons move to a slightly more electronegative carrier as the electrons pass through each step in the electron transport chain. Therefore, the final electron acceptor of this chain, oxygen (C), has the greatest attraction for electrons.

Which amino acid is LEAST likely found in one of the transmembrane domains of GPCR43?

A because aspartate (Asp) has a negatively charged R group and is very hydrophilic

What physical quantity is represented by the area of quadrilateral OABC in Figure 2?

A clue as to what the area represents may be found in the units of the area. The area units would be the vertical axis units of speed (m/s) times the horizontal axis units of time (s) to yield distance units (m). This leads one to conclude that the total area yields the total distance traveled by the mass.

Approximately how many moles of Al3+ are reduced when 0.1 faraday of charge passes through a cell during the production of Al? (Note: Assume there is excess Al3+ available and that Al3+ is reduced to Al metal only.)

A faraday is equal to one mole of electric charge. Because each aluminum ion gains 3 electrons, 0.1 faraday of charge will reduce 0.1/3 moles of aluminum, or 0.033 moles of aluminum.

Which of the following statements most accurately describes the solubility properties of fatty acid salts?

A fatty acid salt contains a long hydrocarbon chain, which is soluble in nonpolar solvents. The salt also contains the charged group -CO2-Na+, which is soluble in polar solvents. Thus, C is the best answer.

binary fission

A form of asexual reproduction in single-celled organisms by which one cell divides into two cells of the same size

Which hypothetical result from the online questionnaire would provide evidence of the fundamental attribution error?

A. Some participants rank personality factors above environmental factors. the fundamental attribution error refers to stressing the importance of dispositional (i.e., personality) factors in one's explanations of other people's behavior and underemphasizing situational factors.

When researchers determined the total cellular concentration of ATP in AlP-exposed rat liver cells, they found the concentration to be equal to the control value. Which conclusion about the metabolic state of the cell is best supported by these data?

ATP production is the same in both control and AlP-exposed cells, and the data in the passage show that mitochondrial ATP production is decreased. This indicates that the flux through glycolysis is increased, because this would be the major pathway for ATP production once the electron transport chain is shut down. A. Glycolytic flux is increased after AlP treatment.

According to Figure 1, at approximately what plasma concentration of glucose is the Tm (320 mg/min) reached?

Above 10 mg/mL, glucose begins to be found in the urine. The Tm for glucose is therefore 10 mg/mL, answer choice B. This can be read from the graph by looking at the concentration in the plasma where the concentration in the urine is zero. In other words, where the clearance line for glucose crosses the axis.

Based on the data presented in the passage, which statement best describes the HSP110ΔE9 allele?

According to Figure 1, as compared to HSP110WT, HSP110ΔE9 slows tumor growth, and thus, it is cancer-suppressing. According to the data in Table 2, HSP110ΔE9 counteracts the ability of HSP110WT to prevent protein aggregation and prevent apoptosis; thus, the HSP110ΔE9 allele is dominant to the HSP110WT allele.

According to Equation 1, the concentration of the polymer with respect to ​​ [HPO4^-2] is equation: nCDP --PNP,Mg+2--> (CP)n + n[HPO4^-2]

According to the balanced coefficients in Equation 1, each time one molecule of the polymer ​​(CP)n is formed, there also are n hydrogen phosphate ions ​​ n[HPO4^-2] produced. Thus, the concentration of ​​(CP)n must be ​​1/n times the concentration of the ​n[HPO4^-2]

All the participants in the study are given information regarding the benefits of a healthy diet. According to the cognitive dissonance theory, which hypothetical finding is most likely?

According to the cognitive dissonance theory, when an individual's attitudes are incongruent with his or her behavior, this leads to cognitive dissonance. To eliminate cognitive dissonance, the individual can either change his or her attitudes or his or her behavior. The theory posits that individuals are more likely to adjust their attitudes to align with their behavior than the other way around. Therefore, obese participants are likely to question the importance of the information provided.

During the ACC1- or ACC2-catalyzed formation of malonyl-CoA, what is the structure of the functional group added to acetyl-CoA?

According to the passage, ACC1 is a carboxylase and will convert acetyl-CoA to malonyl-CoA through the addition of a carboxyl group to acetyl-CoA.

The researchers want to generate a mutant mouse strain that expresses a constantly active variant of ACC2 by replacing the amino acid at position 212 with a different residue. Which residue is the best choice as a replacement?

According to the passage, AMPK inactivates ACC2 through phosphorylation of the residue at position 212. To create a constitutively active variant, the residue at position 212 should be substituted with a residue that cannot be phosphorylated. Of all the amino acids listed, only alanine cannot be phosphorylated.

The process taking place at the cathode was:

According to the passage, H2(g) was produced at the cathode. The reaction taking place at the cathode was 2H+(aq) + 2e- → H2(g), which means the H+ was gaining electrons and undergoing reduction. Thus, D is the best answer. reduction by a gain of electrons

A man with a CRC mutation that results in the synthesis of HSP110ΔE9 and a woman that does not carry this mutation in any of her tissues have a child. What is the percent chance that the child will inherit the CRC mutation?

According to the passage, HSP110ΔE9 transcripts are not present in noncancerous tissue of people with MSI CRC. This means that the deletion mutation in the HSP110 T17 microsatellite is a somatic, not germ line, mutation. Therefore, the mutation will not be inherited, and the percent chance that the child of a person with this mutation will inherit it is 0%.

The information in the passage suggests that PfSET10 has which function in var gene localization or expression? PfSET10:

According to the passage, P. falciparum cells contain the most PfSET10 when the intraerythrocyte parasites are in an actively dividing life cycle phase. To maintain cellular identity, there has to be a mechanism for marking genes that were transcriptionally active before mitosis for reactivation after mitosis. Because PfSET10 colocalizes with the active var gene, it is reasonable to hypothesize that PfSET10 methlytransferase activity is involved in this bookmarking.

Which of the following processes is LEAST directly influenced by adrenergic drugs?

According to the passage, adrenergic drugs mimic activation of the sympathetic nervous system; therefore, the best answer will be the process that is LEAST directly controlled by the sympathetic nervous system. The sympathetic nervous system directly inhibits peristalsis (A) and secretion of digestive enzymes (B). It also increases the blood glucose concentration and causes dilation of the blood vessels that supply the deep muscles and internal organs, which aids nutrient delivery (D) to these tissues. The sympathetic nervous system does not directly affect the activity of digestive enzymes (C) after they have been secreted.

If Solution A contains Ag+, the anion component must be:

According to the passage, the ionic compound in Solution A was completely soluble in water. The information in Table 1 shows that the only Ag+ salt studied that is soluble in water is AgF.

What is the effect produced by the PRK technique designed to correct nearsightedness?

According to the passage, to correct nearsightedness, the laser beam is directed onto the central part of the cornea, resulting in a flattening of the cornea. This means that the radius of curvature of the cornea is increased.

Scientists have hypothesized that mitochondria evolved from aerobic heterotrophic bacteria that entered and established symbiotic relationships with primitive eukaryotic anaerobes. According to this hypothesis, the bacteria that entered primitive eukaryotic cells were able to carry out which function(s) that the primitive eukaryotic cells could not?

According to the provided information, the primitive bacteria were aerobic and heterotrophic, whereas the primitive eukaryotes were anaerobic. If the primitive eukaryotes were anaerobic they could not exist in an oxygen-containing environment nor engage in metabolic processes requiring oxygen. Thus, the bacteria were likely to be able to carry out the oxygen-requiring reactions of the citric acid cycle and the electron transport chain while the primitive eukaryotes were not.

The energy, E, of a hydrogen atom with its electron in the nth shell of a hydrogen atom is given by E = -C/n2 where n = 1,2,3, . . . and C is a positive constant. If an electron goes from the n = 2 shell to the n = 3 shell:

According to the relation given in the passage, the energy of an electron in orbit n = 3 is less negative or greater than the energy in orbit n = 2. Thus energy is required to make the transition from n = 2 to n = 3 and the atom gains energy.

Cytochromes

An iron-containing protein that is a component of electron transport chains in the mitochondria and chloroplasts of eukaryotic cells and the plasma membranes of prokaryotic cells redox reactions

The reacting substrate carbon atom in the mechanism described in the passage undergoes which of the following hybridization state changes during the reaction? (Note: the middle hybridization state refers to an intermediate.)

As described in the passage, deprotonation of water is the preliminary step in the reaction sequence. This knowledge and knowledge of the reaction pathways that lead to amide hydrolysis are enough to conclude that the sequence of events involves nucleophilic attack of coordinated hydroxide on the sp2-hybridized carbonyl carbon atom to generate an sp3-hybridized and tetrahedral intermediate which subsequently eliminates an amine (after proton transfer) to generate an sp2-hybridized carbon atom.

The normal path of sperm movement from the male testis to the point of fertilization in the female is

As sperm cells leave the testis they travel through the epididymis to the vas deferens and into the urethra. The sperm then enter the female's vagina, travel through the cervix and uterus, and enter the fallopian tube, where fertilization most commonly takes place.

To keep the current constant during the discharge cycle:

As the capacitor discharges the voltage across it falls, thus to maintain a constant current, R must be proportionately reduced. This is so from Ohm's law, I = V/R. To keep I fixed, R must fall with V. This is answer B. B. R must continually decrease

Which of the following expressions is equal to a, the acceleration vector of the box as it moves from Point B to Point C? (Note: The coefficient of friction between the box and the surface between Point B and Point C is equal to μkh)

As the mass slides across the horizontal surface it will slow due to a leftward kinetic friction slowing the mass, as friction opposes the motion. The kinetic friction is given by -μkhNi, where N denotes the normal force of the surface on the bottom of the mass. For a free mass on a flat surface, the upward normal force cancels the downward gravity force, mg, hence N = mg which yields a friction force -μkhmgi. This is the net force that acts equals ma, according to Newton's Second Law, thus ma = -μkhmgi or a = -μkhgi

As atomic number increases in a horizontal row of the periodic table, ionization energy generally:

As the number of protons increases, the attraction between the nucleus and electrons also increases. In general, more energy is required to remove an electron from an atom for elements on the right than those on the left of the same period of the periodic table due to the greater effective nuclear charge experienced by the former.

The pKa for the dissociation of H2PO4- ​​ to ​​HPO4^-2 is 6.7. What is the initial ratio of ​​ [HPO4^-2]:[H2PO4^-] in the buffer solution of Experiment 1?

At a given temperature, the relative concentrations of the weak acid H2PO4- and its conjugate base HPO4-2 will depend on [H+] and the Ka of the acid, as can be seen when the Ka expression is rearranged to Ka / [H+] = [HPO4-2]/[H2PO4-]. Since the solution in Experiment 1 was buffered to pH = 8.7, then [H+] is held constant. Since Ka is also a constant, the ratio of the concentrations of weak acid and conjugate base must be fixed. This situation is frequently encountered in solution chemistry.

Gas X has a density of 1.44 g/L and gas Y has a density of 1.54 g/L. Which gas diffuses faster?

Avogadro's law states that equal volumes of different gases at the same temperature and pressure contain the same number of molecules. According to the density values given, 1 liter of gas X weighs 1.44 g and 1 liter of gas Y weighs 1.54 g. Since each sample has the same number of molecules, gas X must have a lower molar mass than gas Y. A lighter molecule diffuses faster than does a heavier one, so gas X diffuses faster than gas Y.

Which of the following recombinant processes depends on the F factor plasmid?

Bacteria can exchange genes by three processes: conjugation, transformation and transduction. The process of conjugation involves production of a special conjugation pilus (sex pilus) by one bacterium and transfer through it of DNA to another bacterium. It requires special genes for the pilus and these are usually present on a plasmid, a separate extragenomic strand of DNA not incorporated into the bacterium's own DNA. This plasmid is referred to as the fertility or F factor. Conjugation is a feature of Gram-negative bacteria. It confers the advantages of sexual reproduction on the bacterium. The plasmid benefits by being able to move from one host bacterium to another through the conjugation pilus. Conjugation is the process described in the stem, so choice C is correct. Choice A and B are the other two processes that do not require a plasmid. In transformation bacteria take up DNA from their surroundings, the media in which they are immersed. Transduction is the process whereby genes are transferred by a virus. Choice D is also incorrect; translocation, is a general term used in biology to describe movement from one place to another (genes on a chromosome, proteins in a cell, sap in a tree).

If Anolis lizards have X-Y chromosomal sex determination, the locus of a gene for the UV reflectance pigment:

Based on the information presented, the gene encoding UV-reflectance pigment could be on a sex chromosome or an autosome. The fact that the pigment is expressed in the dewlap, a structure found only in males, is not sufficient to eliminate any chromosome as the location of this gene.

Which change in a property of pilin will be observed after modification by PptB?

Based on the passage, PptB is a phosphotransferase, which adds a phosphoglycerol to pilin subunits. The addition of this group (phosphoglycerol) adds a negative charge to pilin and thus decreases its isoelectric point.

Suppose that a stream of fluid flows steadily through a horizontal pipe of varying cross-sectional diameter. Neglecting viscosity, where is the fluid pressure greatest?

Bernoulli's equation for horizontal fluid flow (when viscosity is neglected) states that the pressure plus kinetic energy density of the fluid is constant. If the fluid velocity falls, the kinetic energy density decreases so the pressure increases. The fluid velocity is lowest where the diameter of the pipe is a maximum. Thus, the pressure is greatest where the diameter is a maximum.

Suppose a blood sample tested above the range (6.0 mg/dL) of the standards used in the experiment. What modification will provide a more precise reading by data interpolation as opposed to extrapolation using the same standards?

By adding solvent, the concentration of glucose will be lowered, and the resulting absorbance will fall within the range of the standards. This is easily accomplished, and the resulting calculations that account for the dilution are not difficult.

The diameter d versus elution volume V for liposomes using identical chromatographic conditions is shown. d (nm)V (mL)2001015015100205030 Using the ultrasonication and agitation method of liposome preparation, what concentration of lipid is necessary to prepare liposomes with a diameter of 250 nm

By consulting Figure 1 it can be estimated that liposomes with a diameter of 250 nm will elute at approximately 7.5 mL under size exclusion chromatography conditions used in the passage. Using the data presented in Figure 1, this means that a concentration larger than 0.20 mM must be employed in their synthesis.

Which statement reflects the medicalization of smoking and smoking cessation?

C. Smoking is an addictive behavior that can be treated via the use of pharmaceutical intervention. it suggests the expansion of medical authority to smoking cessation and identifies a specific medical therapy. Medicalization refers to the taken-for-granted process in which a problem comes to be defined and treated by the social institution of medicine. A behavior undergoes medicalization when both the definition of the problem and the therapy intended to improve it are couched in medical terms.

Which hypothesis of cultural assimilation, social support, and health outcomes is best supported by the passage information about immigrant groups? More assimilated groups will have:

C. lower levels of support and worse overall health. In the section on social interaction factors, it is suggested that strong social support in local immigrant communities may partly explain the relatively good health of individuals from some immigrant groups in the United States (when compared to U.S.-born individuals with otherwise similar demographic characteristics). Immigrant groups that are more assimilated (or as they become assimilated) tend to have worse health outcomes (or lose their previous health advantages) than less assimilated immigrant groups. Thus, the correct response reflects the hypothesis that more assimilated groups are likely to have less social support over time. Reducing that protective factor could thus lead to worse health overall

Which component of attitudes is assessed by the study's online questionnaire?

C. Cognitive The questionnaire is described as assessing participants' ideas about how people gain weight, as well as their beliefs about the relative contribution of genetics, personality, and environment in gaining, maintaining, and losing weight. This description identifies beliefs and ideas, which are part of the cognitive component of an attitude. The other possible answers are other components that are not relevant to the description of the questionnaire (which measured explicit attitudes).

Under what condition would the level of calcitonin tend to increase?

Calcitonin reduces bone resorption. Bone resorption occurs when the level of calcium in the blood plasma is low, but resorption is not needed when the level of calcium is high. Therefore, resorption would be reduced by calcitonin under conditions in which the level of calcium in the plasma is high.

Production of which of the following hormones will be inhibited by the administration of dietary calcium to prevent osteoporosis?

Calcium levels in the blood need to be kept constant. Parathyroid hormone and calcitonin regulate blood levels of calcium. The passage gave the information that calcitonin analogs inhibit osteoporosis. If calcitonin inhibits osteoporosis, it must function to take calcium out of the blood and into the bone while preventing the loss of calcium from bone into the blood. High levels of blood calcium available for deposit into bone should stimulate this process rather than inhibit it.

Experiment 1 was repeated with 0.40 g of calcium, and the gas that evolved was collected. The identity of the gas, and its approximate volume at 1.0 atm and 27°C were:

Calcium undergoes the following reaction with water: Ca(s) + 2H2O(l)→ Ca2+(aq) + 2OH-(aq) + H2(g). The gas produced was H2. If 0.40 g calcium reacted, then the number of moles of calcium reacted was equal to (0.40 g)/(40.1 g/mol) = 0.01 mol. The amount of H2 formed was also 0.01 mol. At 1.0 atm and 27°C, the volume of 0.01 mol H2 = [(0.01 mol)(0.0821 L×atm/mol×K)(300 K)]/1.0 atm = 0.246 L = 246 mL.

McDonaldization

Chains are predicable, uniform, efficient and automated; however, homogeny of the organization leads to loss of originality/creativity

What is the electronic configuration of the Co(II) center found in vitamin B12?

Co(II) is a dication and is formed from the atomic element by the loss of two 4s electrons. As a consequence, only seven 3d electrons remain in the valence shell. [Ar] 3d^7

Which of the two products was detected during the experiment?

Compound 2b has a much larger set of conjugated double bonds than Compound 2a. In the passage it was stated that Compound 1 was chosen as a substrate since it gave rise to an equilibrium mixture of products, one of which was intensely colored.

A 15.0-eV photon collides with and ionizes a hydrogen atom. If the atom was originally in the ground state (ionization potential =13.6 eV), what is the kinetic energy of the ejected electron?

Conservation of energy requires that the 15.0 eV photon energy first provides the ionization energy to unbind the electron, and then allows any excess energy to become the electron's kinetic energy. The kinetic energy in this case is 15.0 eV - 13.6 eV = 1.4 eV

A physician approaches a new patient with the assumption that the patient is not well educated and thus less knowledgeable about health issues. Does this scenario illustrate discrimination?

D. No; the scenario describes an attitude but does not specify differential treatment. The hypothetical physician displays a biased attitude, and thus a prejudice potentially based on a stereotype. However, no action or behavior is specifically identified with the scenario in the question. Without a description of differential treatment or behavior, discrimination is not identified.

How would a structural functionalist interpret the efficacy of yoga as part of a smoking cessation therapy?

D. The utility of yoga as an effective smoking cessation therapy is an unintended, though beneficial, outcome of a yoga practice. it describes a latent, or unintended, function. Becuase the expected function of yoga is not specifically smoking cessation, its utility as a cessation therapy is a latent function of the social activity. The sociological paradigm of functionalism makes a distinction between manifest, or intended, and latent, or unintended, functions of social activities. From the functionalist perspective, almost all social actions have both manifest functions and latent functions, both of which are connected to overall social stability. The other response options with the question are better linked to the conflict or symbolic interactionist perspectives in sociology.

Enzymes alter the rate of chemical reactions by all of the following methods EXCEPT:

D. altering substrate primary structure.

The information in the passage best supports the conclusion that Intron 8 of HSP110 most likely contains which of the following?

Deletions in the T17 microsatellite located in Intron 8 of HSP110 cause Exon 9 to be omitted from the final mRNA sequence during pre-mRNA processing. This would occur during alternative splicing, which strongly suggests that the microsatellite contains sequences that influence splicing. B. Splice acceptor site

Which of the following figures best illustrates demographic transition theory?

Demographic transition theory addresses changes in the birth rate and the death rate that are associated with economic development (specifically, related to industrialization). The typical pattern begins with a drop in the death rate, leading to population growth, followed by a drop in the birth rate, leading to population stabilization.

Which other cellular components are likely to be located near the lacY6xbs transcript in the cell membrane?

E. coli membranes are roughly 75% protein and 25% phospholipid, by mass.

Based on the passage, what can be determined about the composition of the mRNA that encodes the protein from which ghrelin is cleaved?

Each amino acid is coded by 3 nucleotides; thus, 84 nucleotides are needed to code for the 28-amino-acid peptide ghrelin. To this number, the triplet that codes for the STOP codon needs to be added. Therefore, 87 is the minimum number of nucleotides for the 28-amino-acid peptide ghrelin. However, the passage indicates that ghrelin is cleaved from a longer peptide chain. For this reason, the total number of nucleotides is higher than 87.

According to the hypothesis described in the passage, the bacteria that entered primitive eukaryotic cells were able to carry out which of the following functions that the primitive eukaryotic cells could NOT?

Electron transport in mitochondria and bacteria is described and the electron transport system is needed for eukaryotes to use oxygen. The acquisition of aerobic bacteria as partners would have provided them with the ability to carry out aerobic metabolism through use of the Krebs cycle and electron transport.

According to Hypothesis A, enhanced activity of which of the following basic muscle types would be most likely to cause hypertension?

Enhanced activity of smooth muscles in blood vessels would cause vasoconstriction, and according to Hypothesis A, increased vasoconstriction is a major cause of hypertension.

Nondisjunction

Error in meiosis in which homologous chromosomes fail to separate. Nondisjunction can occur during anaphase of mitosis, meiosis I, or meiosis II.

Which of the following statements correctly describes the distinction between the exocrine and endocrine portions of the testis?

Exocrine glands secrete their products through ducts; endocrine glands release their products into the bloodstream.

Which interpretation(s) is(are) consistent with the observations in the passage? Surface amino acids of Protein X are mostly hydrophilic in aqueous solution. Surface amino acids of Protein X are mostly hydrophilic in presence of DPC. Surface amino acids of Protein X are mostly hydrophobic in presence of DPC.

Figure 2 indicates that Protein X has few exposed hydrophobic surfaces in aqueous solution. In the presence of DPC, hydrophobic amino acid residues are exposed on the surface, as evidenced by ANS binding and increased fluorescence levels I and III only

Uric acid enters the urine both through filtration and secretion in the kidney. The process of filtration of uric acid in the kidney takes place in the:

Filtrate is formed as fluid passes from the glomerular capillaries through the glomerular membrane into the Bowman's capsule. This region of the nephron is known as the glomerulus

Compound 3 is prepared from Compound 2 (Figure 2) by:

First, NaBH4 reduces the ketone to a secondary alcohol, the gamma-hydroxyester intermediate. Secondly, the alcohol group in this intermediate then reacts as a nucleophile with the carbonyl in the ethyl ester in the same molecule, forming a new ester by displacing C2H5OH as a leaving group. This cyclic ester is called a lactone, and the intramolecular transesterification yielding this lactone is called lactonization.

Na2CO3 + HCl → CO2 + H2O + NaCl Consider the above unbalanced equation. For this reaction, how many mL of a 2 M solution of Na2CO3 are required to produce 11.2 L of CO2 at STP?

For this reaction, the balanced equation is: Na2CO3 (aq) + 2 HCl (aq) → CO2 (g) + H2O (l) + 2 NaCl (aq) The problem seeks the volume of 2 M Na2CO3 solution which, when reacted with HCl solution, will produce 11.2 L of CO2 gas at STP (standard temperature and pressure). It is an important fact that one mole of an ideal gas, at STP, will occupy a volume of 22.4 L. Therefore, 11.2 L of CO2 gas, at STP, must represent 0.5 mole of CO2. (Virtually all gases can be approximated as ideal gases at common temperatures and pressures.) According to the balanced chemical equation, one mole of CO2 is produced when one mole of Na2CO3 reacts. Therefore, the amount of Na2CO3 required to produce 0.5 mole of CO2 gas must also be 0.5 moles. We must find the volume of 2 M Na2CO3 solution which contains the required 0.5 moles of reactant: Working through the numbers for this problem, we find that the answer is 250 mL. Thus, answer choice B is the best answer. In problems of this sort, it is crucial to enter the proper units for each number. When the units in the equation cancel out to leave the result in the correct units, it is an important indicator that the problem has been set up correctly.

Which technique separates proteins independently of their charge?

Gel filtration chromatography separates protein only on the basis of their size.

imprinted genes

Genes whose expression is determined by the parent that contributed them

In order to determine the relative speed of approach of a sound source by Doppler measurements, three of the following items of data are necessary. Which one is NOT required?

Given that v is the speed of sound in the medium, the Doppler equation for a source that is approaching (receding from) an observer can be written as f'=v/(v+-Vsource)f

In the pentose phosphate pathway, which enzyme catalyzes the production of 6-phosphogluconolactone?

Glucose 6-phosphate dehydrogenase catalyzes the conversion of glucose 6-phosphate to 6-phosphogluconolactone in the pentose phosphate pathway.

In Equation 1, HPO42- is the conjugate: 2 NH4H2PO4(s) 2 NH4+(aq) + H2PO4-(aq) + HPO42-(aq) + H+(aq)

HPO4-2 is the conjugate base of H2PO4- (answer D). An acid differs from its conjugate base only by the presence of an additional H+. A base differs from its conjugate acid only in the lack of a single H+. Since HPO4-2 has one fewer hydrogen atom, and one more negative charge, than H2PO4-, then HPO4-2 must be the conjugate base of H2PO4-.

Consider the reaction shown in Equation 1 at equilibrium. Would the concentration of [Cu(NH3)4]2+ increase if the equilibrium were disturbed by adding hydrochloric acid?

Hydrochloric acid will protonate ammonia in a Brönsted acid-base reaction and reduce the amount of ammonia present. The disturbed equilibrium responds in a way to restore ammonia, but this causes the amount of [Cu(H2O)2(NH3)2]2+ to decrease. This means that the equilibrium shifts to the left. B. No, because the equilibrium in Equation 1 would shift to the left

The extreme hyperglycemia of these animals suggests that major changes in the normal glucose regulatory mechanisms occur during freezing. Which of the following observations would support this hypothesis?

Hyperglycemia normally elicits insulin secretion. Support for the observation that major changes in the normal glucose regulatory mechanisms occur would, therefore, be supported by the observation that insulin secretion was suppressed during hibernation. Glucagon secretion is suppressed by hyperglycemia. Glycogen break-down contributes to glucose levels; a slowing of glycogen catabolism would still contribute to glucose levels but at a slower rate. Changing the sensitivity of all pancreatic endocrine responses would not be a major change in mechanism, but would instead be a change in threshold of activity. Thus, answer choice A is the best response.

After Sarah's accident, her attending physician detected the protein myoglobin in her urine. What type of injury is consistent with this observation? Broken bone Damaged muscle Damaged kidney

I and III myoglobin is the substance that holds oxygen in the muscles and organs. The physician's observation is consistent with an injury to muscle or organs, but not bone.

H. pylori infection may cause increased proliferation of mucosal cells in the stomach. This may lead to gastric cancer if:

If H. pylori infection causes increased proliferation of mucosal cells in the stomach, this could lead to gastric cancer if genetic mutations occur in proliferating somatic cells that line the stomach

If chromosomal duplication before tetrad formation occurred twice during spermatogenesis, while the other steps of meiosis proceeded normally, which of the following would result from a single spermatocyte?

If all other steps occur normally, then four sperm would be produced. Because replication occurred twice instead of once prior to tetrad formation, each sperm would have twice the normal amount of DNA. As a result, four diploid sperm would be produced instead of four haploid sperm

An ulcer that penetrated the wall of the intestine would allow the contents of the gastrointestinal tract to enter:

If an ulcer penetrated the walls of the intestine, this would allow the contents of the gastrointestinal tract to enter the peritoneal cavity. Membranes surround this cavity, which would prevent further transport of the gastrointestinal contents through the rest of the body. An ulcer in the small intestine would not allow the contents to enter the lumen because this is the normal place in which the contents are found

When an initially heterozygous macronucleus undergoes repeated binary fission, the result will be:

If haploid cells have one copy of each chromosome and diploid cells have two copies of each chromosome, then one can infer that 45-ploid cells have 45 copies of each chromosome. Polyploidy, having more than two copies of each chromosome, is unusual in animals, but important in plants and microorganisms like Tetrahymena. The amitotic division of the macronucleus will result in uneven distribution of chromosomes, hence an unpredictable genome. Answer D is therefore correct; the allele distribution will be uneven.

People who are born without sweat glands are likely to die of heat stroke in the tropics. This indicates that, under tropical conditions, the human body may

If people lack sweat glands, they are unable to make sweat nor to capitalize upon the evaporative cooling of sweat (A). These individuals are forced to rely nearly solely on vasodilation (radiation) for responding to elevated external temperatures. That individuals without sweat glands are likely to suffer heat stroke in the tropics, indicates that radiation alone is ineffective for cooling under these conditions. In fact, the human body may gain heat by absorbing radiation from the sun leading to an elevation in body temperature (B). These individuals are not effectively able to manage their body temperature under tropical conditions, given that they are likely to die of heat stroke (D). Given that the same mechanisms are used for cooling the human body in temperate and tropical zones, it is not likely these individuals are using different mechanisms to maintain body temperature in temperate zones

If an artery that supplies blood to a lung lobe was blocked but ventilation to the lobe was unaffected, how would alveolar gas partial pressures change?

If the blood flow to an alveolus were blocked there would be no flow of hemoglobin-rich red blood cells to take away O2 and no influx of CO2 from the blood. As a result the air in the alveolus would become more like that of the atmosphere. It would acquire a higher PO2 and a lower PCO2.

A peptide consisting of nine amino acids was partially hydrolysed. Three different tripeptides were isolated. None of the tripeptides share a common amino acid. Based on the data, what is the total number of possible structures possible for the full-length peptide?

If the sequences were ABC, DEF, and GHI, they can only be joined in 6 different possible ways to make a nonapeptide. Each of the tripeptides can appear in the first position and can combine in two possible ways with the other two tripeptides: 3 × 2 = 6.

Ile

Isoleucine (I) hydrophobic, nonpolar R group

The results of Experiment 2 indicate that the signaling interaction at the two-cell stage probably most involves which class of macromolecules?

In Experiment 2, two-cell embryos were incubated in either cycloheximide (an inhibitor of translation) or actinomycin D (an inhibitor of transcription). The AB cells were then isolated and washed to remove inhibitors, and grown in culture. AB cells of embryos treated with cycloheximide (the translation inhibitor, which would have prevented production of proteins at the ribosomes of both AB and P1 cells) produced only neurons and skin, while AB cells of embryos treated with actinomycin D (the transcription inhibitor, which would have prevented production of mRNA) produced neurons, skin and muscle—their normal fate. These results indicate that the signaling interaction (between P1 and AB cells) at the two-cell stage probably involves protein, since proteins of the P1 cells could not have been produced to carry the necessary message(s) to the AB cells prior to isolation.

Which of the following structures is derived from the same germ cell layer as the heart?

In gastrulation, the three cell regions, or germ layers, are formed in the embryo. From the outer layer (ectoderm) come cells of the nervous system and epidermis, from the inner layer (endoderm) come cells of the lining of the digestive tube and associated organs, and from the middle layer (mesoderm) come the blood cells, connective tissues (bones, muscles, and tendons), and several organs (kidney, heart, gonads). Thus, the eye and spinal cord are formed from ectoderm, the liver is formed from endoderm, and the heart and bone are formed from mesoderm.

The scientists who developed the experimental protocol described in the passage chose TNBS over many potential candidates to label PE molecules. What characteristic about the rate of reaction between TNBS and outer envelope PE molecules allowed the experiment to provide useful data? The rate of TNBS reaction with outer envelope PE molecules is:

In order for TNBS treatment to accurately determine the state of membrane phospholipids at any given time interval, the rate of the reaction must be faster than the time it takes to establish a new membrane state. Reaction with TNBS must be very fast in order to freeze the state of the membrane so that it may be analyzed.

With which of the above metals can copper form a galvanic cell in which copper is reduced?

In order for an oxidation-reduction reaction to proceed spontaneously, the cell potential Ecell° for the net reaction must be positive. The cell potential Ecell° can be found by subtracting the standard reduction potential Ered° for the substance being oxidized from that of the substance being reduced. The question asks which metals can reduce copper ions. This means that the metals in question are oxidized. The difference in standard reduction potentials Ecell° from the table is positive for lead and zinc but negative for silver.

Why is the Ames test for mutagens used to test for carcinogens?

In the Ames test, the chemicals that cause mutations in Salmonella test strains are possibly carcinogens, due to the fact that they mutate DNA and DNA mutations can cause cancer (B). The mutagens are transforming the Salmonella, and not vice versa (A). While Salmonella may contain genes that are oncogenes, the statement doesn't explain the relationship between mutagens and carcinogens that allows the Ames test to be used to detect carcinogens

What are the oxidizing and reducing agents, respectively, in the reaction below? 2HCl + H2O2 + MnO2 → O2 + MnCl2 + 2H2O

In the reaction pictured, Mn is reduced from +4 to +2; therefore, MnO2 is the oxidizing agent. O is oxidized from -1 in H2O2 to 0 in O2; therefore, H2O2 is the reducing agent. Thus, D is the best answer MnO2, H2O2

In the reaction shown in Equation 1, Al(OH)3 acts as what kind of acid or base?

In the reaction shown in Equation 1, the Al in Al(OH)3 accepts an electron pair from the OH- of NaOH and is therefore acting as a Lewis acid. Thus, A is the best answer.

Which statement best applies Rogers's concept of incongruence to SDT's suggestion for how healthcare professionals can promote autonomous motivation?

Incongruence refers to the gap between a person's actual self and ideal self.

Which one of Piaget's cognitive developmental stages are all participants LEAST likely to have acquired?

Individuals 12 years and older reach Piaget's formal operational stage of cognitive development, acquiring abstract reasoning skills. Participants in the study are ages 10-13 years. Thus, not all participants will already have acquired formal operations

According to Freud, participants in studies 1 and 2 are in which stage of development?

Infants in the studies were 3 to 4 months old and 6 to 7 months old. According to Freud, the oral stage occurs from birth to 12-18 months and is characteristic of interest in oral gratification from sucking, eating, and biting.

Which of the following conclusions about healthcare delivery is best supported by the research in the passage?

Institutional discrimination refers to different treatment between social groups at the system level that generally results from subtle (and possibly unintentional) practices rather than blatant (and intentional) discriminatory actions. However, the study cannot directly prove racial prejudice among providers because it did not assess provider attitudes. C. The study provides evidence of institutional discrimination in revascularization use, but cannot prove racial prejudice in provider attitudes.

Compared to micellular Compound 1, Compound 2 is structurally more rigid as a result of what type of interaction?

It can be reasoned that the interaction described is intermolecular in nature, since multiple molecules of micellular Compound 1 come together to form Compound 2, which is a solid. Multiple pieces of information point to the fact that the interaction is disulfide bond formation, including the fact that an oxidant causes the formation of Compound 2, which can be reversed by the addition of a reducing agent

Hill coefficient

It describes the fraction of the macromolecule saturated by ligand as a function of the ligand concentration; it is used in determining the degree of cooperativeness of the ligand binding to the enzyme or receptor. n>1 positive cooperative binding n<1 negative cooperative binding n=1 noncooperative binding

What is the value of KM (in µM) for the WT enzyme?

KM is the concentration of substrate at which the kinetics experiment reaches half the maximum velocity and this can be read from the graph shown in Figure 1

A student plans to add HCl to a solution containing Pb(NO3)2(aq). To determine how much Pb2+ will precipitate from solution when the HCl is added, the student needs to know which of the following?

Ksp for PbCl2 o determine how much Pb2+ will precipitate, the student must know the solubility of PbCl2, which is related to the solubility product constant Ksp of PbCl2

Leu

Lucine (L) hydrophobic, nonpolar R group

Which statement best explains meritocracy in terms of status? Meritocracy occurs when selections are made based on:

Meritocracy is when appointments, selections, and responsibilities are assigned to individuals based on merit - intelligence, skills, credentials, and other similar factors. Merit is thus derived from achievements. The option pairing that best explains meritocracy is B, achieved status over ascribed status. The options that include social status are not specific, since social status refers to honor or prestige but without an assumption of how that status was attained. Finally, master status refers to a particular status that dominates other statuses that a person might hold.

Based on the information in the passage, which immune cells would mount the initial immune response to N. meningitidis that results in meningitis?

Microglia are phagocytotic innate immune cells specific to the brain. The other cells are adaptive immune system cells and require activation by microglia in order to mount an immune response.

During Reaction 2, did the oxidation state of N change?

NH3(aq) + CO2(aq) + H2O(l) → NH4+(aq) + HCO3-(aq) No; it remained at -3 The part of Reaction 2 that involves nitrogen is the protonation of ammonia (NH3 + H+ → NH4+). Acid-base reactions do not involve oxidation state changes. The oxidation state of N in NH3 is -3. Each H is +1 and is balanced by the -3 of N to make a neutral compound. The oxidation state of N does not change when the N is protonated.

Most bacterial cells and human cells are alike in:

Of the options listed, only A, the ability to produce ATP via ATP synthase is common to both bacterial and human cells. Both types of cells possess a membrane-embedded electron transport chain capable of generating a H+ gradient, which drives synthesis of ATP via ATP synthase.

Of the following tissues, which is NOT derived from embryonic mesoderm?

Of the tissues listed, the only one that does not arise from the mesoderm during embryonic development is nervous tissue (D). Nervous tissue arises developmentally from ectoderm, not mesoderm. Thus, D is the best answer.

How much sodium hydroxide is needed to completely saponify a triacylglycerol?

One hydroxide ion is required to hydrolyze one ester linkage of a triacylglycerol molecule. Because there are three ester linkages in a triacylglycerol, three equivalents of sodium hydroxide will be needed to completely saponify the triacylglycerol.

Which of the following conclusions regarding virion biology is supported by information given in the passage?

One of the characteristics of viruses is that they are obligate intracellular parasites. They are unable to reproduce without the aid of the host cell's metabolic machinery. The passage describes the attempt to grow the particles on "sterilized, noncellular growth media." The failure to grow the unknown agent on such media supports the idea that they are viruses, but does not prove it (many bacteria are also difficult to grow on artificial media). The answer is therefore D that viruses (virions) are obligate parasites.

In mammals, which of the following events occurs during mitosis but does NOT occur during meiosis I?

One of the key differences between mitosis and meiosis occurs during their respective anaphases. During anaphase of mitosis, sister chromatids are pulled apart at the centromeres, each becoming an independent chromosome in the two diploid daughter cells. During anaphase I of meiosis I, homologous pairs of chromosomes are separated into the two daughter cells. However, each chromosome still consists of two sister chromatids joined to each other at the centromere. It is not until anaphase II of meiosis II that the centromere is split and the sister chromatids separate. The splitting of centromeres

Phe

Phenylalanine (F) hydrophobic, nonpolar R group Aromatic

Which of the following must be known in order to determine the power output of an automobile?

Power is defined as the rate of doing work. For the automobile, the power output is the amount of work done (overcoming friction) divided by the length of time in which the work was done.

Like other retroviruses, HIV contains reverse transcriptase, an enzyme that converts the viral genome from:

Reverse transcriptase converts RNA into DNA.

What is the molar concentration of Na+(aq) in a solution that is prepared by mixing 10 mL of a 0.010 M NaHCO3(aq) solution with 10 mL of a 0.010 M Na2CO3(aq) solution?

Since 1 equivalent of NaHCO3 provides 1 equivalent of Na+, the molar concentration of Na+(aq) in 0.010 M NaHCO3(aq) solution is also 0.010 M = 0.010 mole/L. The molar concentration of Na+(aq) in 0.010 M Na2CO3(aq) solution is 0.020 mol/L since 1 equivalent of Na2CO3 provides 2 equivalents of Na+. When equal volumes of these two solutions are mixed, the resulting molar concentration is equal to their average, (0.010 mol/L + 0.020 mol/L)/2.

The intermolecular forces that exist among the molecules of NH3 gas are

Since NH3 is a permanent dipole, it will exhibit dipole-dipole intermolecular forces in addition to the London dispersion forces exhibited by all molecules.

The antibiotic penicillin has the effect of inhibiting the production of the chemical peptidoglycan. Therefore, penicillin is likely to be most effective in treating infection by:

Since penicillin inhibits the production of peptidoglycan and peptidoglycan is a major component of the bacterial cell wall, one can conclude that penicillin will be an effective treatment for a bacterial infection

Why can the positive ions be considered to be fixed during the electrons' oscillations?

Since the ions are thousands of times more massive than the electron, answer B is justified (the hydrogen ion is the lightest ion and is nearly 2000 times more massive than the electron). An ion with so much mass compared to an electron will not be able to respond quickly because of its inertia.

How many hydrogen bonds are present in the two types of G•T mismatches discussed in the passage?

Since the wobble base pair appears as an offset A•T pair, it has two hydrogen bonds. The G•C mimic mismatch would have three hydrogen bonds. This can be directly inferred from it mimicking the G•C pair or by examination of the enol T structure

A researcher taking the perspective of social epidemiology is most likely to focus on:

Social epidemiology emphasizes how social factors, such as class or race/ethnicity, affect the distribution of health and disease. As a subfield of epidemiology, social epidemiology has significant overlap with sociological perspectives on health and disease. Both types of research call attention to the ways in which health and disease are conditioned by the social context.

Sociologists describe the health benefits of social support as resulting from all of the following EXCEPT:

Social support refers to social network ties (friends, family, and other relationships) that provide an individual with various types of assistance, which are associated with improving health or reducing harm. All of the response options except for D address the health benefits of networks. D. social activities that help individuals to avoid loneliness and boredom.

A single point mutation in a gene results in a nonfunctional protein. Individuals heterozygous for this mutation were identified using a Southern blot. Which pair of wild-type (WT) and mutant alleles most likely contains the mutation?

Southern blot uses a restriction digest to differentiate between mutant and wild-type alleles. In order for a Southern blot to be useful, the mutation should either create or eliminate a restriction site, most of which are palindromes and 4 to 6 base pairs long. The mutation shown in this option is the only one that disrupts a palindromic sequence, AAGCTT. This sequence is the recognition sequence for HindIII.

Assume that the side of the water tank is punctured 5.0 m below the top of the water, and that atmospheric pressure is 1.0 × 105 N/m2. What is the approximate speed of the water flowing from the hole?

Take point 2 to be at the location of the puncture and point 1 to be at the upper surface of the fluid. At point 1 above the fluid the pressure P1 is the atmospheric pressure and at point 2 outside the puncture the pressure P2 is also atmospheric pressure, so these terms cancel in Bernoulli's equation for this situation. Also note that the speed of water exiting the small puncture will be much larger than the speed of the upper water level falling at point 1. Thus, v2 >>v1 and v1 may be ignored. Canceling the pressures and setting v1 = 0 leaves the approximate result: ½ ρv22 + ρgy2 = ρgy1, which may be solved to yield v2 = [2g(y1 - y2)]1/2. The examinee is given that the height difference y1 - y2 is 5.0 m, hence, v2 = [2 (9.8 m/s2) (5.0 m)]1/2 = [98 m2/s2]1/2 ≈ 10 m/s.

A sparingly soluble metal hydroxide, M(OH)2 has a molar solubility of S mol/L at 25°C. Its Ksp value is:

The Ksp for a substance, AaBb, equals [A]a[B]b. The Ksp for M(OH)2 = [M][OH-]2. If the solubility of M(OH)2 is S mol/L, then [M] = S mol/L and [OH-] = 2S mol/L. The Ksp = S(2S)2 = S(4S2) = 4S3.

Which nucleotide pairing(s) would be recognized by the MMR system during DNA replication? dTMP and dCMP dGMP and dAMP dAMP and dTMP

The MMR system recognizes and repairs nucleotide mismatches during DNA replication. The nucleotide pairs dTMP and dCMP (I) and dGMP and dAMP (II) are mismatches. dAMP and dTMP (III) is a correct pairing.

Which of the compounds shown below is more soluble in water? (CH3CH2)2NH(CH3CH2)2CH2Compound ICompound II

The N-H bond in Compound I would be capable of forming hydrogen bonds with water making it more soluble that the non-polar Compound II.

Assume that cellular uptake rates and drug delivery rates of compounds 1 and 2 are identical. If the proposed mechanism of sensitization by Compound 1 is correct, what cancer cell treatment protocol is most likely to produce the most apoptosis 20 h after treatment?

The NF-κB signaling pathway is anti-apoptotic and is initiated by degradation and release of the inhibitor binding protein IκB. Once started, this process is not easily reversed. If the chemotherapeutic agent is administered at the same time as the sensitization agent, there is good reason to believe that some cells will be exposed to Compound 2 prior to Compound 1 as a result of random probability. The researchers found that administration of Compound 1 prior to Compound 2 by 0.5 h provided benefit.

Which action requires a larger absolute value of work: lifting the weight from A to B with constant speed, or lowering the weight from B to A with the same constant speed?

The absolute value of the work done is mgΔh where Δh is positively defined, and because none of these values changes in magnitude when the mass goes up or down, these actions involve the same amount of work

When aqueous solutions of the various anions and cations were mixed, precipitates formed because:

The amount of a substance that will dissolve in water is described by the Ksp. The Ksp for a substance, AaBb, equals [A]a[B]b. If the amount of the compound present is in excess of the Ksp, then a precipitate would form to maintain the Ksp.

under normal conditions, the tubular load of glucose (the amount/min that filters into the kidney tubules) is approximately 125 mg/min. The amount of glucose in the urine under these conditions is approximately:

The amount of glucose in the fluid in the capsular space as it enters the tubule system would be the same as the amount of glucose in the plasma, but by the time the fluid reaches the distal tubule much of the glucose will have been reabsorbed. If the amount being filtered per minute is less than Tm, all of it will have been reabsorbed. If it is more than Tm, then the amount will be linearly related to the amount in the plasma. In the case proposed in this question, the amount being filtered per minute (125mg/min) is less than the Tm for glucose given in the passage (320 mg/min), so all of the glucose should be reabsorbed. The correct answer, therefore, is 0 mg/min, answer choice A.

Sites I and II are most likely lined with the side chains of which amino acids?

The answer is A because sites I and II recognize hydrophobic and anionic substrates. It is therefore most likely that positively charged and hydrophobic side chains point into the binding pocket to make favorable interactions. This is a Scientific Reasoning and Problem Solving question because you must reason about the structure of the binding pocket based on the structure of the ligands and a scientific model of ligand binding. A: R and L

Pellagra also results from a deficiency of nicotinamide, which is synthesized from tryptophan. Nicotinamide nucleotides are neither oxidized nor reduced during which step of cellular respiration?

The answer is B because NAD+ is the oxidized form, and NADH is the reduced form of nicotinamide adenine dinucleotide. NAD+ is neither reduced to form NADH, nor is NADH oxidized to form NAD+ specifically during chemiosmosis.

Upon activation, p65 and cRel control the level of IL-6 mRNA by:

The answer is B because both p65 and cRel function as transcription factors, which bind DNA and subsequently recruit RNA polymerase. b.Binding DNA

When choosing a buffer to use for an experiment conducted at pH 5.3, it would be best to choose one with a pKa of:

The answer is B because good experimental design protocols state that a good buffer has a pKa within 1 pH unit of the desired experimental conditions. This is a Reasoning About the Design and Execution of Research question because it requires you to understand the appropriate tools for conducting research under a particular condition.

Addition of laforin to the reaction mixtures used to generate the data in Figure 2 will result in:

The answer is C because laforin is an enzyme that removes phosphate from glycogen. Presumably, the removal of phosphate prior to gel electrophoresis will result in a smaller band corresponding to 32P incorporation. . a decrease in the intensity of the band representing WT without glucosidase when visualized with 32P decay.

Based on the data in Table 1, the binding site of CPFX on BSA was determined to be:

The answer is C because warfarin competes for binding of CPFX to BSA in the amount of more than 50%. Therefore, the binding of CPFX to BSA occurs primarily at Site I. This is a Data-based and Statistical Reasoning question because you must look at the data in Table 1 and use your knowledge of protein ligand binding to evaluate the results of the experiment regarding CPFX binding to BSA. C: primarily at Site I

What is the net charge of both pantothenate and phosphopantothenate in aqueous solution at pH 7?

The answer is C because, in water at pH 7, the carboxylic acid of pantothenate will lose a hydrogen ion to become a carboxylate (−1) for a net charge of −1 on pantothenate. The phosphate group of phosphopantothenate will lose two hydrogen ions (−2) and the carboxylic acid will lose one hydrogen ion (−1) for a net charge of −3 on the phosphopantothenate. This is a Knowledge of Scientific Concepts and Principles question because you must recognize the acidic groups on the molecules (given the structure) and determine what ionization state they will be in at pH 7. C: −1 for pantothenate and −3 for phosphopantothenate

The stabilization of pantothenate in PanK3 is most likely due to an active site

The answer is C because, of the choices listed as response options, only aspartate has the necessary carboxylate side chain. This is a Knowledge of Scientific Concepts and Principles question because you must recognize the presence of carboxylate groups in amino acid side chains.

Researchers used reverse-phase high performance liquid chromatography (HPLC) (polar mobile phase and non-polar stationary phase) to separate Compound 2 from Compound 1. Which statement accurately describes this process?

The answer is D because Compound 2 is more polar than Compound 1 due to the methyl hydroxylation. In RP-HPLC, the more polar compound will elute first. This is a Reasoning about the Design and Execution of Research question because it requires you to reason about the features of a research study and predict the outcome of an experiment based on those features. D: Compound 2 will elute first because it does not interact as favorably with the stationary phase as Compound 1.

What is the minimum amount of heat energy required to increase T from 373 K to 573 K? (Note: The specific heat capacity of iron is 460 J/kg·K.)

The answer to this item is A. The heat q required is given by the relation q = mCΔT, where m is the mass of the iron, C is the specific heat of the iron, and ΔT is 200 K. The answer is q = (4 × 10^-3 kg)(460 J/kg·K)(200 K) = 0.8 × 460 J = 368 J. This question requires Scientific Reasoning and Problem Solving to arrive at the answer.

Mevalonate (Compound 2, Reaction 3) is expected to display prominent absorption in which region(s) of its infrared spectrum? 3200-3500 cm-1 1700-1750 cm-1 1580-1610 cm-1

The answer to this item is B because mevalonic acid contains both -CO2H and -OH groups, but does not contain a conjugated or aromatic C=C bond. The carboxyl group C=O stretching frequency falls in the region 1700-1750 cm-1, whereas the O-H stretching frequency is expected to fall in the region 3200-3500 cm-1. This question requires Scientific Reasoning and Problem Solving to arrive at the answer B. I and II only

What is the concentration of Cl- ions in a 0.1 M solution of calcium chloride?

The answer to this question is .20M because the formula for calcium chloride is CaCl2 and it produces twice as many Cl- ions as Ca2+ ions in solution when it dissolves. This question requires Knowledge of Scientific Concepts and Principles, the nature of ionic solutes when they dissolve in water, to arrive at the answer.

What atom is the site of covalent attachment of AMC to the model tetrapeptide used in the studies?

The answer to this question is A because AMC is attached to the peptide on the carboxyl side. This suggests that an amide linkage involving the N atom in AMC is used to covalently attach the fluorophore to the peptide.

A researcher finds a +0.38 correlation between a test of verbal intelligence and a test of spatial intelligence. Speculating about a common factor, "g," the researcher concludes that:

The answer to this question is A because a correlation of +0.38 shows a potential common factor, but with a significant degree of potential error.

Although highly toxic, amygdalin has been investigated as a potential anti-cancer compound. When refluxed in H2SO4(aq), amygdalin breaks down to produce glucose (2 equivalents), HCN (1 equivalent), and benzaldehyde (C6H5CHO, 1 equivalent). Which structure most likely corresponds to amygdalin?

The answer to this question is A because acetal and cyanohydrin functional groups will hydrolyze in H2SO4(aq). Of the four choices, only A posseses the four pieces in the correct orientation that hydrolysis will release them.

oxidation-reduction reaction

The answer to this question is A because atoms involved in Reaction 1 change oxidation states during the reaction. For instance, Mg changes from an oxidation state of 0 to +2 in converting from elemental Mg to MgCl2. This item requires Knowledge of Scientific Concepts and Principles to arrive at the answer. 2HCl(aq) + Mg(s) → MgCl2(aq) + H2(g)

In humans, the lining of which structure is NOT primarily derived from the endoderm?

The answer to this question is A because most of the lining of the mouth is derived from an invagination of ectoderm. Most of the epithelial tissues inside the body (B, C, D) are derived from endoderm. It is a Knowledge of Scientific Concepts and Principles question because it requires the examinee to recall the major structures arising out of primary germ layers

What is the work done by the patient during a 3-minute exercise on the bicycle?

The answer to this question is A because power P is work W done in a time t interval: P = W/t. Solving for W yields: W = Pt. Using the values given in the text, load P = 30 W and time t = 3 min, leads to W = 5400 J. This question requires Scientific Reasoning and Problem Solving in order to arrive at the answer.

Which amino acid was incorporated into Compound 1 as a future site of covalent attachment to HA prior to mineralization?

The answer to this question is A because serine is the residue that was phosphorylated, and phosphorylation enabled the protein to be subsequently attached to hydroxyapatite (HA)

Organic acids, denoted by HA, are only minimally deprotonated when added to pure water. HA(aq) H+(aq) + A-(aq) When dissolved in blood, however, HA fully dissociate. What factor can be used to explain this discrepancy?

The answer to this question is A because the chemical equation shows that H+ is a product of the reaction. Due to Le Châtelier's Principle, the equilibrium will favor products when the concentration of products is kept low. It is a Scientific Reasoning and Problem Solving question because you are asked to work with a theory to explain a physiological observation. A: In blood, the concentration of H+(aq) is maintained at low levels by other equilibria.

The initial filtration step in the glomerulus of the mammalian kidney occurs primarily by

The answer to this question is A because the initial filtration in the glomerulus occurs as blood pressure forces the fluid from the glomerulus into the lumen of Bowman's capsule. A. passive flow due to a pressure difference.

Based on the mode of action described for BFA in the passage, the drug would be most effective against:

The answer to this question is A because the passage states that BFA disrupts the Golgi apparatus, and of the choices, only eukaryotes have a Golgi apparatus. It is a Knowledge of Scientific Concepts and Principles question because you must recall the characteristics of eukaryotic cells.

The principal quantum number is a measure of which of the following?

The answer to this question is A because the principal quantum number n is most closely associated with the potential energy of the electron. Since potential energy is proportional to the square of the distance of two oppositely charged particles by Coulomb's Law, it is also true that n is associated with the radial "size" of the electron cloud. This question requires Knowledge of Scientific Concepts and Principles to arrive at the answer. A: Approximate radial size of an electron cloud

The cutting of introns and the splicing associated with the expression of the Factor VIII gene occur during which of the following steps in the protein-synthesis process?

The answer to this question is A because the removal of introns and splicing of the remaining pieces (exons) is a step in the production of mature mRNA from the transcript produced from DNA (pre-mRNA). It is a Knowledge of Scientific Concepts and Principles question because it requires you to recall that splicing and intron removal occur during the processing of mRNA after transcription. A. DNA → mRNA

In which phase of meiosis does nondisjunction occur?

The answer to this question is A because, by definition, nondisjunction is the failure of homologous chromosomes to separate during anaphase I of meiosis or the failure of sister chromosomes to separate during anaphase II of meiosis. It is a Knowledge of Scientific Concepts and Principles question because it requires you to communicate an understanding of what occurs during the different phases of meiosis.

Mevalonate exists in equilibrium with Compound X in aqueous solution. What is the structure of Compound X?

The answer to this question is A. Since mevalonic acid contains both -CO2H and -OH groups, the open chain acid in aqueous solution will be in equilibrium with the corresponding cyclic ester (mevalolactone). Of the two possible ring-closing reactions to form a lactone, 6-membered ring formation is thermodynamically much more preferable to a 4-membered ring alternative. This question requires Scientific Reasoning and Problem Solving to arrive at the answer.

Introduction of which amino acid substitution would result in the largest decrease in the entropic penalty associated with a protein folding into its native conformation?

The answer to this question is B because changing a surface-exposed hydrophobic residue for a more hydrophilic residue eliminates the entropic penalty associated with ordered water molecules around hydrophobic groups. It is a Scientific Reasoning and Problem Solving question because you must use the scientific model of the hydrophobic effect to propose a hypothesis about the effect of an amino acid substitution at a particular location in a protein B. Leu to Thr substitution at a surface-exposed site

Which phenomenon will an animal trainer most likely try to avoid when training a rabbit for a television commercial?

The answer to this question is B because instinctual drift is the phenomenon whereby established habits, learned using operant techniques, eventually are replaced by innate food-related behaviors. So the learned behavior "drifts" to the organism's species-specific (instinctual) behavior.

What phenomenon causes static air to be drawn into the mask when oxygen flows?

The answer to this question is B because oxygen pressure is the sum of the oxygen static pressure P and the oxygen flow pressure ρv2/2. In the area of the mask openings, Pair = P + ρv2/2, thus Pair > P. Air enters the mask because the static pressure of the air is larger than the static pressure of the oxygen in flow. This is the Venturi effect, and the mask is called the Venturi mask. B. Venturi effect

What is the frequency of the emitted gamma photons? (Note: Use Planck's constant h = 6.6 x 10-34 Js and the elementary charge e = 1.6 x 10-19 C.)

The answer to this question is B because the frequency f of a photon can be found from its energy using E = hf which gives f = E/h where h is Planck's constant (6.62 × 10-34 J·s). This question requires Scientific Reasoning and Problem Solving in order to arrive at the answer.

The scientist wanted to use antibody B clinically (to treat humans), but this proposal was rejected. Which of the following is the most logical reason for the rejection?

The answer to this question is B because the human immune system will recognize the mouse antibody as a foreign substance (antigen) and generate an immune response towards it, which may include a mild to severe allergic response. The immune response will generate antibodies against the mouse antibody, limiting its usefulness as a treatment. It is a Reasoning about the Design and Execution of Research question because it asks you to use knowledge about the adaptive immune response to evaluate the acceptability of a treatment for use in a real-world situation B: Because the antibody was generated in the mouse, repeated usage in the same patient would elicit the production of human anti-mouse antibodies.

Knowing that the speed of light in the vitreous humor is 2.1 × 108 m/s, what is the index of refraction of the vitreous humor? (Note: The speed of light in a vacuum is 3.0 × 108 m/s.)

The answer to this question is B because the index of refraction of a medium is equal to the ratio of the speed of light in vacuum to the speed of light in the medium, thus it is equal to (3.0 x 108 m/s)/(2.1 x 108 m/s) = 1.4.

In the above figure, an object O is at a distance of three focal lengths from the center of a convex lens. What is the ratio of the height of the image to the height of the object?

The answer to this question is B because the ratio of the image height to the object height is equal to the ratio of the lens-image distance to the object-lens distance. According to the thin lens equation, the distance between the lens and the image is 3F x 1F/(3F - 1F) = (3/2)F, where F is the focal length. The ratio sought is then equal to (3/2)F/(3F) = 1/2

A researcher is interested in assessing the size of a patient's brain ventricles. Which method(s) is(are) appropriate for this purpose? I. An EEGII. A CAT scanIII. A PET scan

The answer to this question is B because the researcher is interested in the structure of the brain, which can be studied through a CAT scan. b.CAT scan only

The scientist claimed that antibody B offers a better means for preventing organ injury than agents such as free radical or protease inhibitors. Which of the following reasons offers the best support for this claim?

The answer to this question is B because the results of the experiment presented in the passage show that the presence of antibody B, which is directed at subunit B, significantly reduces tissue injury. The results also suggest that the function of subunit B is to adhere the neutrophil to vascular endothelium, which occurs before the release of toxic molecules by the neutrophil. Thus, the use of antibody B would be a proactive treatment that prevents neutrophil adherence and the subsequent release of free radicals and proteases, rather than a reactive treatment, such as free radical and protease inhibitors. This supports the claim that antibody B is a better treatment. It is a Scientific Reasoning and Problem Solving question because it asks you to use knowledge about the interaction of antibodies and antigens along with information in the passage to evaluate the validity of a claim. B: Antibody B can block the initiation of events that result in the release of harmful, biologically active molecules

Which of the following experiments would provide the best supporting evidence that neutrophils are the cause of the reperfusion injury

The answer to this question is B because the variable being tested in this experiment is limited to the presence and absence of neutrophils. The experiment presented in A addresses B cells, not neutrophils. The experiments presented in C and D will only provide information about the heterodimer and the beta subunit, respectively, not the neutrophil as a whole; they will not rule out other potential mechanisms by which neutrophils might contribute to tissue damage. It is a Reasoning about the Design and Execution of Research question because it asks you to demonstrate an understanding of how to best design an experiment to test the hypothesis that neutrophils are the cause of reperfusion injury. B: Performing the ischemia/reperfusion experiment using neutrophil-depleted animals and examining whether the degree of tissue damage is reduced

If a cerebral malaria-causing infected erythrocyte, as described in the passage, lost its adhesion to endothelium, it would most likely first flow into which type of blood vessels?

The answer to this question is B because veins, not arterioles (A), capillaries (C), or arteries (D), are the first vessel type downstream of venules. According to the passage, binding of infected erythrocytes to brain venule endothelium causes cerebral malaria. It is a Knowledge of Scientific Concepts and Principles question because it requires the examinee to recall the structural organization of the circulatory system.

The ornithine decarboxylase reaction has been studied extensively by biomedical researchers. The most likely reason for the interest of these researchers is that the reaction is:

The answer to this question is B, because the passage states that in mammals and other organisms ornithine decarboxylase participates in early events in cell division. B. an early event in cell division.

Myopia is a condition of the eye where the crystalline lens focuses the light rays to a position between the lens and the retina. To correct for this condition, a thin lens is placed in front of the eye that will help to focus light on the retina. The type of lens required is:

The answer to this question is C because a diverging lens will spread out the light before it reaches the lens of the eye which will cause light to focus on a point closer to the retina. This question requires Knowledge of Scientific Concepts and Principles, the identification of examples of observations that illustrate scientific principles, in order to arrive at the answer.

What is the net charge of sT-loop at pH 7.2?

The answer to this question is C because at pH 7.2, the N-terminus will be positively charged and the C-terminus will be negatively charged. In addition, the lysine side chain will carry one positive charge and the glutamic acid side chain will carry one negative charge. 0

The average human body contains approximately 5 liters of blood. If the blood has a density of 1060 kg/m3, what is the total mass of blood in the average human? (Note: 1 L = 1000 cm3.)

The answer to this question is C because m = ρV = 1060 kg/m3 × 5 L × 1000 cm3/L × 1 m3/10^6 cm3 = 5.30 kg. This question requires Scientific Reasoning and Problem Solving, determination and use of scientific formulas to solve problems, in order to arrive at the answer.

The individual cells making up a tissue differ from single-celled organisms such as Paramecium in that only the latter:

The answer to this question is C because only the Paramecium, and not tissue cells, can survive independently for an extended period of time. Both Paramicium and tissue cells divide by mitosis (A), have subcellular organelles (B), and can metabolize nutrients (D), but are not capable of extended independent life. It is a Knowledge of Scientific Concepts and Principles question because you are comparing prokaryotic and eukaryotic cells.

What expression gives the amount of light energy (in J per photon) that is converted to other forms between the fluorescence excitation and emission events?

The answer to this question is C because the equation of interest is E = hf = hc/λ, where h = 6.62 × 10 −34 J ∙ s and c = 3 × 10 8 m/s. Excitation occurs at λe = 360 nm, but fluorescence is observed at λf = 440 nm. This implies that an energy of E = (6.62 × 10 −34) × (3 × 10 8) × [1 / (360 × 10 −9) − 1 / (440 × 10 −9)] J per photon is converted to other forms between the excitation and fluorescence events.

Why is the velocity of blood flow slower in capillaries than in arteries?

The answer to this question is C because the high number of capillaries in the body means that the total cross-sectional area of these vessels is larger than any other vessel type in the circulatory system. This causes the velocity of the blood to decrease.

What is the product of the reaction of Compound 1 (shown below) with HBr by the pathway shown in Figure 3?

The answer to this question is C because the incoming nucleophile displaces the leaving group form the opposite side of the reacting center during an SN2 reaction.

According to the passage, the pineal gland has what effect on thermogenesis in wintering natural populations of golden hamsters?

The answer to this question is C because the other responses examine the effect of high temperature exposure. The question asks about the effect of the pineal gland on thermogenesis in wintering natural populations of hamsters. According to the data in the table, BAT mass values for the Sham and Px groups, 275 mg and 287 mg, respectively, did not differ significantly, so there was no effect. It is a Data-based and Statistical Reasoning question because you form a hypothesis about the effect of the pineal gland based on the data in the table. C. No effect, because BAT mass was equivalent in Normal/Sham and Normal/Px hamsters

Assume that a circuit similar to that in Figure 2 is set up in which X = Al, Xn+ = Al3+, Y = Cu, and Ym+ = Cu2+. Which of the following reactions will occur?

The answer to this question is C because this response depicts the balanced chemical reaction that occurs when the four substances [Al(s), Al3+(aq), Cu(s), and Cu2+(aq)] are mixed. 2Al(s) + 3Cu2+(aq) → 2Al3+(aq) + 3Cu(s)

If the concentration of amino acid transport protein is increased, the transport affinity Kt of L-alanine will:

The answer to this question is C because transport affinity for a given substrate (in this case, alanine) is a property of the amino acid transport protein itself, and thus, Kt will not change as a result of there being more transport protein. In other words, the Kt value is not dependent on the concentration of the amino acid transport protein. It is a Knowledge of Scientific Concepts and Principles question because it requires you to recall what transport affinity Kt represents.

Based on the passage, CDKAL1 deficiency most likely causes which of the following to increase in mouse β cells?

The answer to this question is C because, according to the passage, CDKAL1 deficiency prevents the synthesis of ms2t6A, which is necessary for the accurate translation of lysine codons. This would cause aberrant transcript processing during insulin synthesis, as the transcript contains two lysine codons. Mistranslation of codons, which can result in the incorporation of incorrect amino acids, would most likely increase protein misfolding. It is a Scientific Reasoning and Problem Solving question because it asks you to use knowledge about the relationship between the primary structure of proteins and protein folding in order to propose a hypothesis. C. Misfolding of proinsulin

A neutrophil has point mutations in the genes coding for the alpha and the beta subunits of the adhesion receptor. However, this cell can still migrate through endothelium. Which of the following conclusions about the effect of this mutation can be drawn?

The answer to this question is C because, according to the passage, neutrophil migration into endothelium occurs after adhesion to endothelium. Thus, this cell must still be able to bind to endothelium, suggesting that the mutation either does not affect the cell's adherence ability or enhances it. It is a Scientific Reasoning and Problem Solving question because it asks you to combine the knowledge that mutations can affect protein function with information in the passage to explain why a neutrophil mutant retains its ability to migrate through endothelium. C: The cell can bind to endothelium.

Which of the following statements explains why the boiling point of HF is abnormally high when compared to the boiling points of other Group 7A hydrides?

The answer to this question is C. Hydrogen bonding is a special type of intermolecular force that happens when H is bonded to highly electronegative elements such as N, O, and F. Since hydrogen bonding is a higher than anticipated dipole-dipole interaction, it causes substances that exhibit it to have higher than expected boiling points and lower vapor pressures. This question requires you to bring together theory and observation to make conclusions and is a Scientific Reasoning and Problem Solving question. C: HF is affected by hydrogen-bonding interactions to a much greater degree than the other Group 7A hydrides.

What is the work generated by a healthy adult who circulates 9 L of blood through the brachial artery in 10 min?

The answer to this question is D because a flow of 9 liters in 10 minutes means a flow rate of 900 mL/min, and according to the graph, it corresponds to a power of 200 W. The work is then 200 W x 600 s = 120 kJ.

By allowing STx to move from the early endosome to the Golgi, subunit B permits the toxin to bypass which organelle?

The answer to this question is D because by its presence in the early endosome, STx would be bound for the lysosome without the retrograde trafficking. It is a Knowledge of Scientific Concepts and Principles question because it requires you to recognize that evasion of degradation would most likely involve bypassing the lysosome. D. Lysosome

What is the approximate density of the wire at 473 K? (Note: The volume of the wire is 5 × 10-7 m3.)

The answer to this question is D because density is the ratio of mass to volume, which in this case is 4 × 10-3 kg/5 × 10-7 m3 = 8000 kg/m3. This question requires you to use Scientific Reasoning and Problem Solving, the correct determination and use of a scientific formula, to arrive at the answer.

When a striated muscle cell metabolizes glucose in the complete absence of O2, which of the following substances is NOT produced in a significant amount?

The answer to this question is D because in the absence of oxygen, pyruvate is converted to lactate in the cytoplasm and is not transported to the mitochondria to be converted to acetyl-CoA. It is a Knowledge of Scientific Concepts and Principles question because it requires you to recognize that the absence of oxygen will cause lactic acid fermentation to occur.

Which of the following best describes the bond that would form between the following two nucleotides if they were located adjacent to each other as shown in a single strand of DNA?

The answer to this question is D because nucleotides are linked to one another by phosphodiester bonds between the sugar base of one nucleotide and the phosphate group of the adjacent nucleotide in a way that the 5' end bears a phosphate, and the 3' end a hydroxyl group

During Trial 5, the wire was heated from 293 K to 673 K while V was held constant at 28 V. How did the current through the circuit change during this time?

The answer to this question is D because of the linear relationship between resistivity and temperature. If the temperature T increases by a factor of 673/293 ≅ 2.3, the resistance R will increase by the same amount. From Ohm's Law, current and resistance are inversely proportional, thus current will decrease by a factor of 2.3, from 4.6 A to 2 A. This question requires Scientific Reasoning and Problem Solving, determination and use of a scientific formula, in order to arrive at the answer.

The environment of the retinal binding site is most likely:

The answer to this question is D because retinal is composed of mainly carbon and hydrogen, making it largely hydrophobic.

When used in place of spHM, which peptide would be most likely to achieve the same experimental results?

The answer to this question is D because the phosphorylated threonine would most likely be mimicked by glutamic acid in terms of size and charge. FLGFTY --> FLGFEY

Which type of enzyme is responsible for activating subunit A?

The answer to this question is D because the subunit A is part of a polypeptide chain that is subject to hydrolytic cleavage. This reaction is most likely catalyzed by a protease. It is a Knowledge of Scientific Concepts and Principles question because you must recognize that hydrolytic cleavage activates the subunit and that proteases are the type of enzyme that catalyzes this reaction. D. Protease

Recombinant DNA technology allows scientists to mass-produce some human proteins by transferring human genes into bacteria. The characteristic of bacteria that would most likely prevent Factor VIII from being produced in this way is that bacteria:

The answer to this question is D because, according to the passage, the synthesis of Factor VIII requires mRNA splicing. mRNA processing does not occur in prokaryotes. Because transcription in prokaryotes occurs in the cytoplasm, ribosomes are able to bind and begin translation even before transcription is complete; prokaryotic mRNA requires no additional processing after transcription. It is a Scientific Reasoning and Problem Solving question because it asks you to use your understanding of transcription in prokaryotes to evaluate the use of recombinant DNA technology to mass-produce Factor VIII. D: lack a mechanism for splicing out introns.

In the assay presented in Figure 2, which combination of Lys529 codon (AAA or AAG) and genetic background (wild-type or ΔyqeV) resulted in the highest level of mistranslation of the codon?

The answer to this question is D because, based on the passage, levels of mistranslation of the Lys529 codon can be determined by measuring relative firefly luciferase activity (the FL to RL activity ratio). Because Lys529 is critical for FL activity, mistranslation of Lys529 will reduce FL activity. Lower FL activity reduces the FL to RL activity ratio (assuming that RL activity is not affected, or as affected, by mistranslation). According to Figure 2, the FL/RL ratio is lowest when Lys529 is encoded by AAG in the ΔyqeV background. It is a Data-based and Statistical Reasoning question because it asks you to use data from the luciferase activity experiment to draw a conclusion about the effect of a single codon change on the translation of a specific protein. D. AAG codon in the ΔyqeV background

What factor explains how a single stereoisomer is formed in Reaction 2?

The answer to this question is D since it is the only possible explanation for the observed stereospecificity of the reaction. Neither reactant is chiral, but a chiral product forms because the catalyst (the enzyme) is chiral. This question requires Scientific Reasoning and Problem Solving to arrive at the answer. D: the enzyme is chiral

An object with a mass of 0.1 kg absorbs 250 J of heat while changing temperature from 20°C to 25°C. What is the specific heat of the object?

The answer to this question is D which can be obtained by dimensional analysis of the units. (250 J/(0.1 kg·5°C) = 500 J/(kg·°C). This question requires Scientific Reasoning and Problem Solving to arrive at the answer.

Addition of which disaccharide to a solution of Ag2O in NH3(aq) will NOT result in the deposition of shiny silver mirror on the walls of the reaction vessel?

The answer to this question is Sucrose because sucrose does NOT possess a hemiacetal functional group, will NOT undergo mutarotation, and will NOT be oxidized by Ag+. Saccharides undergo mutarotation if they are capable of ring-opening. The process of ring-opening occurs at a hemiacetal group, and the intermediate is an aldehyde. Such sugars are termed "reducing sugars" as a result of their ability to be oxidized by Ag+. This question requires Scientific Reasoning and Problem Solving to arrive at the answer.

Suppose that at the end of Reaction 1 the level of the aqueous solution were 26 cm higher inside the buret than outside. Compared to ambient pressure, the pressure of the gas inside the buret would be:

The answer to this question is lower because the level of the liquid inside is higher than outside which results from the fact that the air pressure outside is higher than the pressure inside. Pressure builds on the inside by raising the liquid level. When the sum of the inside air pressure plus liquid pressure equals the outside air pressure, the liquid level stops rising (attains equilibrium). This question requires Reasoning about the Design and Execution of Research, and knowledge of a barometer and how it works, in order to arrive at the answer.

What is the maximum volume of PH3(g) that a chemist can obtain from the reaction shown by Equation 1a, if 0.005 mol LiPH2 reacts with 0.002 mol R2NBCl2 at 0oC and 1 atm?

The answer to this question relies both upon the molar ratios of Equation 1a and on the fact that, at STP, one mole of a gas will occupy 22.4 L. Equation 1a shows that mixing 2 moles of R2NBCl2 with 5 moles of LiPH2 will produce 3 moles of PH3. Under the conditions given in the question, 0.003 mol PH3 will be produced. At 0oC and 1 atm (STP), this will occupy 0.003 mol × 22.4 L/mol or 67.2 mL. Thus choice C is the correct answer.

The autonomic nerve fibers that directly innervate the heart to cause cardiac slowing are:

The autonomic nerve fibers that innervate the heart to slow it are the parasympathetic motor fibers of the vagus nerve. Sympathetic motor fibers increase heart rate. Sensory fibers carry information from the heart to the central nervous system

Suppose a defibrillator successfully returns a baby's heart to normal beating. Suppose further that 20 g of blood enters the heart at 25 cm/s and leaves 0.10 s later at 35 cm/s. What is the estimated average force on the 20 g of blood as it moves through the baby's heart?

The average acceleration is (35 cm/s - 25 cm/s)/0.10 s = 100 cm/s2 = 1 m/s2. The average force is 20 g × 1 m/s2 = 0.020 kg × 1 m/s2 = 0.020 N.

Suppose that CH4(g) reacts completely with O2(g) to form CO2(g) and H2O(g) with a total pressure of 1.2 torr. What is the partial pressure of H2O(g)?

The balanced equation for the complete combustion of CH4(g) is shown below. CH4(g) + 2O2(g) → CO2(g) + 2H2O(g) The pressure of the gaseous products is 1.2 torr. For every three product molecules, two are water. Therefore, the partial pressure of water is 2/3 the total pressure, because the total pressure is a function of the total number but not kind of molecules. Two-thirds of 1.2 torr is 0.8 torr.

An object that is totally immersed in benzene (specific gravity = 0.7) is subject to a buoyancy force of 5 N. When the same object is totally immersed in an unknown liquid, the buoyancy force is 12 N. What is the approximate specific gravity of the unknown liquid?

The buoyant force on an immersed object is the product of: (density of the liquid) × (volume of the object) × (acceleration of gravity). Forming the ratio of buoyant forces in the two cases gives: 12/5 = (density of the unknown liquid)/(density of benzene, 0.7). Solving for the specific gravity of the unknown liquid, which is the ratio of its density to that of water, gives (12/5) × 0.7 = 1.7.

An object with 15 grams mass is immersed in benzene and suffers an apparent loss of mass of 5 grams. What is the approximate specific gravity of the object? (Data: Specific gravity of benzene = 0.7.)

The buoyant force on an object immersed in a fluid is equal to the weight of the fluid displaced by the object (Archimedes' principle). There were 5 g of liquid displaced; thus, the ratio of object mass to fluid mass is 15/5 = 3. The specific gravity of the object (mass per unit volume compared with water) is three times the specific gravity of benzene (3 x 0.7 = 2.1) because the volumes of object and displaced liquid are equal.

An unknown solid weighs 31.6 N. When submerged in water, its apparent weight is 19.8 N. What is the specific gravity of the unknown sample?

The buoyant force on the solid is the difference between its weight (31.6 N) and its apparent weight in water (19.8 N). This force equals the weight of the water displaced, which, in turn, equals the product of the volume of the solid and the density of water. The specific gravity of the solid, i.e., the ratio of its weight to the weight of an equal volume of water, is then: 31.6/(31.6 -19.8) = 2.68.

Compound 1 is a stronger acid than Compound 2 because the anion of Compound 1 is better stabilized by:

The carboxylate ion formed from Compound 1 can be stabilized by hydrogen bonding with the hydroxy group. The carboxylate ion formed from Compound 2 is not in a suitable orientation to hydrogen-bond with the hydroxy group. Thus, D is the best answer.

What is the coefficient of friction between the sled and the ground when the sled and rider are to the right of Point B?

The coefficient of friction is defined as the frictional force divided by the normal force. In this case, the frictional force is given as 50 N. The normal force on a level surface is the product of the mass and the gravitational acceleration, g. The dimensionless coefficient in the case described is 50 N/(50 kg × 10 m/s2) = 0.1.

After a block began to slide, how did its speed vary with time? (Note: Assume that the tension and kinetic friction forces on the block were constant in magnitude.)

The coefficient of kinetic friction is always lower than that of static friction. Therefore there is a net accelerating force on the block once it starts to slide. A constant force on a mass produces a constant acceleration (Newton's second law). Thus, the velocity of the block increases linearly with time

Based on the description in the passage, which component of attitudes does the gender role attitude scale measure?

The cognitive component of an attitude consists of an individual's beliefs about the attitude object. The passage states that study participants were asked to indicate their level of agreement with statements that assert beliefs about gender and family life.

Which of the following compounds has the most ionic character?

The compound with the greatest difference in electronegativities between the metal and nonmetal has the most ionic character. The data in Table 1 show that electronegativities tend to decrease down a group of the periodic table. Cs would have an electronegativity of approximately 0.8 or lower and Cl has an electronegativity of 3.0. The difference between the electronegativities of these two elements is the greatest of the compounds listed.

NH3 acts as a weak base in water with Kb = 1.76 × 10-5 at 25°C. The corresponding equilibrium is shown below. NH3(aq) + H2O(l) ⇌ NH4+(aq) + OH-(aq) At 25°C, the equilibrium concentration of the NH4+ ion in a 10 M aqueous solution of NH3 would be closest to which of the following?

The concentration of NH4+ at equilibrium can be estimated using Kb and the equilibrium constant expression. Kb = 1.76 × 10-5 = [NH4+][OH-]/[NH3]. Plugging in 10 M for [NH3], which is a good approximation since very little NH3 ionizes, and noting that [NH4+] = [OH-], it is possible to solve for [NH4+]: 1.76 × 10-5 = [NH4+]2/10 → 1.76 × 10-4 = [NH4+]2 → 1.32 × 10-2 = [NH4+]

Based on the passage, the mental functioning of individuals with AS is most compatible with which approach to intelligence?

The correct answer is B. The passage states that individuals with AS have difficulty understanding emotions and mental states of others but have normal language and cognitive skills. This is compatible with Gardner's suggestion that people have different types of intelligences. Spearman's idea of a general intelligence wouldn't predict that some aspects of intelligence would be impaired by a disorder while others remain intact. To make judgment regarding Galton's idea of hereditary genius, one would need to have a more detailed account of the intelligence history of the participants' families. Binet's idea of mental age does not produce any predictions that might or might not be supported by data on AS individuals.

Country A has a relatively weak economy, a weak political infrastructure, and competes poorly in the globalized economy. Country A is also dependent on Country B, which has a much stronger economy. According to world systems theory, Country A is a:

The correct answer is C because the stem describes Country A as a periphery nation in World Systems Theory, which characterizes global inequalities in terms of core nations (more economically developed with strong governments and institutions) and periphery nations (less economically developed with weak governments and institutions). It is a Scientific Reasoning and Problem Solving question because it involves reasoning about a theory of global inequality from Sociology.

Is proactive interference expected in Alzheimer patients for the word list used in the study?

The correct answer is C. Proactive interference involves the interference of information from long term memory with new information. Figure 2 suggests that Alzheimer patients are unable to commit information to long term memory, which makes proactive interference unlikely for this population. CNo, because proactive interference requires information, previously stored in long term memory, which can impede the learning of new information

Based on the hypothesis of the study, the researcher most likely expects which finding?

The correct answer is D because the hypothesis was that intensely negative emotions would be associated with increased confidence in memory for an event. D. A negative correlation between the participants' scores on the emotion scale and their scores on the confidence scale

Given the skin conductivity results from Study 2, participants in the self-doubt condition are likely to display all of the following characteristics EXCEPT:

The correct answer is D because the increased level of skin conductivity in the self-doubt condition indicates that the sympathetic branch of the autonomic nervous system is active. One effect of sympathetic NS activation is the deactivation of normal digestive processes. Thus, peristalsis would be decreased. Alternatives A, B, and C all represent expected results of increased sympathetic NS activity. D. increased peristalsis along the digestive tract.

Based on the results of the first study, can the researchers conclude that genes play a causal role in anxiety disorders?

The correct answer is D because the relationship that is observed between the anxiety symptoms of the parents and the anxiety symptoms of the children can potentially be due to observational learning (social learning) rather than genetic inheritance. D. No because although parental history of anxiety predicts childhood anxiety, this may also be a result of social learning

The finding in the last paragraph regarding the retrieval of related words supports:

The correct answer is option A. False memories of words that are semantically or associatively related to studied words is consistent with spreading activation, which suggests that when a concept is activated, the activation spreads to related concepts. Depth of processing refers to the type of attention applied to words during encoding. The serial position effect refers to improved memory for words at the beginning and at the end of a list. The visuospatial sketchpad is proposed as a subcomponent of working memory. A. spreading activation.

Which concept is LEAST applicable to the stories of Alice and Bill?

The correct answer is option C. As recounted in the passage, the stories of Alice and Bill do not represent social reproduction, which refers to the perpetuation of inequalities through social institutions (such as education or the economy). Alice's and Bill's stories contradict social reproduction. On the other hand, Alice and Bill each show unusual success and upward social mobility. Given the success in their stories, it is likely that meritocracy (reward according to individual talent or effort) was involved. As Pathmakers, relative poverty applies to Alice's and Bill's childhood experiences in the passage.

One company sells a defibrillator for home use that uses a 9-volt DC battery. The battery is rated at 4.2 A•hr (amp•hour). Roughly how much charge can the battery deliver?

The definition of current is flow of charge per unit time. Thus, charge equals current multiplied by time, hence 4.2 A × 1 hr = 4.2 A × 3600 s = 15,120 C ≈ 15,000 C.

DNA polymerase catalyzes the replication of chromosomal DNA in bacteria as shown below. A double-stranded DNA molecule contains bases with a ratio of (A + T)/(G + C) = 3:1. This molecule is replicated with DNA polymerase in the presence of the four deoxynucleoside triphosphates with a molar ratio of (A + T)/(G + C) = 1:1. What is the expected ratio of (A + T)/(G + C) in the double-stranded daughter DNA molecule?

The double stranded daughter DNA molecule would be an exact duplicate of the parent molecule. It would have the same (A + T)/(G + C) ratio. The correct answer is therefore 3:1, choice D.

What formula is the same for compounds 1 and 2?

The empirical formula of a compound gives the relative number of atoms of each element it contains. Thus Compound 1, B2P2H2N2R4, has the same empirical ratio (1B:1P:1H:1N:2R) as does Compound 2, B3P3H3N3R6 where R = C3H7. The molecular formula represents the actual composition of the molecule. The valence-bond and structural formulas represent the actual structure of the molecule and cannot be the same for molecules with different absolute numbers of atoms.

A pump is used to force an aqueous solution through a pipe at high temperature according to Poiseuille's Law: where ΔP is the pressure difference applied by the pump, r is the radius of the pipe, L is the length of the pipe, and η is the viscosity of the solution. Which graph depicts the rate of energy consumed over time in order to maintain constant flow through a pipe subject to boiler scale?

The energy consumed by the pump is used to create the pressure difference that maintains the viscous flow through the pipe. For a constant flow to occur while the pipe's diameter decreases, the pressure difference should increase in time. According to Poisseuille's Law, at constant flow rate, the pressure difference is inversely proportional to the radius of the pipe to the fourth power. The energy used is equal to the work done in moving the fluid, hence is proportional to the pressure difference. In other words, energy is inversely proportional to the radius of the pipe to the fourth power. As the radius decreases in time, the energy used increases more due to the fourth power dependence of the radius. In other words, more power (energy per unit time) is required to maintain the same flow through a pipe with decreased radius. Hence, the only choice that depicts such a power increase that is getting larger in time is shown in graph D.

A way to make lower-toned instruments would be to use:

The equation in the passage also gives the relation between frequency and string density f α (1/r)1/2. Therefore, to lower the frequency one would increase the string density.

The phenomenon exemplified in the equilibrium equation below is called

The equation shows the migration of a hydrogen atom from an α-carbon atom to an oxygen atom. The hybridization of the α-carbon atom and oxygen atom change during the process. This is a description of tautomerism, answer choice C; the other three answer choices are terms for distinctly different processes. Thus, answer choice C is the best answer.

What is the equilibrium constant expression for Reaction 1?

The equilibrium constant expression can be formulated by the Law of Mass Action: [products]/[reactants] with each component raised to a power equal to their stoichiometric coefficient. Water, as liquid and solvent, does not appear in the expression.

A 0.5-kg uniform meter stick is suspended by a single string at the 30-cm mark. A 0.2-kg mass hangs at the 80 cm mark. What mass hung at the 10-cm mark will produce equilibrium?

The examinee is given information about a uniform 0.5 kg meter stick: (1) it is suspended by a single string at the 30 cm mark, (2) a 0.2 kg mass hangs from it at the 80 cm mark (d = 50 cm from the suspension point), and (3) to achieve equilibrium, an unknown mass m is hung at the 10 cm mark (d = 20 cm from the suspension point). The examinee seeks mass m. At equilibrium the net torque left of the suspension must equal the net torque to the right of it. The torque of force mg applied at a distance d from the suspension is mgdsinθ, with θ is the angle between the stick and the force. At equilibrium the forces are all vertical while the beam is horizontal, hence θ = 90° and sin90° = 1. Thus, the torque left of the suspension will be mg(20 cm). There are two torques right of the suspension, the hanging mass torque (0.2 kg)g(50 cm) and the weight of the meter stick's torque (0.5 kg)g(20 cm), where 20 cm represents the distance from the suspension to the center of mass of the meter stick. Thus, the net right torque is (0.2 kg)g(50 cm) + (0.5 kg)g(20 cm). Equating the left and right net torques yields m = 1 kg

Consecutive resonances occur at wavelengths of 8 m and 4.8 m in an organ pipe closed at one end. What is the length of the organ pipe? (Note: Resonances occur at L = nλ/ 4, where L is the pipe length, λ is the wavelength, and n = 1, 3, 5,...)

The examinee is given two wavelengths (8 m and 4.8 m) that yield resonance in an organ pipe closed at one end. From this information the examinee must determine the length of the pipe. The question reminds the examinee that the resonant wavelengths λ are related to the length of this pipe via: L = nλ /4, where n = 1, 3, 5... Since n increases from n to n + 2 for two consecutive wavelengths one may write for the two given wavelengths, 4L = n(8 m) and 4L = (n + 2)(4.8 m). Equating the right sides of these two equations: n(8 m) = (n + 2)(4.8 m) which is easily solved to show that n = 3. Putting n = 3 into 4L = n(8 m) yields 4L = 24 m, so L = 6.0 m.

Would methane gas (CH4) be a candidate for determination by the method described in the passage?

The experimental procedure requires a gas to be reduced at the silver anode. If methane (CH4) gas can be determined by this procedure, either H or C must be reduced. To determine whether or not carbon or hydrogen can be reduced, their oxidation states in CH4 are found by subtracting their assigned electrons from their periodic group number. There are no nonbonding valence electrons in CH4, and the bonded electrons are assigned to the more electronegative element of the two atoms that share the electrons. Since carbon is more electronegative than hydrogen, carbon is assigned all eight of the bonded electrons, and each hydrogen atom is assigned zero electrons. The group number for carbon is IVA and for hydrogen IA. Therefore, the oxidation state of carbon in CH4 is 4 × 8 = ×4, and that of each H is 1 × 0 = +1. Theoretically, each H could be reduced from +1 to 0 (i.e., to H2), but the only answer choice involving hydrogen, answer choice C, states the opposite (i.e., that hydrogen cannot be reduced), which is false.

The UV-VIS spectrophotometer used by the researchers contained a detector that had low sensitivity and was unable to measure high absorbance samples. Which approach to the experiment makes the most sense with this limitation in mind?

The experiments done at high pH should be diluted relative to those at low pH, or the path length of the high pH experiments should be decreased. To not have an absorbance reading that is below the limits of the detector, the experiments done at pH values that result in the highest absorptivity should be diluted. The alternative is to decrease the path length of the experiments that have the highest absorptivity.

Which of the following reasons best explains why it is possible to separate a 1:1 mixture of 1-chlorobutane and 1-butanol by fractional distillation?

The fact that 1-chlorobutane will have a boiling point that is substantially lower than that of 1-butanol can be rationalized from chemical principles. The molecules have similar molecular weights, but 1-butanol has a hydroxyl functional group that can participate in hydrogen bonding. Hydrogen bonding is a particularly strong force of intermolecular attraction.

Which saturated fatty acid is the most soluble in water?

The fatty acid that is the most soluble in water will have the shortest alkyl chain.

Which of the following atoms has the largest first ionization energy?

The first ionization energy is the energy required to remove an electron from the outer shell of an atom. For any given period, it is lowest when the removal of the electron results in a complete shell or subshell, and highest when the removal of the electron disrupts a complete shell or subshell. Krypton, a noble gas, has a complete outer shell of electrons and therefore has an extremely high ionization energy.

Two additional compounds were studied: NO2(OH) dissolved in water and produced an acidic solution, and Ni(OH)2 dissolved only in an acidic solution. What type of compounds were these?

The first substance is nitric acid, HNO3. Since this substance dissolves to generate an acidic solution, the bond between O and H in the structural formula NO2OH breaks when the substance dissolves, making it an oxyacid. The latter substance, Ni(OH)2, is apparently insoluble in neutral water, but will dissolve if the solution is acidic. This behavior is typical of substances that feature basic anions. The bond between Ni and O is the one that breaks when Ni(OH)2 dissolves. The hydroxide ion that is produced quickly reacts with protons in solution and cannot react again with Ni2+ to form a precipitate. This question requires Knowledge of Scientific Concepts and Principles to arrive at the answer.

An astronaut on Earth notes that in her soft drink an ice cube floats with 9/10 of its volume submerged. If she were instead in a lunar module parked on the Moon where the gravitation force is 1/6 that of Earth, the ice in the same soft drink would float:

The floating ice cube implies that its weight is balanced by the buoyant force on it Wice = mg = rfluidVsubmergedg Note that both the weight and the buoyant force are proportional to g, making the numerical value of g irrelevant to the volume of the ice cube that is submerged

Changing which of the following will change the focal length of the convex mirror in Figure 2?

The focal length of the mirror depends only on the radius of the curvature. Because there is no refraction, the index of refraction is irrelevant and the properties of Lens B will not affect the focal length of the mirror. Radius of curvature of mirror

What pair of compounds found in Table 1 can form extensive networks of intermolecular hydrogen bonds with both participating?

The formation of a hydrogen bond requires both a hydrogen bond donor: a molecule with a hydrogen bonded to a highly electronegative atom such as oxygen or nitrogen, and a hydrogen bond acceptor: an electronegative atom (such as O or N) with an available lone pair of electrons. Of the pairs listed, only glycine and methanol are able to act both as hydrogen bond donors and acceptors. Choice C is the best answer.

Ignoring stereochemistry, how many different tripeptides may exist that contain the same three amino acids as the molecule shown below?

The formula for the number of possible peptides that contain one each of n amino acids is n! (n factorial). For n = 3 (a tripeptide), n! = 3! = 3 × 2 × 1 = 6 = Answer C.

Four organic compounds: 2-butanone, n-pentane, propanoic acid, and n-butanol, present as a mixture, are separated by column chromatography using silica gel with benzene as the eluent. What is the expected order of elution of these four organic compounds from first to last?

The four compounds have comparable molecular weights, so the order of elution will depend on the polarity of the molecule. Since silica gel serves as the stationary phase for the experiment, increasing the polarity of the eluting molecule will increase its affinity for the stationary phase and increase the elution time (decreased Rf). A. n-Pentane → 2-butanone → n-butanol → propanoic acid

What thermodynamic and chemical changes (if any) occur during aspartate transamination (Figure 1)?

The free energy of the reaction as written in either direction is the same since Keq = 1, and ΔG = -RTln(Keq) and therefore ΔG = 0. When a reaction is thermoneutral ATP is neither produced, nor required for the reaction to proceed.

A stationary receiver detects a change in frequency of the signal from a jet flying directly away from it at 300 m/s. Which of the following receivers will detect the same change in frequency from a jet moving away at 600 m/s?

The frequency shift of the Doppler effect for radio waves depends on the relative motion of the source and detector. The condition that gives the same shift as the case of the stationary receiver with a moving jet is the one that has the same relative velocity between the receiver and jet, that is, the one in which the receiver moves in the same direction as the jet.

What mechanism probably would be responsible for the increased urine output induced by hypertension according to Hypothesis B?

The glomerular filtration rate is proportional to the glomerular capillary blood pressure minus the sum of the plasma osmotic pressure and the Bowman's capsule hydrostatic pressure. An increase to the systemic blood pressure would initially increase the glomerular capillary blood pressure, which would increase the glomerular filtration rate. Without a corresponding increase in the rate of tubular reabsorption of water, this would lead to an increase in urine output

The concentration of the protein cyclin rises and falls during the cell cycle as shown in Figure 1. What mechanism could account for this oscillation of cyclin protein concentration

The graph shown in the question indicates that the concentration of cyclin rises and falls in a regular manner throughout the cell cycle, reaching a peak just at the beginning of mitosis, gradually declining during mitosis, reaching a minimum at the end of mitosis, and gradually increasing during interphase. The mechanism that can best account for this oscillation in the concentration of cyclin is translation of cyclin mRNA (creating the protein from mRNA template) followed by proteolysis (destruction) of cyclin protein during mitosis.

Which of the following changes in flow rate or in solute concentrations would NOT occur if the blood inflow rate were increased, increasing the pressure in the dialysis chamber?

The illustrations in the passage help clarify the location of the blood inflow, which comes from the patient and enters the dialysis machine. A, C, and D are incorrect because they identify changes that would occur. Blood volume reaching the outflow tube would increase per unit time (A). Osmotic concentrations of proteins in the blood would increase or remain the same because proteins are generally too large to pass through the membrane (C). The filtration rate across the membrane would increase because the pressure in the chamber would be greater (D). However, the osmotic concentration of proteins would not be expected to increase in the dialysate fluid because proteins generally do not pass through the membrane.

Normally the immune system avoids attacking the tissues of its own body because:

The immune system is designed to attack foreign material in the body. It avoids attacking tissues of its own body because it suppresses cells that are specific to its own body's antigens (surface molecules that would otherwise initiate an immune response).

Which psychological theory best explains the role of community values in the preliminary studies of tobacco cessation?

The incentive theory of motivation calls attention to how factors outside of individuals, including community values and other aspects of culture, can motivate behavior.

Which historical factor primarily accounts for the projected increase in the population aged 65 and older in the United States?

The increasing share of the population over the age of 65 primarily stems from the baby boomers, the post-World War II generation in the United States and Canada. Birth rates were relatively high for almost two decades after World War II. Demographers define the baby boom generation as those individuals born between approximately 1946 and 1964. The baby boom generation is the main sociohistorical factor that explains the projection in the passage about the increasing share of the population over 65 years of age.

In eukaryotes, oxidative phosphorylation occurs in the mitochondrion. The analogous structure used by bacteria to carry out oxidative phosphorylation is the:

The inner membrane of a mitochondrion is analogous to the plasma membrane of a prokaryote. The enzymes for oxidative phosphorylation are embedded in the inner membrane. The endosymbiotic theory suggests that mitochondria are descendents of prokaryotes that were engulfed by endocytosis into a vesicle lined with a membrane derived from the cell membrane of the eukaryote host. This is the outer membrane. The inner membrane is the plasma membrane of the endosymbiotic prokaryote, so answer choice D is correct.

The intensity level of Sound B is 20 dB greater than the intensity level of Sound A. How many times greater is the intensity level of Sound B than the intensity level of Sound A?

The intensity level β in dB is defined by: β = 10Log(I/I0), where I is the intensity of the sound and I0 is the minimum intensity audible to the human ear. "Log" indicates the base 10 logarithm. Two sounds A and B would have intensity levels βA = 10Log(IA/I0) and βB = 10Log(IB/I0) which may be subtracted to yield: βB- βA = 10Log(IB/IA). The question states that βB is 20 dB larger than βA, hence βB- βA = 20 dB. Thus, 20 = 10Log(IB/IA) or Log(IB/IA) = 2. This yields IB/IA = 102 = 100.

Some of the DNA sequences that are eliminated during macronuclear differentiation (Figure 1, Step 6) may be sequences involved in:

The macronucleus does not go through the process of meiosis. Genes involved in meiosis are therefore superfluous in this genome. The correct answer is choice C, that DNA sequences involved in meiosis may be eliminated. On the other hand the primary use of the macronucleus is to provide the proteins for the cell's day-to-day functioning. The functions of transcription (choice A), translation (choice B) and ribosome production (choice D) must be coded for by macronuclear genes because they are necessary for it to direct protein synthesis.

What does the behavior of liposomes prepared from compounds 1 and 2 upon mixing indicate about the energetics of their transformations? Liposomes prepared from:

The mixing experiments demonstrate that liposomes formed from Compound 1 cannot attain their thermodynamically preferred state and are therefore under kinetic control. Mixing liposomes of different sizes of Compound 2, on the other hand, results in the formation of new liposomes which are of an intermediate size indicating that they rapidly attain their thermodynamically preferred state.

Which of the following best explains why bacterial colonies formed on Plate IV in Figure 1?

The most likely explanation for the presence of these colonies is that they are due to spontaneous mutations (C) in the DNA of the Salmonella. One can assume, based on information provided in the passage, that the air (A) and media (B) to which the bacteria are exposed are pure and free of mutagens, and thus not causing any mutations in the test strains. If DNA repair systems were activated any back-mutations that occurred would be corrected and no colonies would form on the plates

The terminal electron acceptor in the metabolic pathway responsible for the chemical changes observed when Culture A was electrically stimulated is

The muscle cells in Culture A use lactic acid fermentation to provide the energy for the contractions that result from electrical stimulation. In this process, NADH reduces pyruvate to produce lactate. Therefore, pyruvate serves as the electron acceptor in production of lactate

An electron is ejected from the cathode by a photon with an energy slightly greater than the work function of the cathode. How will the final kinetic energy of the electron upon reaching the anode compare to its initial potential energy immediately after it has been ejected?

The near equality of the photon energy and the work function means that little initial kinetic energy will be left for the electron. This initial kinetic energy is small compared to the 50 eV it will gain from the potential difference between electrodes.

What is the dependent stressor associated with the need for reassurance?

The need for reassurance is an individual characteristic that leads to the isolation. The stressor depends on this need for this isolation to manifest itself.

In which organelle of a eukaryotic cell is the pyrimidine uracil, as part of uridine triphosphate (UTP), incorporated into nucleic acid?

The nitrogenous base, uracil, combined with the sugar ribose and phosphate makes up the nucleotide uridine. It is found in RNA, but not in DNA. The corresponding DNA nucleotide is thymine. Uridine is incorporated into RNA in the nucleus where transcription of DNA into RNA takes place. RNA is manufactured in the nucleus from a DNA template.

When the number of photons incident on the cathode with energies above the value of the work function increases, which of the following quantities also increases?

The number of incident photons affects only the number of electrons, not their energies. The electron energies depend on photon energy, the cathode work function, and the potential difference between the cathode and anode.

Suppose that in a randomly mating population of mammals, 160 of its 1,000 members exhibit a specific recessive trait that does not affect viability of the individual. How many individuals in this population are heterozygous carriers of the gene that causes this trait?

The number of individuals carrying the described trait is 480. The gene frequency of the recessive trait, q, can be ascertained by taking the square root of 16/100 or 0.4. Given this frequency, the frequency of p must be 1.0 - 0.4 = 0.6. According to the Hardy-Weinberg law, the frequency of carriers is thus given by 2 pq or 2 X .4 X .6 = .48 or 48%. 48% of 1000 = 480.

What factor can account for the relative rate of Ca2+ transport across the membrane as a function of pH?

The overall rate of ion transport does not change much (consider the sum of the two rates in Table 1), but the rate of Ca2+ increases at the expense of K+ at high pH. This can only be due to deprotonation of the X groups. The overall entity maintains electrical neutrality by preferentially binding to a divalent cation.

What is the oxidation number of aluminum in Na[Al(OH)4](aq)?

The oxidation number of Na is +1, and the charge on a hydroxide ion is -1. For the compound to be neutral, aluminum would have to have an oxidation number of +3.

At the stoichiometric (equivalence) point in a titration of benzoic acid with NaOH(aq) shown by Equation 3, the pH is:

The pH at the equivalence point in any titration is the pH of the salt solution formed. At the equivalence point of the titration, the solution contains sodium benzoate and water. The passage states that benzoic acid is a weak acid; therefore, its conjugate base, the benzoate anion, is a stronger base than OH- and reacts with water to produce OH- and undissociated benzoic acid. The resulting solution then has a pH > 7, because of the hydroxide ions produced by this reaction. The reaction of an ion such as benzoate ion with water is called hydrolysis.

What is the pH of a buffer solution that is 0.2 M in HCO3- and 2 M in H2CO3? (Note: The first pKa of carbonic acid is 6.37.)

The pH of the solution can be calculated using the Henderson-Hasselbach equation: pH = pKa + log([base]/[acid]). Plugging in the values provided in the question gives pH = 6.37 + log(0.2/2) = 5.37.

The pancreas produces which of the following substances for the digestive system?

The pancreas produces several proteolytic enzymes, which are released into the small intestine where they are converted to their active forms of trypsin, chymotrypsin, and carboxypeptidase.

Opiates, such as morphine and heroin, increase impulse traffic on the PS nerves to the iris. Therefore, after a morphine injection, the pupils will be more:

The parasympathetic nerves to the iris release acetylcholine onto the circular muscles causing pupillary constriction. Because opiates increase impulse traffic along these nerves, acetylcholine release is augmented causing the pupils to constrict. The finding of constricted, "pinpoint" pupils is a diagnostic sign of opiate intoxication.

The passage's description of delivering culturally competent care advocates for:

The passage describes delivering culturally competent care as being aware of one's own cultural standards and biases while understanding that others may have different cultural standards and biases. This idea is best represented by the contrast in option B. Ethnocentrism involves relying too heavily on one's own cultural standards to judge those from another culture. Cultural relativism, on the other hand, raises awareness of the fact that cultures have different standards and biases.

Based on the information in the passage, a mature AT1 mRNA is most likely to contain a sequence coding for which genetic factor(s)?

The passage indicates that AT1 is a transmembrane protein. Transmembrane proteins enter the endomembrane system by docking at the rough ER. This is facilitated by using a signal sequence. Mature mRNAs are not likely to contain introns (B) or promoter sequences (C). A nuclear localization signal permits proteins to enter the nucleus. This is not likely for a transmembrane protein (D). A. Signal sequence

Which compound is predicted to mimic the effect of exercise in undifferentiated muscle cells in vivo?

The passage notes that MYOD1 upregulates muscle cell differentiation by induction of Cdkn1a levels. Figure 1 shows that CTS, a simulator of exercise, upregulates MYOD1 mRNA levels, inferring that it is a positive regulator of muscle cell differentiation. The figure also shows that TNF-α downregulates the MYOD1 mRNA levels, inferring that it is a negative regulator of muscle cell differentiation. Therefore, it can be predicted that an antagonist of TNF-α (a negative regulator muscle cell differentiation) would mimic the effect of exercise (a positive regulator of muscle cell differentiation).

A man is treated with low doses of an estrogen analogue to destroy an estrogen-responsive adrenal tumor. Compared to an age-matched control (no estrogen treatment), this patient's chances of developing osteoporosis will most likely be:

The passage only states that post-menopausal women show the accelerated bone loss and the acceleration slows after eight to ten years. From this it seems likely that the acceleration has something to do with withdrawal of estrogen. Administration of estrogen to men would then have no therapeutic effect on bone loss, since they should not be suffering from withdrawal of the hormone, having never had high levels. Men given estrogen should have the same chance of developing osteoporosis as a control population

Which of the following statements is consistent with the incorrect conclusion that HCl is an ionic compound?

The passage states that an aqueous solution of an ionic compound conducts electricity. Thus, C is the best answer. Conducts electricity in a 1M solution

The structure and bonding of diamond, which is formed from graphite at extreme pressures, should be similar to that of elemental:

The passage states that at extreme pressures an elemental solid assumes the structure and bonding characteristics of a heavier element in the same column of the periodic table. The structure and bonding of diamond, which is a form of carbon, would therefore be most like other elements in the same group of the periodic table as carbon. This group contains silicon and germanium

The finding that incidence rates cut across demographic groups contradicts the perception that child maltreatment is subject to:

The passage states that research shows the problem of child maltreatment cuts across demographic groups. Social stratification refers to the objective hierarchy in a society, and often more specifically addresses the class-based hierarchy. Based on the passage, the assumption that child maltreatment is stratified is not supported by the finding of incidence rates cutting across demographic groups. Thus, C is correct. The terms in the other options are not applicable to the finding in the question. Social reproduction refers to the perpetuation of inequality through social institutions. Institutional discrimination refers to treatment stemming from institutional culture or policies (rather than individual action). Cultural relativism is the evaluation of another culture based on that culture's standards, which is not relevant for the statement in the passage.

A microbe pathogen was hypothesized as the causative agent of the disease described in the passage because: I. suspicious objects were found in blood samples from ill patients.II. in vitro cultivation of the probable pathogen was difficult.III. the disease was infectious.

The passage states that suspicion of a microbial agent was established before the virion was found. Option I can not be correct because it refers to particles found in the blood. Viruses can not be cultured in media so option II can not be correct. The fact that the disease was infectious was sufficient to put forth that hypothesis. Option III is correct, the fact that the disease was infectious caused the hypothesis to be formulated.

Based on the passage, what is the primary type of interaction that RT makes with Compound 2?

The passage states that the strongest RT interactions with the inhibitors come from the side chains of valine and phenylalanine. These side chains are hydrophobic, so they would make hydrophobic interactions with the inhibitor.

What nitrogenous base would promote the formation of uric acid crystals in gout?

The passage states that uric acid is formed by the breakdown of purines to xanthine, a uric acid precursor. Guanine is one type of purine that is found in cells.

Presumably, hyperglycemia promotes cellular dehydration because:

The passage suggests that glucose release from glycogen stores raises the osmotic pressure of body fluids and dehydrates cells protecting them from freezing. The glucose acts as an antifreeze, lowering the freezing point of the frog's body fluids. Freezing is gradual in that water moves osmotically out of cells into the concentrated unfrozen extracellular fluid. Ice formation can thereby be restricted to extracellular water; ice crystal formation within cells is lethal. An animal that can permit some freezing of its body fluids and sustain life is said to be freeze-tolerant. Although glucose is an energy source and can be transported across the plasma membrane, the protection of glucose in this harsh environment is due to its effects on the diffusion of water (i.e., its osmotic effects). Water diffusion does not require ion-exchange pumps. Glucose is not exchanged for water molecules across the membrane. Thus, answer choice C is the best response.

If the frequency of the first harmonic in Figure 2a is 100 Hz, what is the period of the second harmonic?

The period T and frequency f of a tone are related by T = 1/f. If the first harmonic has a frequency of 100 Hz, then the second harmonic has a frequency of 200 Hz. The period corresponding to 200 Hz is 1/200 s-1 = 0.005 s.

Destructive interference occurs in photodiode detectors when direct and scattered light rays take paths to the photocell that differ in phase by:

The phase difference corresponding to a half of a wave is 180°. Half a wave difference in phase between two waves corresponds to destructive interference.

A reaction is designed to produce ammonia from the gas phase equilibrium of nitrogen and hydrogen. N2(g) + 3 H2(g) 2 NH3(g) Introducing a catalyst into the system will cause the amount of ammonia at equilibrium:

The position of equilibrium in a reaction is determined solely by the difference in free energy between products and reactants. A catalyst speeds the reaction by lowering the activation energy of the reaction. It can do nothing to affect the energies of products and reactants and thus cannot affect the position of the equilibrium. It only allows the reaction to reach equilibrium more quickly.

In a healthy person standing at rest, a comparison of arterial blood pressure measured in the arm with that measured in the leg shows that the pressure in the leg is:

The pressure at the bottom of a tube of height h filled with liquid of density r is given by rgh due to the weight of the liquid. This is called hydrostatic pressure. Blood pressure in the arteries between the arm and leg has a pressure difference given by this hydrostatic term, independent of blood flow or viscosity. greater because the column of blood between the arm and leg has a hydrostatic pressure

A student uses thin layer chromatography on silica plates to monitor the progress of the reaction below. Does the product have a higher or lower Rf than the starting material?

The product of the reaction contains an -OH group and is therefore more polar than the starting material. Because of the -OH group, the product interacts with silica more strongly than the starting material and thus, has a lower Rf than the starting material.

Compared to the concentration of the proteasome, the concentration of the substrate is larger by what factor?

The proteasome was present at a concentration of 2 × 10-9 M, while the substrate was present at 100 × 10-6 M. The ratio of these two numbers is 5 × 104.

What are the units for the rate constant ku discussed in the passage?

The protein unfolds by a unimolecular mechanism. The rate of unfolding has units of M/s and is given by the expression Δ[unfolded protein length]/Δt = ku[folded protein length]. The units of ku are therefore s-1.

A study is conducted in which researchers observe how physicians deal with role strain when treating patients with terminal illnesses. This study is best described as following which type of research design?

The question describes the observation of role strain (stress experienced from contradictory demands of the same social role) in physicians treating patients with terminal illnesses. This scenario refers to ethnographic research, which involves observing social interactions in real social settings. Studying the experience of role strain through observation can increase our understanding of how physicians cope with the challenging demands of extending life with interventions while accepting the reality of death.

According to the passage, the Tm represents the rate of plasma filtration that just exceeds the:

The rate of flow through the tubule at which the substance begins to be observed in the urine is Tm. At that point the rate of flow of fluid through the tubule begins to exceed the capacity of the kidney tubule cells to reabsorb the substance. This description of Tm is given only in answer choice C, the correct answer

A public health agency seeks to improve population health by motivating people to modify their health behaviors. If the agency followed the rational choice perspective, which of the following strategies would they most likely use?

The rational choice perspective assumes that individual behavior will be based on an implicit analysis of the costs and benefits of actions. The correct answer presents a strategy in which the cost of healthier food decreases while the cost of unhealthy food increases. From a rational choice perspective, individuals should then be more inclined to select the healthier foods.

The solution in Tube 1 turned dark blue more rapidly than did the solution in Tube 4, because the:

The reaction conditions for tubes 1 and 4 are identical except Tube 1 was run at 22ºC and Tube 4 was run at 12ºC. The higher the temperature, the greater the average kinetic energy of the components of the reaction.

What is the energy content in kcal of one peanut, if the temperature of 1 kg of water in a calorimeter increases by 50oC upon the combustion of 10 peanuts?

The requested value is the energy of one peanut, if it takes ten peanuts to raise the temperature of 1 kg of water 50o. This value is obtained by multiplying the mass of water by the heat capacity by the temperature increase, and dividing the result by ten. The process is represented by the following equation: (1 kg) × (1000 g/kg) × (1 cal/g oC) × (1 kcal/1000 cal) × (50oC) = 50 kcal which gives the energy provided by the combustion of ten peanuts. One peanut therefore provides 5 kcal.

Which amino acid residues were incorporated into Compound 1 to promote the adhesion of cells on the scaffold surfaces?

The residues that were engineered into the peptide for cell adhesion are arginine and aspartate as can be reasoned based on the structure of the peptide provided, and the description of the roles of the various residues provided in the passage.

Suppose that a ball is thrown vertically upward from earth with velocity v, and returns to its original height in a time t. If the value of g were reduced to g/6 (as on the moon), then t would:

The round-trip time t for a ball thrown vertically is given by t = 2v/g. If g is replaced by g/6, then t is increased by a factor of 6.

Which statement below most accurately describes the roles of the proteins actin and myosin during muscular contraction?

The sliding filament model describes the interaction of actin and myosin during muscle contraction. According to this model, neuronal impulses cause the release of calcium from the sarcoplasmic reticulum within muscle cells. The calcium then binds to troponin, a molecule that along with tropomyosin, blocks the binding sites for myosin on actin molecules. Calcium binding to troponin causes a shift in the troponin/tropomyosin complex, revealing the binding site for myosin. Myosin then binds to actin, causing a conformational change in myosin that "cocks" the head of the myosin molecule and slides the actin filament relative to myosin. ATP binds to myosin, causing it to detach from actin and "recharge" (rebend again). Bridge formed over and over

Which experimental approach can be used to analyze the metal content of soapy precipitate produced by Reaction 1? Dissolve the solid in a known volume of:

The soapy precipitate is the salt of a weak base. Addition of HCl will generate the free fatty acid which can then be neutralized by NaOH. The difference in the number of moles of HCl and NaOH required to reach the endpoint gives the number of moles of RCO2 which can then be used to calculate the mass present in the solid and the percentage of the metal content

When switch S is closed to the left, charge begins to accumulate on the capacitor. Charge cannot accumulate indefinitely because:

The two plates of the capacitor collect charges of opposite sign. As more charge arrives it is harder and harder to fill the plates until finally an equilibrium occurs, thus C is correct. c. successive charges brought to the plates are repelled by charges accumulated earlier

If ocean waves strike the shore every 3.0 s and the horizontal distance between adjacent crests and troughs is 1.0 m, what is the average speed of the waves?

The speed of a wave is given by the product of its frequency times its wavelength: v =fλ. The wave frequency is 1/(3 s); the wavelength is 2 × 1 m because a crest and the adjacent trough are a half wavelength apart. The wave speed is therefore 2 m/3 s = 0.67 m/s. Thus .67

In which of the following does sound travel most rapidly?

The speed of sound is greatest by far in a solid because of the strong intermolecular bonds and close proximity of the molecules. Because the temperatures are close to each other (and even favor iron slightly), iron will have the highest sound speed.

Which of the following procedures would be LEAST likely to prevent bacterial synthesis of the superantigen protein?

The stem asks students to identify the procedure that would be LEAST likely to prevent synthesis of the superantigen. Foils B (binding of the operator), C (binding of the mRNA), and D (insertion of a stop codon) would all be effective ways to prevent synthesis of the superantigen. Adding tRNA nucleotides would be unlikely to prevent synthesis of the antigen.

In the macronucleus, the genes for rRNA are located extrachromosomally. This suggests that the rRNA genes are:

The stem of this item states that the rRNA genes are extrachromosomal and asks what this suggests. Genes are strands of DNA, so his must mean that they are strands of DNA that are not part of the cell's chromosomes. These strands still must self-replicate just as the chromosomes do if they are to be passed from one generation to the next, so choice C is the correct answer.

What is the total mass of D-glucose dissolved in a 2-μL aliquot of the solution used for this experiment?

The stock solution consisted of 72 g of glucose in 1.0 L of solution (72 μg/μL). Therefore, 2 μL contains 144 μg, or 1.4 × 10-4 g of the solute

A gas laser has a cavity length of 1/3 m and a single oscillation frequency of 9.0 x 1014 Hz. What is the cavity mode number?

The wavelength is given by the speed (3 x 10 8 m/s) divided by the frequency (9.0 x 1014 Hz), which givesλm = 0.33 x 10-6m. Using the equation given in the passage, m = 2L/λm, gives the cavity-mode number, m = 2 x 106. Answer B is correct.

An astronomer observes a hydrogen line in the spectrum of a star. The wavelength of hydrogen in the laboratory is 6.563 x 10-7m, but the wavelength in the star's light is measured at 6.56186 x 10-7m. Which of the following explains this discrepancy?

The wavelength of the light detected from the star is smaller than the laboratory value on Earth. This implies a Doppler shift (a blue shift) associated with approaching relative velocity between the star and the Earth

Staphylococcus and Streptococcus bacteria cause problems in acute infections such as toxic shock syndrome primarily by:

There is much discussion in the passage, however, about the effects of these organisms on the immune response, that they produce superantigens and activate 1 in 20 T cells rather than 1 in 100,000. Therefore, option B is the correct answer.

In human females, mitotic divisions of oogonia that lead to formation of presumptive egg cells (primary oocytes) occur between:

These stages occur in the following order: fertilization, birth, puberty, menopause. All of the mitotic divisions that form primary oocytes occur prior to birth.

Given that antibiotics like erythromycin are metabolized by estrogen-sensitive P450 enzymes within the liver, which graph best predicts the expected relative half-life of erythromycin in adult males versus adult females?

This is the only graph that correctly illustrates a quicker metabolism—or shorter half-life—of erythromycin in females. Given the continuous presence of GH in females, the cytochrome P450 enzymes are up-regulated, resulting in more rapid metabolism of certain drugs (like erythromycin). As a result, the half-life of erythromycin is expected to be lower in females (in other words, it would take less time to metabolize the drug due to the greater presence of P450 enzymes).

The figure above shows an arrow, pointing upward, located a distance p in front of a converging lens of focal length f. If p > f as in the figure, what kind of image of the arrow is formed by the lens?

This item may be understood through a ray diagram. Ray #1, traveling parallel to the lens axis from the tip of p, will emerge from the lens and go through the focal point f2 a distance f to the right of the lens. Ray #2, passing through the focal point f1 from the tip of p, will emerge from the lens parallel to the lens axis. The rays cross at the gray image shown: a real image whose arrow tip points in the opposite direction of the original object. Thus, A is the best answer. a. real and pointing down

When the wavelength for maximum light emission of a given scintillator is 450 nm, what is the energy of the light photon?

To find the energy E of a photon from its wavelength λ , use the formula: E = hc/λ . The constants h and c are given in the passage. Therefore E = (6.6 x 10-34 J∙s) x (3 x 108 m/s)/(450 x 10-9 m) = 4.4 x 10-19 J don't forget to convert nm to m

Researchers conducted an experiment to study Weber's Law. Going from 10- to 12-pound weights created the just noticeable difference for one participant. For this participant, how many pounds need to be added to a 20-pound weight to create the just noticeable difference?

Weber's Law states that just noticeable difference is a ratio of the existing stimulus intensity. Two pounds are 1/5 of 10 pounds and they create the just noticeable difference for 10 pounds. To create the just noticeable difference for 20 pounds, 4 pounds (1/5 of 20 pounds) are needed.

Delayed ovulation, as a cause of tubal pregnancy, would most likely be associated with delayed secretion of which of the following hormones?

While the sex hormones progesterone (A) and estrogen (B) are either secreted in response to the luteinizing hormone surge or actually trigger the luteinizing hormone surge, respectively, they are not directly involved in triggering ovulation.

Some studies find that the association between discrimination and health is stronger for U.S.-born members of certain ethnic minority groups than it is for immigrant members of the same ethnic minority group. This finding suggests that the possible effect of discrimination on health is most likely related to:

With the question's focus on strength of association between discrimination and health, social integration in the U.S., option D, provides the most likely explanation. U.S.-born members of ethnic minority groups experience the system of stratification in the U.S., and its manifestations across different social institutions, throughout their lives. In contrast, immigrant members of the same ethnic minority group will have less exposure to the U.S. stratification system, and fewer experiences with U.S. social institutions (for example, consider an adult immigrant who did not go through the U.S. educational system as a child). Social segregation is a potentially relevant factor, but option A is incorrect because segregation varies across ethnic minority groups and is not closely tied to nativity status (whether a person is native or foreign born). Length of residence, option B, is superficially related to nativity status, but social integration provides a better explanation of the association between discrimination and health. Finally, place of residence, option C, is not a relevant explanation because the question does not describe geographic differences. D. social integration in the U.S.

Sound of a known frequency, wavelength, intensity, and speed travels through air and bounces off an imperfect reflector which is moving toward the source. Which of the following properties of the sound remains the same before and after reflection?

Within still air, the speed of sound remains constant.

A force F is used to raise a 4-kg mass M from the ground to a height of 5 m.

Work is the product of force and distance. The easiest way to calculate the work in this pulley problem is to multiply the net force on the weight mg by the distance it is raised: 4 kg × 10 m/s2 × 5 m = 200 J.

According to the passage, which of the following parts in the frozen body of a freeze-tolerant frog would contain ice?

You would expect to find ice in the extracellular fluid - i.e., blood plasma (II) and lymph (III) in the frozen body of a freeze-tolerant frog. However, ice in the cytoplasm (III) would be lethal because ice crystal formation within the cells disrupts structural organization. Thus, answer choice D is the best answer.

In a species of beetle, red body color is dominant to brown. Two red beetles are crossed and produce 31 red and 9 brown offspring (F1 generation). If two red F1 beetles are crossed, what is the probability that both red and brown beetles will appear in the F2 generation? (Note: Assume Mendelian inheritance patterns.)

a 3:1 ratio of F1 offspring means that the original crossed beetles are both heterozygotes, and the F1 offspring are 25% red (homozygous dominant), 50% red (heterozygous), and 25% brown (homozygous recessive). If two red F1 beetles are crossed and both red and brown beetles appear in the F2 generation, the F1 red beetles that were crossed must both be heterozygotes. The probability that, of the red F1 beetles, both were heterozygous is 2/3 × 2/3, or 4/9 (only red beetles were selected from and 2/3 of the red F1 beetles were heterozygous).

Which amino acid substitution will most likely result in upregulation of leptin signaling?

a Y985F substitution within LEPRb sequence eliminates the binding site for SOCS3, a protein that functions as negative regulator of leptin signaling as it blocks STAT3 recruitment to the LEPRb/JAK2 complex.

What is the balanced equation for the nonproductive reaction when lysine is the substrate?

a careful examination of Reaction 1 and the passage indicates that the nonproductive reaction is between NADPH and O2, with NADP+ and H2O2 as the products and FAD as a regenerated cofactor (not in the balanced equation). A. H+ + NADPH + O2 → NADP+ + H2O2

A Rac variant, in which the residue at position 61 was replaced with an alanine (Rac61A), was synthesized. Wild-type Rac and Rac61A were incubated separately with VopC. To obtain data to support that VopC modifies Rac at residue 61, the samples should be analyzed for the presence of which compound?

a deamidation reaction releases NH3, thus, if NH3 is detected in the sample containing wild-type Rac but not Rac61A, this will provide data to support that VopC deamidates Rac at residue 61.

Anomie

a sense of aimlessness or despair that arises when we can no longer reasonably expect life to be predictable; too little social regulation; normlessness

Conflict Theory

a theoretical framework in which society is viewed as composed of groups that are competing for scarce resources

Which finding would support the validity of the DMQ measure?

a valid measure is one that actually measures what it is intended to assess. Because both the DMQ (a self-report measure) and the CG (a behavioral measure) were intended to assess risky behavior, a positive correlation between the two measures would support the validity of the DMQ measure

Based on Piaget's theory of cognitive development, an 8-year-old child's cognition is likely to be characterized by all of the following EXCEPT:

according to Piaget, an 8-year-old is in the concrete operational stage and cannot engage in hypothetical reasoning, which is not mastered until the formal operational stage.

Max attends a party and does not make eye contact with, or approach, his acquaintance, Sam. According to the actor-observer bias, which graph best represents how Max and Sam view this behavior?

according to the actor-observer bias, actors attribute their own behavior to situational factors (not feeling well) whereas observers attribute actors' behavior to dispositional factors (social awkwardness)

Scientists are hopeful that NPY can be used in combination with GnRH to treat certain cases of female infertility. Individuals with a deficiency in what receptor system would be most likely to benefit from such a treatment?

according to the data in Table 1, NPY enhances the normal function of GnRH in stimulating blood LH levels. GnRH

Which experiment would best provide data to support the mechanism by which an ODN inhibits IN activity?

according to the passage, ODNs are competitive inhibitors of viral DNA for integrase. Competitive inhibition can be determined through rate experiments by applying the principles of the Michaelis-Menten equation. By keeping enzyme concentration constant, varying substrate concentration, and either including or excluding the inhibitor, the effect of the inhibitor on the Vmax and apparent Km of the reaction can be determined. A competitive inhibitor will increase the apparent Km and not affect the Vmax.

Based on the passage, an experimental feature that distinguishes the PAC technique from the classic calorimetry technique based on thermometers is that PAC:

according to the passage, PAC uses a microphone to detect the heat produced by very fast heating processes in the specific volume where light is absorbed.

The amino acid residue at position 61 in Rac is most likely

according to the passage, VopC removes an amide from the side chain of the amino acid residue at position 61 in Rac via a deamidation reaction. Of the options, only glutamine contains a side chain with an amide group

Given that the tumorigenicity of a certain mouse mammary tumor cell line is dependent on ErbB2-mediated intracellular signaling, mice injected with variants of this cell line that have which of the following modifications would be most likely to survive the longest?

according to the passage, cancerous tumors are associated with the inactivating scurfin allele, not the wild-type Foxp3 allele. Because Foxp3 regulates ErbB2 transcription and high levels of ErbB2 mRNA are found in cancerous epithelium, the best modification to a cell line would be Foxp3-expressing plasmid, to repress transcription of ErbB2. C. A Foxp3-expressing plasmid introduced

According to the passage, relative to slow-twitch fibers, fast-twitch fibers are likely to exhibit which property?

according to the passage, slow-twitch fibers are adapted for aerobic exercise which suggests they are likely, relative to fast-twitch fibers, to have increased capillary density, larger numbers of mitochondria, and higher levels of oxygen-binding proteins. The names of the fibers themselves give insight into their relative rates of contraction. Thus, it is unlikely that slow-twitch fibers will have greater Ca2+-pumping capacity, than fast-twitch fibers.

The middle region, as opposed to either end, of each of the seven α-helical domains of Frizzled is most likely to contain a high proportion of which type of amino acid residue?

according to the passage, the seven α-helical domains are transmembrane domains and, thus, cross the phospholipid bilayer. As a result, these domains are most likely to have a high proportion of hydrophobic residues. Hydrophobic residues are nonpolar A. nonpolar

A vDNA sequence encoding a protein is inserted into a host genome by IN. The protein is translated from the hypothetical mRNA sequence shown. 5-GGCAACUGACUA-3 Based on the passage, the segment of the original viral genome that encoded this protein had what nucleotide sequence?

according to the passage, viral DNA integrated into a host cell genome by integrase would originate from a retrovirus. mRNA transcribed from retroviral DNA is either used to synthesize viral proteins, or used as the RNA genome for progeny viruses. Thus, the sequence of the nucleotide in the original viral genome will be the same as that of the transcribed mRNA.

Which amino acid contains an unbranched alkyl side chain?

alanine contains a methyl side chain, which is not considered a branched alkane.

Thermal denaturation experiments can be used to follow the transition of double-stranded DNA into single-stranded DNA. Which of the following parameters affects the Tm of dsDNA in this experiment? pH of solution Ionic strength of solution Length of DNA strands

all of these parameters would affect the thermodynamic stability of the DNA double helix. Significant drops in pH would result in protonation of hydrogen-bond acceptors, leading to a loss in base-pairing interactions. The presence of positive ions in solution (particularly Mg2+) leads to stabilization of the DNA fold via shielding of the repulsion between phosphate groups within the DNA backbone. The length of DNA strands would also play a role. Longer DNA strands are held together by more hydrogen bonds, meaning that more energy is required to denature the double-stranded DNA

After the depletion of hepatic glycogen in newborns, which compounds can be used as precursors to sustain the blood glucose level? Acetyl-CoA Lactate Oxaloacetate α-Ketoglutarate

among the listed options, only lactate, oxaloacetate, and α-ketoglutarate are used as starting materials in gluconeogenesis.

long-term potentiation

an increase in a synapse's firing potential after brief, rapid stimulation. Believed to be a neural basis for learning and memory.

What additional substance is necessary for Reaction 2 to take place?

an oxidizing agent is required for Reaction 2 to proceed. Of the choices listed, only FAD is a cofactor oxidant

What is the concentration of hydroxide ion for the solutions with the highest pH that was studied?

at the highest pH tested (10) the [H3O+] is 10-10 M. This means that [OH-] is 10-4 M, because [H3O+] × [OH-] = 10-14 for aqueous solutions at 25°C

Which process moves chlorine ions into the cells of the green algae?

based on graph --> in order to maintain a higher concentration of chlorine ions inside the cell, the ions must be moved into the cell against their concentration gradient, which requires energy. This process is active transport. In the other processes, ions would move along their concentration gradient, either with or without the help of transport proteins.

Compared to WT, what is the most likely effect of the W140L substitution on the stability of the PRR-prorenin complex?

based on results presented in Table 1; in W140L variant, the replacement of tryptophan (a hydrophobic, aromatic amino acid) by leucine (a hydrophobic but non-aromatic amino acid) results in a decrease in stability of the PRR-prorenin complex as indicated by the increase in Kd.

Given that secretory lysosomes form normally in the melanocytes and CTLs of ashen and dilute mice, the data in Table 1 best support the conclusion that melanosome secretion and lytic granule secretion differ in that the secretion of lytic granules does NOT require:

based on the data in Table 1, lytic granule secretion (as indicated by killing by CTLs) remains unaffected by a mutation that inactivates myosin Va. Therefore, it can be concluded that in contrast to melanosome secretion, the secretion of lytic granules does not require myosin Va.

Based on the passage, CatB and CatL most likely act on EGP in which of the following cellular compartments to facilitate membrane fusion?

based on the passage, the entry of the virus into the host cell requires CatB and CatL proteases and involves endocytosis through the fusion of the viral membrane with the host cell membrane. Internalization of viral particles through endocytosis is mediated by endosomes. C. endosomes

Based on the information in the passage, which protein domain of STAT3 is NOT predicted to play a role in its signaling?

because based on the passage, STAT3 is a nuclear protein, which means that it requires a nuclear localization domain for nuclear translocation and a DNA binding domain for binding to regulatory regions of targeted genes. The passage also states that in addition to forming a homodimer, STAT3 associates with LEPRb/JAK2 complex which infers the presence of a protein binding domain within the STAT3 sequence. In contrast, signal sequence domains are protein domains required for proteins that are directed toward secretory pathways. I

Pairs of research participants interacted for 10 min. They rated themselves and their partners on personality traits and then rated the accuracy of their partners' ratings of them. The partners' ratings were rated as more accurate if they were close to participants' own self-ratings. This finding illustrates:

because self-verification refers to the tendency to seek out (and agree with) information that is consistent with one's self-concept.

Based on the information in the passage, what is the most likely mechanism of inheritance for HPRCC?

because the allele must be inherited in a autosomal dominant pattern in order for individuals II-1 and II-2 to produce both affected and unaffected offspring. The inheritance pattern for the offspring of these individuals is consistent with autosomal dominant if the parents are heterozygous. The rest of the pedigree is consistent with this mode of inheritance as well. In addition, the offspring of individuals II-1 and II-2 rule out autosomal recessive, X-linked recessive, and Y-linked.

According to Piaget's theory, in what stage of cognitive development are the children who are targeted by advertisements for high caloric foods?

children in Piaget's preoperational stage are typically between the ages of 2-6 or 7.

Researchers hypothesized that cocaine is pharmacologically active in the primate fetal brain when pregnant primates are administered cocaine at doses typically used by human drug users. Which experimental observation best supports their hypothesis?

cocaine is a stimulant it would have a physiological effect similar to stress and hence glucose metabolism is expected to increase. B is incorrect because transport of cocaine to the fetal circulation does not indicate whether the drug is pharmacologically active in the fetus. C is incorrect because the fetus may not be mature enough to feel pain. In any case, pain relief is associated with heroin, not cocaine. D is incorrect because, if the receptors were internalized, dopamine could not be pharmacologically active. Neurotransmitters bind to receptors on the outside, not inside, of neurons.

Certain viruses contain RNA as their genetic material. One of the ways these RNA viruses replicate themselves is to:

code for or carry a transcriptase that copies viral RNA.

Based on the data presented in Figure 1, overexpression of PRR: increases blood pressure in part through an angiotensin II-dependent pathway. increases blood pressure in part through an angiotensin II-independent pathway. decreases blood pressure in part through an angiotensin II-dependent pathway.

compared to control, the PRR overexpression alone increased ROS (an indicator of blood pressure) levels even in presence of losartan (angiotensin II antagonist), indicating that PRR overexpression induces ROS formation in an angiotensin II-independent manner. Moreover, the addition of exogenous prorenin, a ligand for PRR, induced an increase in ROS formation which can be reversed by losartan, indicating its angiotensin II-dependent nature.

One consequence of advanced malnutrition is reduced amounts of plasma proteins in the blood. This condition would most likely cause the osmotic pressure of the blood to:

decrease, resulting in an increase of fluid in the body tissues. large plasma proteins, such as albumin, in the blood increase the osmotic pressure of the blood, which in turn, increases the return of fluid to the circulatory system from the body tissues. Therefore, with malnutrition, the osmotic pressure of the blood would decrease resulting in an increase of fluid in the body tissues.

Neuroticism

degree of emotional instability or stability

Infusion of which peptide hormone will most likely prevent brain injury in newborn infants exposed to high glucose levels during their fetal development?

exposure to high glucose levels results in elevated levels of circulating insulin which prevents mobilization of endogenous glucose storage. Infusion of glucagon will result in mobilization of endogenous glucose storage thereby preventing hypoglycemia and brain injury.

When concentrated urine is being produced, in which of the following regions of the kidney will the glomerular filtrate reach its highest concentration?

glomerular filtrate is most concentrated in the medullary portion of the collecting duct, compared to the other kidney structures listed

The initial source of energy replacement in the liver of newborn infants is formed by glycosidic bonds between glucose molecules through:

glucose polymer in liver (glycogen) is formed by glycosidic bonds between glucose molecules through α(1→4) linkage linearly and α(1→6) linkage at branch point.

Several strains of STx-producing E. coli are responsible for disease. Despite the availability of antibiotics that lyse E. coli cells, the reason this is NOT a recommended course of treatment is because:

he answer to this question is B because the reason antibiotics are contraindicated for STx-producing bacterial infections is that increased release of STx will cause increased cleavage of ribosomal proteins and subsequent disruption of translation. Cell lysis will not prevent STx release. It is a Scientific Reasoning and Problem Solving question because you are required to reason about the effect of bacterial cell lysis with respect to STx release from the lysed cells, which involves predicting an outcome based on the model of STx activity. B. the bacterial lysate will disrupt host translation.

Which of the following changes would NOT interfere with the repeated transmission of an impulse at the vertebrate neuromuscular junction?

he only change that would not result in blockage of the impulse transmission is option C, which is the correct answer. An increase in acetylcholine receptors on the postsynaptic membrane would enhance transmission, not interfere with it.

Similar activation levels in which brain region were LEAST likely in S and TSD participants when they were viewing the images presented in the study?

the cerebellum is not a primary structure of the reward system

An extra S phase occurs during amitotic division in a small macronucleus to minimize fluctuations in DNA content. This is most likely triggered by the presence of:

he passage suggests that there might be some alternative to mitosis: amitotic division. When building a new macronucleus Tetrahymena does not bother with mitosis which would equally divide the chromosomes. In stead it makes an amitotic, approximate division into two uneven nuclei. An extra S (synthesis) phase results in an additional doubling of the DNA. The extra S phase ensures each nucleus contains at least one copy of each vital gene. This would minimize fluctuations only if DNA levels were low. The passage states that the extra S phase occurs only in small macronuclei. For the extra S phase to be advantageous to the organism it must, in some way, be responsive to low DNA levels, so A is the correct answer.

When the environmental temperature is 45° C, which of the following organisms will have the highest body temperature?

he question asks the examinee to identify the organism with the highest body temperature in response to elevated external temperatures. This organism will be the organism that is least able to utilize the cooling mechanisms of vasodilation and sweating. A lizard (D) has an impermeable integument, thereby eliminating vasodilation and sweating as options for cooling.

Dendrotoxin from the mamba snake blocks voltage-gated potassium channels in somatic motor neurons that regulate skeletal muscle contraction. In what way would initial exposure to dendrotoxin affect the ability of a somatic motor neuron to propagate an electrical signal in response to a stimulus?

if potassium ion channels are blocked, the membrane would fail to repolarize, extending the length of the action potential and simulating excessive muscle contractions. C. It would prolong the action potential.

To determine if a small molecule acts like a LEDGIN with respect to IN, a researcher plans to incubate purified IN both with and without the small molecule and then perform a Western blot to detect IN in each sample. Under which condition(s) should the gel electrophoresis step be performed?

if the intent is to confirm that a small molecule induces the formation of integrase tetramers from integrase dimers, it is necessary to visualize the proteins in their native state. Use of a denaturing agent will disrupt the interactions between monomers. Use of a reducing agent only will disrupt any disulfide bonds.

What experimental set-up is most likely to result in a person with a severed corpus callosum saying "ball" after a researcher asks, "What do you see?"

images projected to a visual field are represented in the opposite hemisphere of the brain. If "ball" is projected to the right visual field, the word will be represented in the left hemisphere where it could be verbalized by left-sided language centers.

Which control experiments should be included in the design of Experiment 2 to validate the results shown in Figure 2? Evaluating the protein levels of FASN and mtKAS following siRNA treatment Evaluating the siRNAs specificity for FASN and mtKAS by assessing the cellular levels of unrelated proteins Evaluating the siRNAs specificity by showing that a non-specific siRNA has no effects on the protein levels of FASN and mtKAS

in order to connect the effects of siRNA to the observed results in Figure 2, researchers must ascertain that I. siRNA treatment results in knocking down the protein levels of FASN and mtKAS; II. each siRNA is specific to its target protein and does not interfere with expression of other unrelated prpteins; and III. a non-specific scrambled siRNA does not interfere with the expression of FASN or mtKAS.

The side chain of which amino acid is most likely to form a hydrogen bond with the side chain of glutamate?

in order to hydrogen bond with glutamate, an amino acid side chain must contain a hydrogen bond donor group to match with the glutamate acceptor. Glutamine has an NH2 group that can act in this way.

What bond is cleaved by IN during the first reaction of integration?

in the first reaction of integration, integrase cleaves a dinucleotide from the 3 end of a strand of viral DNA. This involves the cleavage of a P-O bond

Researchers studied the effect of lipoic acid on mtKAS expression. The results of their investigation are shown. (Note: * indicates p < 0.05 and ** indicates p < 0.01.)

increasing the concentration of LA, a downstream product of mtKAS, results in an increase in the level of mtKAS mRNA, indicating a positive feedback loop at the level of transcription.

Which event is directly mediated by a ligand-gated ion channel?

influx of Na+ across the motor end plate occurs when Na+ ion channels bind the ligand acetylcholine.

Insulin signaling results in the phosphorylation of the downstream target, Akt, which can be quantified by Western blot analysis. Which graphic shows the effect acetate administration will likely have on Akt phosphorylation levels in WAT and muscles of WT mice?

insulin signaling is inhibited by activation of GPCR43 by acetate. This effect would only expect to be seen in WAT and not muscle cells, where GPCR43 is not expressed.

Visible light travels more slowly through an optically dense medium than through a vacuum. A possible explanation for this could be that the light:

is absorbed and re-emitted by the atomic structure of the optically dense medium

Which methods separate proteins based on their charge? SDS-PAGE Isoelectric focusing Ion-exchange chromatography Affinity chromatography

isoelectric focusing separates proteins based on their isoelectric point (the pH at which the net charge of the protein is zero) and ion exchange chromatography separates proteins based on their net charge. In contrast SDS-PAGE separates proteins based on their mass and affinity chromatography separates proteins based on their interactions with specific ligands.

A heterotrimeric protein consists of three subunits with molecular weights of 10 kDa, 40 kDa, and 100 kDa. The protein is denatured with 3 M urea and subjected to size-exclusion chromatography. Which solute is the last compound to elute from the column?

ize-exclusion column allows larger molecules to pass more freely through the column matrix and hinders the passage of smaller molecules. Urea is this smallest solute and would be the last to elute.

Assume that S. typhi immediately enters the bloodstream from the small intestine. Of the following, which would be the first major organ that bloodborne S. typhi would encounter?

liver because blood from the small intestine is transported first to the liver, which regulates nutrient distribution and removes toxins from the blood.

Which concept describes the self-concept as a product of social interaction, emerging out of the way an individual perceives others to view him or her?

looking-glass self the looking-glass self suggests that the self-concept is more than just the product of self-reflection. Instead, the way in which people see themselves is based on how they believe others perceive them during social interactions.

Liposomes derived from Compound 2 are prepared at pH 8.5 from two different solution concentrations (0.10 mM and 0.20 mM) as described in the passage. What is the expected appearance of the size-exclusion chromatograph of the liposomal suspension that results after mixing equal volumes of these?

mixing the two suspensions will result in the same liposomal suspension that is created when the lipid concentration is 0.15 mM. This is the average of the two concentrations, since the volumes were identical.

In the overall electrochemical reaction:

oxidation always occurs at the anode and reduction at the cathode of an electrochemical cell. Since nitrogen decreases in oxidation state during the reaction, it is reduced. Hydrogen, on the other hand, increases in oxidation state and is, therefore, oxidized.

Based on the passage, which metabolic pathways are upregulated during the transition from intrauterine to extrauterine environment?

passage states that the initial source for energy replacement is from hepatic glycogen indicating upregulation of glycogen breakdown (glycogenolysis). The passage also states that after hepatic glycogen stores are exhausted the glucose pool must be replenished from non-carbohydrate sources implying upregulation of the gluconeogenic pathway

The pentose phosphate pathway results in the generation of:

pentose phosphate pathway results in the generation of NADPH, which is utilized in reductive reactions.

theory of mind

people's ideas about their own and others' mental states—about their feelings, perceptions, and thoughts, and the behaviors these might predict.

Which statement describes a characteristic of FSH?

peptide hormones are hydrophilic and soluble in blood. Hormones that must bind transport proteins are steroid proteins, which are lipophilic.

When a weak acid (HA) is titrated with sodium hydroxide in the presence of an indicator (HIn), the pH at which a color change is observed depends on the

pka of HIn The indicator will change color over a specific pH range. The range at which the color change takes place depends on the point at which HIn is converted to In-, and this depends on the pKa of the indicator

Which concept CANNOT explain the ability of the participants in both studies to visually process the faces they were shown?

place theory posits that one is able to hear different pitches because different sound waves trigger activity at different places along the cochlea's basilar membrane. Therefore, it is a concept that does not involve the visual system.

If no braking occurs, a total of how much power would be required to keep the railcar moving at 40 m/s?

power required must match the work done by the friction force that tends to slow down the railcar, which is equal to the decelerating force multiplied by the constant speed, so 1000 N × 40 m/s = 40 kW

Where in the human male reproductive system do the gametes become motile and capable of fertilization?

sperm, produced in the seminiferous tubules of the testes, completes maturation and becomes motile in the epididymis

Recordings of brain wave activity revealed that the participants in the S groups spent the most time in the stage of sleep characterized by sleep spindles. This observation indicates that they were in which stage of sleep?

stage 2 of sleep is characterized by sleep spindles.

According to Mead, which behavior is an expression of the Me component of the Self?

studying for an exam instead of going to a party represents the socialized and conforming aspect of self, described by Mead as the ME component of Self.

Which statement best explains why the absorbance levels for FSH differ from those for FSHpep?

the absorbance levels show that FSH-Ab binds FSHpep at higher levels than FSH. The difference between the two is that FSH is a fully folded protein, whereas FSHpep is a peptide sequence. Thus, the most likely explanation for why FSH-Ab binds FSH at lower levels than FSHpep is because the tertiary structure of FSH interferes with FSH-Ab binding.

The energy of activation for the reaction described in the passage is given by the energy of

the activation energy for a reaction represents the minimum energy barrier necessary to be overcome by the reactants on the path to products.

Specific activity is a measure of:

the activity is a measure of the amount of enzyme per milligram of total protein. This provides a measure of the purity of an enzymatic mixture.

Under certain conditions, PKA and GSK-3 have been shown to autophosphorylate. The control group used in Experiment 2 (Lane 1) was designed to account for this possibility. Given this, the control group most likely contained

the best control against the variable of enzyme autophosphorylation would be the enzymes alone without substrate. ATP must be included so that autophosphorylation would be possible. The substrate should be excluded so that it is clear that any phosphorylation detected is due to autophosphorylation and not phosphorylation of the substrate.

What is the total amount of charge and energy, respectively, that the capacitor will store if it is connected to the battery on the railcar?

the charge is given by the relation 100 mF × 12 V = 1.2 C and the energy is equal to 0.5 × 100 mF × (12 V)2 = 7.2 J.

Based on the passage, is CatL expression sufficient for VSV-EGP infection of the mouse cell lines presented in Figure 1? (Note: In these experiments, assume that values of <3% of total cells infected are too low to be measured accurately.)

the data presented in Figure 1 indicate that compared with the control mouse cell line (bacterial; column 5), introduction of only the CatL gene into the CatB−/−, CatL−/− cell line (column 7) does not affect cell infectivity and therefore is not sufficient for VSV-EGP infection of the mouse cell lines C. No, because VSV-EGP does not infect CatB-/- cells expressing CatL better than it infects CatB-/- cells not expressing CatL

Based on the information in the passage and in Figure 1, what effect does epigenetic modification have on iM pH-dependent denaturation?

the drop in pK of the transition denotes a decrease in stability. Because DNA unfolding occurs as the pH increases, it can be inferred that this is due to cytosine deprotonation. Since 5hmC-WT has the lowest pK, hydroxymethylation decreases the stability by increasing the acidity of cytosine

Assuming the same isoenzyme concentration, which isoenzyme has the highest catalytic efficiency?

the efficiency of an enzyme is measured by kcat/KM ratio. Since Vmax = kcat[E], at constant concentration, the isoenzyme I has the highest kcat/KM ratio

According to the data in Table 1, what is one of the values of the electromagnetic energy delivered during one pulse by the ionizing radiation?

the energy delivered in one pulse is given by the product between the power and the pulse duration. According to data in Table 1, the energy of the 266 nm radiation is 7.5 µJ and that of the 325 nm is 4.4 µJ.

What is the reading of the energy meter in Figure 1 when an appropriate laser is used in PAC to dissociate a particular chemical bond?

the energy in the photochemical reaction ΔHu is the difference between the laser pulse energy Em and the heat detected ΔHnr, so the reading of the energy meter is the energy that is neither ΔHu nor ΔHnr. Based on the energy conservation, this is equal to zero for a laser used to dissociate a particular chemical bond.

An ITC experiment is conducted by injecting Compound 1 into a solution of Protein A giving large measured heats and a high affinity constant K1. A CITC experiment under identical conditions but in the presence of 10 mM Compound 2 gives the same heat curve and Kapp = K1. What change to the experimental conditions can result in measurable heat differences and Kapp < K1?

the experiment depends on Compound 2 successfully competing for the binding sites available on Protein A and this is the only response that will improve the chances of this happening.

What is the kinetic energy of a photoelectron produced in the energy meter of the PAC device when the frequency of an incident photon that is NOT absorbed in the solution is f = 5.0 × 1015 Hz? (Note: Use h = 4.1 × 10-15 eV•s.)

the kinetic energy of a photoelectron is equal to hf - 3.4 eV = 20.5 eV - 3.4 eV = 17.1 eV.

What kind of image is formed by the lenses of the glasses worn by a 68-year-old male who sees an object 2 m away?

the lenses have a negative focal length which means they are diverging lenses. Such lenses form virtual and reduced images of objects situated at distances larger than the focal length.

An inactive tetramer of IN is expected to have approximately what molecular weight?

the passage indicates that an integrase monomer is composed of 288 amino acids, which will have an approximate molecular weight of 32 kDa (the average molecular weight of an amino acid is 110 Da). Thus, a tetramer will have an approximate molecular weight of 128 kDa.

In a female mouse born with two wild-type Foxp3 alleles, what is the minimum number of inactivating, recessive mutations that might be sufficient in the Foxp3 alleles in mammary epithelium to significantly increase the likelihood that this mouse will develop mammary tumors?

the passage indicates that approximately 60% of female Foxp3sf/+ mice, which carry one inactivating scurfin (sf) allele and one wild-type (+) allele, spontaneously develop mammary tumors. Therefore, the minimum number of inactivating mutations to increase the likelihood of tumor formation is one

Based on the information from the passage, the deletion of mtKAS most likely results in decreased level of:

the passage states that deletion of the gene encoding for mtKAS results in dissipation of the electrochemical gradient generated by the electron transport chain. The electron transport chain uses the free energy from redox reactions to pump protons from mitochondrial matrix to the intermembrane space thereby generating an electrochemical gradient across the inner mitochondrial membrane. Therefore any event that causes the dissipation of the electrochemical gradient across the inner mitochondrial membrane will result in decreased level of protons in mitochondrial intermembrane space. I

A patient being treated for chronic pain receives weekly doses of morphine from the same doctor, with whom the patient enjoys conversing. On another occasion, the patient is treated at another location and finds that the same dose is ineffective. What may have caused this change in effectiveness?

the patient's regular morphine treatments that were successful were the result of the interaction of the treatment dose and the positive association with the doctor; therefore, the dose was not as effective in a new location where there was no positive association with the new doctor.

What is the ratio of the minimum sound intensities heard by a 64-year-old male and a 74-year-old female?

the relative intensities of the two sound waves are 20 dB and 40 dB, respectively. The difference is 20 dB, meaning that the decimal log of the ratio of their intensities is 2, which means that the ratio of their intensities is 100.

The relative thermodynamic stability of isomeric organic compounds can be inferred from which of the following types of experimental data?

the relative thermodynamic stability of isomers can be determined based on the amount of heat produced when the compounds are combusted; less heat, greater stability.

Which ideal solution exhibits the greatest osmotic pressure?

the solution listed has the greatest concentration of solute particles. CaCl2 dissociates into three ions, making the solution listed 0.6 M in solute particles.

In a study, each trial involves administering a drop of lemon juice to the participant's tongue and measuring the participant's level of salivation. As more trials are conducted, the researcher finds that the magnitude of salivation declines. After a certain point, the researcher switches to administering lime juice. This researcher is most likely studying which process?

the study involves reduced responding to a repeating stimulus, which is best described as habituation. The researcher then changes the stimulus, which will likely lead to dishabituation

Assume that in the study with the rat tissues, fluid flows at a speed of 0.30 mm/s through a typical capillary opening caused by a burst microbubble. Given this, which of the following is closest to the volume flow rate of fluid passing through the opening?

the volume flow rate is given by 2.5 × 104 µm2 × 0.30 mm/s = 7.5 × 106 µm3/s.

Given that the speed of sound in the rat tissues was 1500 m/s, the wavelength of the ultrasound wave used in the study was closest to:

the wavelength is given by (1500 m/s)/(2.3 MHz) = 0.65 mm.

An enzyme is more effectively inhibited by uncompetitive inhibitors when: the substrate concentration is decreased. the substrate concentration is increased. the inhibitor concentration is increased.

uncompetitive inhibitors bind their target enzymes only when the substrate is first bound to the enzyme. Since at higher substrate concentrations, the substrate-enzyme complex are more abundant, the uncompetitive inhibitor will work most effectively when the substrate concentration is the highest. Additionally, an increase in the inhibitor concentration results in increased enzyme binding and inhibition II and III

The macronuclei of the asexual progeny in Tetrahymena and the cytoplasm of the ova-producing cells of female vertebrates share a common feature in that both:

undergo uneven division When a vertebrate oocyte divides in meiosis the cytoplasm is distributed to the two daughter cells. Most of it goes to the daughter cell destined to be the ovum. The other daughter cells, the polar bodies, are cast off with little cytoplasm. The amount of nuclear material in each, however, is the same, so choice B is incorrect. The distribution of cytoplasm is uneven, as is the distribution of macronuclear DNA during the amitotic cell division in Tetrahymena. The answer is A, that the common feature is that both cytoplasmic division in ova and division of nuclear material in Tetrahymena are both uneven.

How many molecules of reduced electron carrier are generated during conversion of α-ketoglutarate to oxaloacetate in the citric acid cycle?

uring conversion of α-ketoglutarate to oxaloacetate in the citric acid cycle 2 molecules of NADH and one molecule of FADH2 are generated.

A large carbohydrate is tagged with a fluorescent marker and placed in the extracellular environment around a macrophage. The macrophage ingests the carbohydrate via phagocytosis. Which cellular structure is most likely to be fluorescently labeled upon viewing with a light microscope soon after phagocytosis?

when a macrophage ingests foreign material, the material initially becomes trapped in a phagosome. The phagosome then fuses with a lysosome to form a phagolysosome. Inside the phagolysosome, enzymes digest the foreign object. Of the cell structures listed, the labeled carbohydrate is most likely to be microscopically visualized within a lysosome (phagolysosome). C. Lysosome

Scientists have hypothesized that NPY is necessary for the generation of the preovulatory LH surge, a hormonal event that triggers ovulation. Which of the following findings best supports this hypothesis?

when the function of only NPY is blocked, the LH surge and ovulation no longer occur. This provides strong evidence that NPY function is required for these events.


Ensembles d'études connexes

SmartBook Assignment Chapter 18: Acid-Base Equilibria and Solubility Equilibria

View Set

Encountering the Old Testament, Chapter 20

View Set

Application of Systems of Equations

View Set

Acne Medications Generic and Brand Name

View Set

Chapter 15 - Stockholder's Equity

View Set

Exam 1 review (All the Mastering A&P homeworks)

View Set